Текст
                    

Московский государственный университет им, М.В. Ломоносова Учебно-научный центр довузовского образования Т.Т. Фискович ГЕОМЕТРИЯ БЕЗ РЕПЕТИТОРА (Испытание геометрической эрудиции) Москва 1998
УДК 373.167.1:514 ББК 22.151 я 721 Ф63 Рецензент: профессор В.Ф. Бутузов, заведующий кафедрой | математики физического факультета МГУ Ф63 Фискович Т.Т. Геометрия без репетитора. М.: Издат. отдел УНЦ ДО МГУ, 1998. - 152 с. ISBN 5-88800-076-0 В пособии представлен минимум учебных задач и специально ор- ганизованный теоретический материал, что позволяет увидеть пер- спективу применения общего метода к решению любой задачи. Пособие адресовано абитуриентам и предназначено для системати- зации и усовершенствования имеющихся геометрических знаний. ISBN 5-88800-076-0 © Фискович Т.Т., 1998 © Учебно-научный центр довузовского образования МГУ, 1998 © Обложка. Алмазова Е.Е., Шваров В.В., 1995
Аннотация Авторы многочисленных объемных пособий для поступаю- щих в вуз, равно как и профессионалы-репетиторы, готовящие с гарантией в данный (а иногда и в любой) вуз и, особенно, «решебники», распространяемые «по сходным ценам», предна- значены для того, чтобы за сравнительно короткий срок и так же на' короткий срок начинить резервуары кратковременной памяти образцами решения с большой вероятностью встречаю- щихся на вступительных экзаменах геометрических задач. Наше пособие преследует совсем иные цели. В нем сделана попытка систематизировать фактический материал путем пере- структурирования полученной в отдельных темах геометриче- ской информации, увидеть общность, аналогию и специфику не только в изучаемых объектах, но и смотреть на ранее изученный материал с более высокой по степени абстрактности точки зре- ния. Это позволит при дальнейшем изучении математики по- степенно приближаться к современному пониманию ее как тео- рии основных математических структур. По-новому организованный теоретический материал лучше традиционного соответствует обучению творческого поиска ре- шения задач, а не запоминания алгоритма решения их с исполь- зованием материала определенной темы. Нами отобран минимум задач, решение которых призвано: 1. Привести в готовность и использовать весь ключевой фак- тический материал геометрии средней школы. 2. Проявить известную творческую изобретательность в вы- боре и использовании при решении геометрических задач раз- личного математического аппарата. 3. Прочувствовать силу и овладеть самым общим методом решения любых задач на нахождение и доказательство. 4. Памятуя о том, что старшеклассник уже достиг в опреде- ленной степени умения обобщать и глубже проникать в сущ- ность структуры математических знаний, мы попытались не традиционно по темам (в основном, связанным с изучением объектов), а обозримо с учетом отношений между ними, дать сжатое трехблочное изложение всего фактического материала, структура которого помещена на одной странице пособия в виде прямоугольной таблицы-матрицы в 10 строк и 11 столбцов. 3
5. Не забыт и дифференцированный подход к пользователю. В контрольный минимум включены достаточно трудные, по- сильные не стандартные, охватывающие часто несколько блоков теории, задачи, которые требуют напряжения и сосредоточенно- сти. Задачи посильные для всех. Но если какая-либо из них не поддается решению, помогут задачи-спутники и специально организованный справочный материал. 6. Уделено внимание и обучению поиска решения задач и развитию перспективности мышления. 7. Определения понятий групп блоков даны так, что допус- кают дальнейшее расширение объема понятия, а значит и рас- ширение класса задач при дальнейшем обучении в вузе или са- мообразовании. В приводимом нами фактическом материале не уделено специального внимания логическому строению геометрии, ибо мы убеждены, что работа по нашей методике приведет к тому, что некоторые из нынешних выпускников будут строить аксиома- тическую теорию как решение большой теоретической задачи.
«Каждая решенная мною зада- ча становилась образцом, кото- рый служил впоследствии для решения других задач» Декарт. Рассуждение о методе. Избр. пр., с. 247. Дорогие друзья! Цель данного пособия помочь Вам переосмыслить изучен- ный Вами фактический материал по геометрии, проверить каче- ство его усвоения путем решения специально подобранного ми- нимума задач, охватывающих весь материал планиметрии и сте- реометрии, увидеть пути его дальнейшего обобщения и углубле- ния — тем самым подготовить почву для более эффективного усвоения новых знаний в дальнейшем. Вместе с тем цель предлагаемого пособия состоит и в со- вершенствовании Вашего умения решать задачи в самом широ- ком смысле слова, в выработке общего метода решения любых задач, включая моделирование реальных процессов. Ведь нау- читься решать задачи (а не запомнить их решение) — цель жиз- ни каждого. Решение школьных учебных задач с фигурами и числами — важнейший этап подготовки к решению жизненных задач и надежный критерий усвоения основных математических фактов, необходимых для дальнейшего математического образо- вания и для использования их в практической деятельности. Решать задачи... Искусство решать задачи... От чего оно за- висит? Ведь каждый из вас знает столько определений, теорем о признаках, свойствах фигур... А самих фигур, каждая из которых отличается от другой особенными свойствами... Сколько их? Пролистав учебник, ты убеждаешься, что их более сотни. А сколько теорем о них? Какие из них нужно вызвать из памяти при решении конкретной задачи? Пусть при решении задачи ты пришел к необходимости нахождения длины отрезка, скажем, бокового ребра усеченной пирамиды. Ты перебираешь в памяти, чем может быть этот отрезок в других фигурах: медианой, бис- сектрисой, высотой, стороной, диагональю, хордой, радиусом, апофемой... чего-то. И это «что-то» ты должен представить в 5
воображении, чтобы увидеть «замаскированную» фигуру (а иногда ее приходится достроить, чтобы временно использовать в задачкой ситуации). Причем, для тебя не существенно, пред- ставляешь ли ты эту фигуру по тем ее свойствам, которые за- фиксированы в определении или в теореме. Даже если ты обложишься справочниками, учебниками, решебниками, ты теряешься в поиске. Что применить? Барахта- ешься в море определений и теорем. А оказывается всю геометрию (планиметрию и стереомет- рию) можно поместить на одном листе бумаги, памятуя о том, что содержанием наполняется геометрия тогда, когда появля- ются отношения между объектами (прежде всего фигурами). Неужели помещать все 120 фигур да еще и числа (без них не решить задач на нахождение длин, площадей, объемов и т.д.). Попробуем вместе осуществить этот замысел. Надеемся, что три первые фигуры: точку, прямую и плоскость каждый отчет- ливо представляет без определения (Им и в учебниках не дает- ся определения. Они основные, неопределяемые, как и неко- торые отношения: принадлежать, лежать между, расстояние). Уже из этих фигур можно «наломать» или «нарезать» отрезки, лучи, а из последних сконструировать угол, все виды треуголь- ников, четырехугольников и вообще и-угольников. А в про- странстве — призмы, пирамиды, октаэдры — в общем, много- гранники. Но не получить шара, конуса, цилиндра и других... круглых тел. Чтобы их получить, достаточно к предыдущим добавить круг, выделив в нем окружность. Это сделать нетруд- но, имея понятие «расстояния». Так что, вообще говоря, объ- екты, изучаемые в геометрии, можно свести к четырем клас- сам: I — неопределяемые, II — полупрямая, полуплоскость, полупространство, III — угол и все «угловатые» фигуры (многоугольники, многогранники), IV — круглые тела и доба- вить V класс объектов — числа. Но так как в обучении искусству решения задач важную роль играет аналогия, обобщение, специализация, то мы да- дим несколько другое деление на классы, основанное на раз- личных способах образования одной и той же фигуры (призма — как многогранная поверхность и как частный вид цилиндрической поверхности), и, наоборот, один и тот же (или аналогичный) способ образования различных фигур (прямой круговой цилиндр, прямой круговой конус, усечен- ный конус, шар — тела вращения и их поверхности — по- верхности вращения). 6
Иногда важно для понятий угла, многоугольника и много- гранника различать фигуры, называемые одним термином, на- пример, угол как фигура, которая состоит из точки и двух лу- чей, исходящих из этой точки и как фигура, состоящая из угла и его внутренней области. То же о двугранном угле, о много угольниках и многогранниках. Различие же аналогичных поня- тий для криволинейных фигур облегчается наличием разных терминов: окружность и круг, сфера и шар. По сложившейся традиции на вступительных экзаменах по математике даются геометрические (одна или две) задачи на вы- числение (длины отрезка, меры угла, площади фигуры, объема пространственного тела). Для непосредственного ответа на во- прос задачи достаточно знаний, предусмотренных 11-ой колон- кой нашей таблицы-матрицы, даже ее частью: формулами, вы- ражающими значения геометрических величин. Их можно за- помнить, можно выписать, можно, наконец, посмотреть в спра- вочник, но искусство решения — в добывании данных для под- становки в формулу, вернее в нахождении путей к надежно за- прятанным данным. Для нахождения этих путей нужны знания и опыт применения знаний из других 10-ти колонок. Каких? — Вот где суть искусства. Решают ли задачу обучения решению задач всевозможные репетиторы, дошедшие до крайности: выпуск решебников (на коммерческой основе) даже всех 665 задач из «Сборника кон- курсных задач по математике» под редакцией М.И. Сканави? (Кстати, только 60 задач по планиметрии из 437 и 18 задач по стереометрии из 230 требуют «доказать», «установить» что-либо, а не применить формулу). Названные пособия не только не ре- шают, но вредят развитию и логического, и пространственного, и интуитивного мышления — сердцевины геометрии. Кроме единственного разумного общего совета «напиши формулу», все остальное — повелительное «проведи», «рассмотри фигуру» или «подобные фигуры»..., но как догадаться о том, что именно их надо рассматривать остается скрытым от ученика. В лучшем случае на его долю остаются алгебраические или тригонометри- ческие преобразования выражений. А ведь искусство не в том, как решить, а как увидеть, вычленить подзадачи, задачи- спутники, задачи-помощники, для решения основной задачи. Для этого, очевидно, нужна система задач, включающих в качестве основной (по постановке) задачи по применению зна- ний всех колонок нашей таблицы. 7
Работая с нашим пособием, Вам не придется перелистывать страницы учебника в поисках предположительно нужного факта для решения конкретной задачи. В предлагаемой ниже таблице указаны «адреса» нужного материала, который должен быть из- влечен из кладовой памяти, либо, в крайнем случае, из ПРИ- ЛОЖЕНИЯ 3, в котором теоретические факты также изложены в соответствии с таблицей. В I части пособия «Совершенствование культуры ума» в со- ответствии с таблицей, помещенной в ПРИЛОЖЕНИИ 1, дает- ся система упражнений, предназначенных для «разминки», «умственной гимнастики» перед решением задач. Эти упражне- ния способствуют развитию умения «видеть», «находить», «угадывать», «предполагать» как можно больше возможных отно- шений сначала между двумя, а затем и большим числом матема- тических (прежде всего, геометрических) объектов, способству- ют актуализации нужных знаний при решении задач. Эти уп- ражнения — базис искусства видеть путь решения задач. Ответы на нйх не однозначны. Они зависят от накопления фактиче- ского материала. Чем старше класс, тем больше можно вызвать из памяти отношений между геометрическими объектами. Чем больше, тем лучше, тем легче будет выбирать способ решения задачи впоследствии. С этой частью пособия хорошо работать и вдвоем-втроем, в виде игрового соревнования «Кто больше», причем, эту игру можно разнообразить: отвечать не в порядке написания упраж- нений, а на ту клетку таблицы, которая будет названа (или вы- падет по жребию): например, (5, 8), (3, 9) и т.д. При необходи- мости можно ориентировочно сверить свои ответы в разделе «Фактический материал», помещенном в ПРИЛОЖЕНИИ 3 по соответствующей нумерации. Примечание. В разделе «Важнейшие отношения между двумя объектами» мы объединяем пары (а, Ь) и (Ь, а) между двумя объектами. Например, (1, 7) — отношения отрезка и многоугольника и (7, 1) — отношение многоугольника и от- резка. После выполнения упражнений I части мы рекомендуем оз- накомиться с ПРИЛОЖЕНИЕМ 4, где кратко описана сущность каждого метода решения геометрических задач в зависимости от математического аппарата. 8
II часть пособия «Подумаем вместе* начинается с кратких советов об этапах общего метода решения любых задач. Далее в нем описаны образцы поиска решения отдельных задач, показа- ны приемы овладения искусством составления задач-спутников, помогающих найти путь к решению основной задачи, искусст- вом использования особенностей своего ума. Задачи-спутники составляются по разным принципам: задачи-близнецы, задачи- тренеры, задачи-помощники, даже спутники-провокаторы. Но все они повышают культуру ума и ведут к совершенству геомет- рического мастерства. III часть пособия «Задачи — всемогущий контролер и репе- титор* состоит из четырех разделов. Первый раздел содержит контрольный минимум задач, снабженных только ответами. В этом минимуме 50 задач, охва- тывающих весь теоретический материал планиметрии и стерео- метрии. Во втором разделе этой части даны задачи-спутники к кон- трольным задачам. Они даны под теми же номерами, что и кон- трольные, но с добавлением букв А, Б, В. Решение этих, задач, как правило, наталкивает на путь решения основной задачи. В третьем разделе этой части даны решения конечных за- дач-спутников. Четвертый раздел содержит дополнительные задачи из шкаль- ных учебников геометрии Атанасяна Л.С. и Погорелова А.В., а также из «Сборника конкурсных задач» под ред. Сканави М.И. В конце раздела даны ответы и некоторые указания путей поис- ка их решения. Особое внимание уделено обобщению фактов, подмеченных на частных примерах. В IV части пособия «Справочный материал* даны четыре приложения: ПРИЛОЖЕНИЕ 1. Таблица-матрица «Структура геометрии». ПРИЛОЖЕНИЕ 2. Аксиоматики геометрии, положенные в основу различных школьных учебников. ПРИЛОЖЕНИЕ 3. Фактический материал. ПРИЛОЖЕНИЕ 4. Методы решения геометрических задач. 9
Рекомендации к пользованию пособием Прочитав внимательно содержание текста задачи, выделите данные и искомые объекты, а также условия, связывающие их. Представьте в воображении комбинации фигур и их частей, сделайте рисунок, выделив на нем данные и искомые. Если план решения задачи готов, приступайте к его реали- зации. Решив задачу, переходите к следующей. Если и после воспроизведения теоретического материала за- дача не поддается решению, открывайте второй раздел III части пособия, где под тем же номером что и контрольная задача, но с добавлением букв А, Б, В помещены задачи-спутники, реше- ние которых, как правило, наталкивает на мысль о решении ос- новной задачи. После решения спутника А вернитесь к основ- ной задаче. В случае, когда и спутник А не решается, попытай- тесь решить Б, но каждый раз после решения задачи-спутника возвращайтесь последовательно к решению предыдущего, пока не дойдете до решения основной контрольной задачи. Желание решить задачу самостоятельно и уверенность в своих силах, соединенные с упорным трудом, непременно при- несут успех. 10
I часть 5 СОВЕРШЕНСТВОВАНИЕ КУЛЬТУРЫ УМА (умственная гимнастика, разминка для решения геометрических задач) Прежде чем приниматься за решение задач раздела «Контрольные задачи», где понадобится применение самого разнообразного фактического материала, устанавливающего свойства геометрических объектов и отношения между ними, мы предлагаем ряд упражнений, способствующих развитию умения «видеть», «находить», «угадывать», «предполагать» как можно больше возможных отношений. Эти упражнения впо- следствии явятся необходимым элементом поиска пути решения более сложных задач. Упражнения эти будут даны в полном соответствии с таблицей. Это упражнения, способствующие обучению актуализации нужных знаний при решении задач, упражнения — базис искус- ства видеть путь решения. Ответы на поставленные вопросы не однозначны. Полнота ответа зависит от объема накопленного к данному времени фак- тического материла. 1.1. Даны две различные точки плоскости (пространства). При каких геометрических преобразованиях они являются соответст- венными? То же задание для двух совпавших точек М — М'. 1.2. В каких отношениях может находиться точка А и прямая а в плоскости (пространстве). Может ли точка быть образом прямой? 1.3. В каком отношении может находиться точка А и плос- кость а в пространстве? 1.4. Перечислите все возможные случаи взаимного располо- жения точки А и луча MN\ точки А и полуплоскости с границей а; точки А и полупространства с границей а. 1.5. Чем может являться данная точка для данного отрезка? (для данного направленного отрезка?) 1.6. Перечислите все возможные случаи взаимного положе- ния точки и а) плоского угла; б) двуграного угла; в) многогран- ного угла. 1.7. Чем может быть точка для любого плоского много- угольника вместе с внутренней областью. 11
Какие «дополнительные обязанности» приобретает точка для частных видов конкретных многоугольников (трехугольников раз- ных видов, четырехугольников, правильных многоугольников)? Для каких многоугольников существует точка, являющаяся центром симметрии (самосовмещения)? Вставьте пропущенное слово, чтобы было верным следую- щее предложение: «Во внутренней области всякого ... много- угольника существует единственная точка, поворот вокруг кото- рой приводит к самосовмещению этого многоугольника». В каких отношениях может находиться точка с любым много- гранником? Какие дополнительные функции может «выполнять» точка для частных видов конкретных многогранников (пирамиды, призмы, тетраэдра, октаэдра, гексаэдра, додекаэдра, икосаэдра). Перечислите многогранники, имеющие центр симметрии. 1.8. Точка с кругом (шаром), окружностью (сферой). 1) Какие общие свойства («обязанности») точки по отноше- нию к окружности и кругу, сфере и шару, окружности и сфере, кругу и шару. 2) В следующих предложениях вставьте пропущенное слово: «Все точки .., удаленные от центра на расстояние г, при- надлежат окружности (сфере)». 1.9. В каких отношениях может быть точка с конусом? 1.10. а) Какие положения может занимать точка по отноше- нию к цилиндру? б) В каком случае точка является центром симметрии цилиндра? 1.11. а) Когда положение точки вполне определяется зада- нием одного числа а? б) Когда положение точки определяется парой действитель- ных чисел (а, Р)? в) Когда положение точки вполне определяется тройкой чи- сел (а, р, у)? 2.2. Две прямые в плоскости могут быть а)..., б)..., в)..., г).... Две прямые в пространстве могут быть: а) ..., б) ..., в) ..., г) ..., д) .... Даны две пересекающиеся прямые а и а. При каких преоб- разованиях плоскости эти прямые являются соответственными? При каких преобразованиях пространства они являются со- ответственными? (т. е. одна из них является образом другой). Даны две параллельные прямые а и Ь. При каких преобразова- ниях плоскости (пространства) они являются соответственными? Даны две совпавшие прямые а и Ь. При каких преобразованиях плоскости (пространства) одна из них является образом другой? 12
2.3. Прямая — плоскость. Перечислите известные вам слу- чаи взаимного расположения прямой и плоскости, выделив осо- бо один из важных частных случаев пресечения. В каких случаях прямая может служить направлением про- ектирования точек пространства на данную плоскость? 2.4. Прямая — луч (полуплоскость, полупространство). 1) Изобразите на рисунке все возможные случаи взаимного расположения прямой относительно луча (полуплоскости, полу- пространства). 2) Как расположить прямую, чтобы она являлась осью сим- метрии луча (полуплоскости, полупространства)? 3) Как расположить прямую, чтобы отражение от нее (осевая симметрия) отображало луч в дополнительный, полу- плоскость — в дополнительную полуплоскость? 2.5. Прямая — отрезок. 1) Изобразите на рисунке возможные взаимные положения прямой и отрезка. 2) Задайте прямую и различные по положению к этой пря- мой отрезки. Постройте отрезки, симметричные данным отно- сительно данной прямой. 3) При каком взаимном расположении данная прямая явля- ется осью симметрии отрезка? (Два случая). 2.6. Прямая — угол (плоский, двугранный, многогранный). 1) Проведите прямую через вершину плоского угла так, чтобы а) все точки угла принадлежали полуплоскости, определяе- мой этой прямой; б) все точки угла разбились на два класса' по принадлежно- сти к разным полуплоскостям, определяемым этой прямой; в) эта прямая была осью симметрии угла. 2) Проведите прямую так, чтобы она пересекала обе сторо- ны угла в различных точках. 3) Проведите прямую так, чтобы она пересекала только од- ну сторону угла. 4) Как называется прямая, если она принадлежит двум гра- ням двугранного угла? 5) Как провести прямую, чтобы она полностью принадле- жала одной грани двугранного угла. 6) Как провести прямую, пересекающую одну грань дву- гранного угла, но не пересекающую вторую. 7) Для какого двугранного угла всегда существует прямая, пересекающая обе грани в различных точках. 8) Можно ли провести прямую, полностью принадлежащую грани многогранного угла? 13
2.7. Прямая — многоугольник (многогранняк). 1) Проведите прямую а. Изобразите частныенгглы известных вам многоугольников, для которых эта прямая является осью симметрии. Если многоугольник имеет несколько осей симмет- рии, изобразите его в различных положениях так, чтобы симмет- рия относительно прямой а переводила его в самого'себя. 2) Рассмотрите отношение прямой, не принадлежащей плоскости многоугольника и некоторых частных видов много- угольников. При каком взаимном положении и с помощью ка- ких преобразований можно добиться самосовмещения? 3) Задайте прямую а. Перечислите и по возможности изо- бразите частные виды известных вам многогранников, для кото- рых эта прямая является пространственной осью симметрии. 2.8. Прямая — окружность (сфера), круг (шар). 1) Чем может быть прямая для окружности (сферы)? Изо- бразите возможные случаи. 2) Как провести прямую, чтобы она было осью симметрии окружности (сферы), круга (шара)? 3) Сколько осей симметрии имеет окружность, круг, сфера, шар? 2.9. Прямая — конус. 1) Перечислите и изобразите различные положения прямой относительно прямого кругового конуса. 2) В каком случае прямая является осью симметрии конуса? 2.10. Прямая — цилиндр. 1) Перечислите и изобразите не менее пяти принципиально различных положений прямой относительно прямого кругового цилиндра. 2) В каком случае прямая является осью симметрии цилиндра? 2.11. Прямая — числа. 1) Какое множество чисел «превращает» прямую линию в координатную прямую? 2) Сколько чисел определяют положение прямой на коор- динатной плоскости? 3.3. Плоскость — плоскость. 1) Перечислите возможные отношения между двумя плоско- стями. 2) Даны две пересекающиеся плоскости. При каких преоб- разованиях пространства одна из них является образом другой? 3) Даны две параллельные плоскости. При каких преобразо- ваниях пространства они являются соответственными? 14
4) Даны две совпадающие плоскости. При каких преобразо- ваниях пространства одна из них является образом другой? 3.4. Плоскость — луч. Перечислите возможные случаи взаимного положения луча с плоскостью. 3.5. Плоскость — отрезок. Задайте плоскость П (с помощью изображения). Расположи- те отрезок АВ так, чтобы: 1) Все точки отрезка совпали со своими образами при сим- метрии пространства относительно плоскости П\ 2) Концы отрезка принадлежали разным полупространствам с границей П, а образом отрезка АВ при симметрии пространст- ва относительно П был тот же отрезок; 3) Концы отрезка принадлежали одному полупространст- ву. Найдите образ отрезка АВ при симметрии относительно плоскости П. 4) В прямоугольной системе координат плоскость задана уравнением Зх + 2у — 6z - 0 и точки, заданные координатами: 0(0, 0, 0), А(3, 5, 0), В(4, 5, 1), С(1, 2, 3), 0(2, 3, 1). По этим данным можно сделать вывод о взаимном расположении точек относительно плоскости. Попробуйте «открыть» эту закономер- ность и ответить на некоторые вопросы (пересекают ли плос- кость П отрезки ОА, ОВ, ОС, АВ, АС, ВС ? Какая из точек при- надлежит плоскости?). 3.6. Плоскость — угол (плоский, двуграный, многогранный). 1) Что будет образом плоского угла при симметрии про- странства относительно плоскости, перпендикулярной плоско- сти угла и проходящей через его биссектрису? 2) Каким свойством обладает биссекгориальная плоскость двугранного угла? 3) Рассмотрите несколько многогранных углов для которых существует плоскость симметрии. Как она проходит? Вы поняли суть составления упражнений в соответствии с таблицей. Постарайтесь продолжить сами и составление и ре- шение для случаев: 3.7, 3.8, 3.9, 3.10, 3.11. Не забудьте о такой «ответственной работе» точки, прямой и плоскости как разбие- ние и о таком важном отношении 3.7' аналогичном 2.7, которое лежит в основе понятия выпуклого многогранника. Мы рассмотрели отношения основных (неопределяемых) фигур со всеми остальными. Теперь нам предстоит рассмотреть более богатые отношения между определяемыми фигурами с 15
многочисленными вариациями аналогов. Так, отношение 4.4 охватывает вариации отношений 4.4', 4.4',' 4'.4', 4'.4',' 4'.'4'.' Как возросло число различных ситуаций, а значит и упражнений. 4.4. Луч — луч. 1) Два луча могут быть .... и ...., а могут не быть ни теми, ни другими. 2) Два луча с общей вершиной О являются симметричными относительно .... и соответственными при повороте с центром .... на угол .... 3) Два сонаправленных луча Оа и О а' являются соответст- венными при параллельном переносе на вектор ...., при любой гомотетии с центром .... 4) два противонаправленных луча Оа и О а' являются соот- ветственными при симметрии относительно .... и гомотетии с центром .... 4.4'. Луч — полуплоскость. 5) Проведите несколько лучей из вершины, принадлежащей полуплоскости (в том числе и ее границе). Для каждого ли луча с этой вершиной можно в полуплоскости провести противопо- ложный луч? Сколько пар противоположных лучей с общей вершиной можно провести из каждой точки полуплоскости? 4.4'.' Луч — полупространство. 6) Аналогично предыдущему упражнению. 4'.4'. Полуплоскость — полуплоскость. 7) Какие фигуры можно получить в пересечении полуплос- костей с параллельными и пересекающимися границами? 4'.'4'.' Полупространство — полупространство. 8) Какие фигуры можно получить в пересечении полупро- странств с пересекающимися и параллельными границами? 4.5. Луч — отрезок. 1) Даны луч Оа и отрезок АВ. Расположите отрезок АВ по отношению к лучу Оа так, чтобы получились следующие сово- купности фигур: а) отрезок и луч; б) отрезок и два луча; в) три отрезка и три луча. 2) В плоскости заданы луч Оа и отрезок АВ. Не меняя по- ложения луча, расположите отрезок АВ так, чтобы он не имел общих точек с лучом и занимал следующие положения: а) был параллелен лучу; б) составлял острый угол с лучом; 16
в) был перпендикулярен к лучу. 3) В плоскости задан луч Оа и отрезок АВ. Не меняя поло- жения луча, расположите отрезок АВ так, чтобы он имел общие точки с лучом и занимал следующие положения: а) принадлежал дополнительному лучу; б) составлял острый угол с лучом; в) был перпендикулярен лучу. Сравните наличие общих то- чек в различных случаях. 4'.5. Полуплоскость — отрезок. 4) Полуплоскость задана прямой а и точкой М. Изобразите отрезок МК так, чтобы все точки отрезка МК принадлежали этой полуплоскости и отрезок МК был: а) параллелен границе; б) перпендикулярен ей; в) составлял острый угол с прямой а. 5) Полуплоскость задана прямой а и точкой М. Изобразите от- резок Л/А'так, чтобы его концы принадлежали разным полуплоско- стям с границей а. Может ли в этом случае отрезок МК быть па- раллелен, перпендикулярен, составлять острый угол с границей д? 6) Пусть прямая b определяет две полуплоскости ЬМ и ЬК. Сформулируйте условие принадлежности точек А и. В одной по- луплоскости и разным полуплоскостям. 4'.'5. Полупространство — отрезок. 7) Переформулируйте три последних упражнения для полу- пространства, определяемого плоскостью и точкой, и отрезка. Сравните ответы на поставленные вопросы. Заполните пропу- щенные слова в следующих предложениях: а) «Если одним из концов отрезка является данная точка, то существует .... отрезка, параллельных границе полуплоскости, а отрезков, параллельных границе полупространства — ...... б) «Если одним из концов отрезка является данная точка, то отрезков, составляющих данный острый угол с границей полу- плоскости, существует .... , а отрезков, составляющих данный острый угол с границей полупространства, — ...... в) «Если одним из концов отрезка является данная точка, то существует .... отрезок, перпендикулярный границе и полуплос- кости, и полупространства». г) «Если точки А и В принадлежат одной полуплоскости [одному полупространству], то отрезок АВ.... ». д) «Если точки А и В принадлежат разным полуплоскостям [полупространствам] с общей границей, то отрезок АВ .... ». 17
4.6. Луч — плоский угол. 1) Перечислите не менее трех отношений луча и плоского угла. 2) Дан Z(aZ>) и М — внутренняя точка этого угла. Проведите несколько лучей с вершиной в точке М так, чтобы они содержа- ли только внутренние точки данного угла. 3) Верны ли предположения: а) «Любой луч с вершиной во внутренней точке угла пере- секает хотя бы одну из его сторон». б) «Существует луч с вершиной во внешней точке угла, ко- торый пересекает только одну сторону угла». в) «Существует бесконечно много лучей с вершиной во внутренней точке угла, которые не пересекают ни одной сторо- ны угла». г) «Любой луч, содержащий все точки, одинаково удаленные от сторон угла, является биссектрисой этого угла». 4.6' . Луч — двугранный угол. 4) Замените в предыдущих упражнениях (4.6) слова «стороны плоского угла» словами «грани двугранного угла» и исследуйте полученные отношения. 5) Рассмотрите дополнительные «роли» луча для двугран- ного угла. 4.6'. Луч — многогранный угол. 6) Какие отношения, кроме перечисленных, добавляются между лучом и многогранным углом? 4'.6. Полуплоскость — плоский угол. 7) Прямая а и точка вне ее М определяют полуплоскость аМ. Задайте угол {Ьс) так, чтобы возникли следующие ситуации: а) можно назвать только один угол; б) можно назвать только два угла, считая развернутый и ну- левой; в) можно назвать только три угла. 4'.6'. Полуплоскость — двугранный угол. 8) Если граница полуплоскости совпадает с ребром двугран- ного угла, то эта полуплоскость может быть: а)...., б)...., в). 4'.6'.' Полуплоскость — многогранный угол. 4'.'6. Полупространство — плоский угол. 4'.'6'. Полупространство — двугранный угол. 4'.'6'.' Полупространство — многогранный угол. 18
9) — 12). Укажите известные вам отношения между указан- ными парами объектов. Установите известные вам отношения между следующими парами объектов. 4.7. Луч — многоугольник. 4.7'. Луч — многогранник. 4'.7. Полуплоскость — многоугольник. 4'.7'. Полуплоскость — многогранник. 4'.'7. Полупространство — многоугольник. 4'.'7'. Полупространство — многогранник. 4.8. Рассмотрите различные отношения между лучом и ок- ружностью (сферой), между полуплоскостью и окружностью (кругом), между полупространством и сферой (шаром). Дополните предложения: «Множество точек полуплоскости, удаленных от точки, принадлежащей границе полуплоскости, на данное расстояние есть ...... «Множество точек полуплоскости, удаленных от данной точки, принадлежащей границе, на рас- стояние не большее данного, есть ...... «Множество точек полу- пространства, удаленных от данной точки, принадлежащей его границе, на данное расстояние, есть ...... 4.9. Исследуйте различные взаимные расположения луча (полуплоскости), ((полупространства)) по отношению к конусу. 4.10. То же по отношению к цилиндру. 5.5. Отрезок — отрезок. 1) Расположите два равных отрезка так, чтобы они были со- ответственными а) при осевой симметрии; б) при повороте; в) при цен- тральной симметрии; г) при параллельном переносе. 2) Расположите два неравных отрезка так, чтобы они были соответственными при гомотетии. 3) Перечислите другие отношения, в которых могут нахо- диться два отрезка. 5.6. Отрезок — угол (плоский, двугранный, многогранный). 1) Изобразите угол (ab) и отрезок АВ так, чтобы через лю- бую внутреннюю точку отрезка проходил внутренний луч дан- ного угла. 2) Дан двугранный угол с ребром а. Известно, что любая внутренняя точка отрезка АВ и прямая а определяют полуплос- кость, состоящую только из внутренних точек двугранного угла. Как расположен отрезок АВР. 19
3) Дан многогранный угол с вершиной Л/. Как расположить отрезок АВ, чтобы плоский угол с вершиной в точке М содержал все лучи, проходящие через точки этого отрезка, разбивал мно- гогранный угол на два многогранных угла? Каково наименьшее число граней у данного угла? 5.7. Отрезок — многоугольник (многогранник). 1) Перечислите не менее 10-ти отношений («обязанностей») отрезка с многоугольниками и многогранниками, выделяя их частные виды. 5.8. Отрезок — окружность (сфера), круг (шар). 1) Аналогичное задание для отрезка и указанных фигур. 5.9., 5.10. Отрезок — конус и цилиндр. 1) Назовите те роли отрезка, которые именуются одними и теми же терминами как для конуса так и для цилиндра. 5.11. Отрезок — число. 1) Продолжите предложение, назвав не менее 20-ти «собственных имен» отрезка: «При решении задач часто при- ходится находить длину гипотенузы прямоугольного треуголь- ника, ....,...... .... , 2) А как используются числа для характеристики направ- ленного отрезка? 6.6. Угол (плоский, двугранный, многогранный) — угол. 1) Дан плоский угол (ab). Постройте а) угол, равный данному. Сколько таких углов в плоскости? б) угол, меньший (больший) данного. Сколько их? в) угол, смежный с данным. Сколько их? г) угол, вертикальный данному. Сколько их? 2) Аналогичные упражнения для двугранного угла. 3) Аналогичные упражнения для многогранного угла. 4) Дан двуграный угол (АВ). Постройте линейный угол дву- гранного угла. 5) Дан многогранный угол (п = 3, 4, 5, 6). а) сколько в каждом случае задано двугранных углов? б) сколько плоских углов в каждом случае? 6.7. Угол (плоский, двугранный, многогранный) — много- угольник (многогранник). 1) Изобразите плоский многоугольник (п = 3, 4, 5, ..., п) а) отметьте внутренние углы многоугольника; б) отметьте внешние углы многоугольника; в) отметьте углы, прилегающие к одной стороне (не приле- гающие); 20
г) у каких многоугольников можете назвать пары противо- положных углов? 2) Изобразите несколько правильных многоугольников. От- ветьте на вопросы: а) какие правильные многоугольники имеют острые внут- ренние углы; б) какие правильные многоугольники имеют прямые внут- ренние углы; в) сколько и какие правильные многоугольники имеют ту- пые внутренние углы? 3) Те же вопросы относительно внешних углов правильных многоугольников. 4) Приведите примеры многогранников. Изобразите неко- торые из них. Назовите углы граней этих многогранников. На- зовите двугранные и многогранные углы в некоторых из них. 5) Сколько правильных многогранников вы знаете? Какие? Какими особенностями отличаются их плоские, двугранные и многогранные углы? 6.8. Угол — окружность (круг). 1) Задайте окружность (О; ОМ) и угол АВС. Постройте угол, равный данному так, чтобы он был: а) вписанным в данную окружность; б) описанным около данной окружности; в) с вершиной внутри круга; г) центральным углом для этой окружности. 6.9. Угол — конус. 1) На изображении конуса укажите следующие углы: а) угол образующей с основанием; б) угол осевого сечения конуса; в) угол произвольного сечения конуса плоскостью, прохо- дящей через вершину конуса. 6.10. Угол — цилиндр. 1) На изображении наклонного кругового цилиндра укажите угол произвольной образующей цилиндра с его основанием. 2) На изображении прямого кругового цилиндра укажите двугранный угол, образованный сечением, не проходящим через ось цилиндра с плоскостью основания этого цилиндра. 6.11. Угол — число. Аналогично отношению (5.11), число характеризует величи- ну (меру) угла в градусах или радианах. Поэтому чтобы хорошо решать задачи связанные с нахождением углов, надо знать те 21
функции, которые выполняют углы в различных фигурах и за- висимости между различными углами в этих фигурах. Как правило, вместе с присвоением имени углу указывается и способ его измерения или вычисления. Так, например, «Внешний угол треугольника равен сумме внутренних, не смежных с ним». Можно продолжить в том же плане выявление отношений между объектами (7.7), (7.8), (7.9), (7.10), (7.11); затем, (8.8), (8.9), (8.10), (8.11); (9.9), (9.10), (9.11); и, наконец, (10.10), (10.11). Но уже напрашиваются аналогии не только между объектами, но и самими отношениями. Так, отношения: (7.11), (8.11), (9.11), (10.11) связаны с вычислениями длин, величин углов, площадей и объемов фигур, часто являющихся элементами других фигур. Особый интерес представляют пары отношений между од- ноименными фигурами: (5.5), (6.6), (7.7), (8.8), (9.9), (10.10). Для всех пар указанных фигур имеет место отношение ра- венства, обладающее следующими тремя свойствами: а) каждая фигураФ равна сама себе (Ф = Ф) (рефлексивность); б) если Ф1 = и Фг = Ф\ (симметричность); в) если Ф1 = 02, а Фг = Фз, то Ф] = Фз (транзитивность). 1) Назовите другие отношения, обладающие этими тремя свойствами для пар фигур (2.2), (3.3), (4.4), (5.5), (6.6), (7.7), (8.8), (9.9), (10.10). 2) Назовите отношения между этими же фигурами, которые не являются: а) рефлексивными; б) симметричными; в) транзитивными. Таким образом, рассмотренные упражнения затрагивают бо- лее 100 отношений между фигурами и их элементами, а также между фигурами и числами. Умение привести в готовность нужные из них и служит залогом умения решать задачи. Между тем в большинстве «Сборников задач», предназначенных для тренировки в решении задач фигурируют явно или неявно толь- ко 14: (7.7), (7.8), (7.9), (7.10), (7.11); (8.8), (8.9), (8.10), (8.11); (9.9), (9.10), (9.11); (10.10), (10.11). Мы рассмотрели примеры установления отношений между двумя объектами, выделенными явно в таблице, но неявно при- влекали третий, чаще всего, прямую или плоскость. Если же рассматривать тройки объектов, то отношения будут богаче, сложнее. Выявлению их посвящены последующие задачи, зада- чи-спутники, подзадачи. 22
II часть ПОДУМАЕМ ВМЕСТЕ (искусство поиска решения задач) А. Общие рекомендации (Несколько советов на тот случай, когда задача не поддается решению). Исследуйте задачу наиболее естественным путем, допуская, что она решена, и постарайтесь в соответствующем порядке наглядно представить все соотношения, которые, согласно усло- вию, должны иметь место между неизвестными и данными. От- ветьте себе, что дано и что требуется найти. Помните, что целью задачи на нахождение является неиз- вестное. Чтобы сосредоточить свое внимание на этой цели, спросите себя: что представляет собой неизвестное? Целью задачи на доказательство является заключение. Спросите себя: в чем состоит заключение? Испытайте задачу на правдоподобие. Переформулируйте задачу. Подумайте, не встречали ли похожую. Используйте аналогию. После решения вернитесь: «Нельзя ли решить проще?» Б. Примеры поиска. Искусство создавать задачи-спутники ведет к искусству ре- шать любые самые сложные задачи, больше того, не только ре- шать, но и ставить самим еще более сложные задачи. Как же рождаются эти спутники? Проследим ход мысли на примере любой задачи. Задача 1. Дан угол со сторонами Оа и ОЬ и внутри него две точки М и Р. Построить трехзвенную ломаную МАВР мини- мального периметра так, чтобы концами ее были данные точки М и Р, а две другие точки А и В принадлежали соответственно сторонами Оа и ОЬ данного угла. Прочитайте еще и еще раз условие задачи, сделайте эскиз рисунка, изобразите на нем несколько правдоподобных лома- ных, сравните хотя бы на глаз их длины., И если никакой идеи не возникло, попробуйте составить вспомогательную задачу, 23
решение которой было бы легче, прозрачнее, обозримее. По- пробуйте уменьшить число звеньев ломаной до двух, а прямых до единственной. Задача примет вид: Построить ломаную МАР, чтобы МА + АР имела наимень- шую длину и вершина А лежала на данной прямой а. Задайте прямую а и точки М и Р вне ее. Но как задать точ- ки? Их можно задать в разных полуплоскостях относительно а, а можно и в одной. Сделайте для обоих случаев рисунки (рис. 1а) и (рис. 16). р В первом случае решение > мгновенно, ибо периметр это , .......... а сумма длин отрезков МА + АР, ./г А а она будет наименьшей тогда, М когда точки М, А, Р лежат на р одной прямой. Ломаная вы- yZ прямлена. Попробуйте свести к такому случаю и второй вариант _______[yZ б расположения точек (рис. 16). |Z А Свойства осевой симметрии М' помогли «выпрямить» ломаную Рис. 1а,б МАР путем замены звена МА на равное ему звено М 'А. Теперь вернитесь к исходной задаче и используйте добытую идею «выпрямления» к трехзвенной ломаной. Мы уверены: успех и удовлетворение, полученные при ре- шении последней побудят вас составить свою, но более слож- ную задачу. В каком направлении можно усложнять? Можно составить пространственный аналог решенной задачи, а можно увеличить на единицу и число звеньев ломанной и число данных прямых, на которых будут находиться вершины А, В и С ломанной МАВСР. Итак, задача: Внутри треугольника АВС даны точки М и Р. На сторонах ВС, АС и АВ найти такие точки А\, В\, и Q чтобы периметр ло- маной МА\В\ С\Р имел наименьшую длину. Многократно используя осевую симметрию Вам удастся найти решение. Задача 2. Построить треугольник, сторона которого прохо- дила бы через данную точку М, а биссектрисы его лежали бы на трех данных пересекающихся в одной точке прямых. 24
Не всегда создаются задачи-помощники только за счет уменьшения заданных элементов. Вот и в этой задаче: заданы только прямые, на которых должны лежать биссектрисы и толь- ко на одной стороне единственная точка. Слишком большая свобода, неопределенность в проведении стороны. Даже прав- доподобный рисунок трудно подобрать. Попробуем уменьшить на одну число данных прямых, но увеличим на одну число данных точек на сторонах, причем, вы- скажемся более определенно: Пусть две пересекающиеся прямые содержат биссектрисы двух углов при основании треугольника АВС с вершиной в точке С, а две данные точки лежат соответственно на двух бо- ковых сторонах. Построить этот треугольник. Пусть а и b заданные бис- секториальные прямые, кото- рым принадлежат искомые вершины Аи В, аМиР — точки на сторонах АС и ВС (рис. 2). Теперь любое свойст- во биссектрисы позволит най- ти две точки на прямой, кото- рой принадлежит искомая сторона треугольника АВ. (Одинаковая удаленность от сторон угла или симметрич- ность). Найдя точки М' и Р\ симметричные точкам М и Р относительно прямых а и b и проведя прямую М'Р ’, найдем вершины треугольника А и В как точки пересечения этой прямой с биссекториальными прямыми а и Ь. Пересечение же прямых AM и ВР опреде- ляют вершину С. После решения этой задачи решением основной задачи. Вернитесь к ней (рис. 3). В ряде задач, особенно на построение частных видов фигур, некоторые точки которых должны принадлежать другим фигу- рам, догадка связана с использованием преобразования плоско- сти (поворота, параллельного переноса, гомотетии или цен- в Рис. 3 A Mt м вы, безусловно, справитесь с 25
трального подобия, осевой или центральной симметрии). Выбор и использование преобразования зависит от умения увидеть в условии соответственные элементы того или иного преобразо- вания. Так, если в условии задачи речь идет о равностороннем треугольнике с заданной вершиной, то поможет поворот на 60° вокруг этой вершины; если нужно построить квадрат или рав- нобедренный прямоугольный треугольник с вершиной в данной точке, то смело используйте поворот на 90° с центром в этой точке; вообще, если в фигуре имеются два равных отрезка с об- щей вершиной и известным углом между ними, то они опреде- ляют (задают) поворот в двух направлениях на данный по вели- чине угол, а так же осевую симметрию относительно биссектри- сы этого угла. Аналогично, два равных параллельных отрезка являются соответственными при параллельном переносе, цен- тральной симметрии, а при особом расположении — и осевой симметрии. Два же неравных параллельных отрезка определяют два вида гомотетии. Используйте аппарат этих преобразований плоскости при решении контрольных задач №№ 3, 4, 5, 6. Если этого указания недостаточно, обратитесь к готовым за- дачам-спутникам с соответствующими номерами. Задача 7. Дана прямая /, окружность (О, г) и точка А. Построить равносторонний треугольник с вершиной в точке А так, чтобы другие его вершины В и С лежали соответственно на прямой I и окружности (О, г) (рис. 4). Рис. 4 то для его построения или С. Допустим, что Анализ — поиск реше- ния. Так как искомый тре- угольник — равносторонний и одна из его вершин А задана, достаточно найти еще одну вершину В треугольник АВС уже построен. Тогда точку В можно перевести (отобразить) в точку С поворотом R^°, который переводит прямую / вместе с принадлежащей ей точкой В в прямую /' проходящую через точку С. Отсюда следует способ построения точки С и треугольника АВС. 26
Построение. 1) R^° (1) = Г; 2) Г n (О, г) = {С, Q}; 3) На отрезках АС и ACi, как на сторонах, строим равносторон- ние треугольники АСВ и АС[В[ по ту сторону, где расположена прямая 1. Эти треугольники являются искомыми. Доказательство. Из построения следует, что треуголь- ник АВС равносторонний, одна его вершина совпадает с данной точкой А, а другая принадлежит окружности (О, г). Докажем, что В принадлежит прямой /. По построению (1) = Г; со- гласно свойству поворота, имеем R^°° (Г) = /, и, следова- тельно, Кд°а точку С, принадлежащую /' переводит в точку В, принадлежащую прямой /. Исследование. Число решений равно числу общих то- чек прямой I и окружности (О, г). Если поворот задан центром и направленным углом, то число решений может быть 2, 1, 0. Если же только величиной — соответственно 4, 3, 2, 1, 0. Задача 8. В трапеции ABCD угол между боковыми сторо- нами прямой. Провести окружность через точки А и В, касаю- щуюся прямой CD. Пусть L — точка каса- ния (рис. 5), К — середина Е отрезка АВ, Е — точка пе- / ресечения боковых сторон р~ / s q АВ и CD. Центр искомой окружности О лежит на пересечении двух перпен- //X / vv дикуляров: hi — середин- VK ного перпендикуляра к от- д ’Ч " '/ ' ' р резку АВ, йг — перпенди- куляра к касательной CD в точке касания L. Тогда ра- Рис. 5 диус искомой окружности г = OL = КЕ. Следовательно, центр О можно построить как точку пересечения окружности (А, г = КЕ) и серединного пер- пендикуляра hi к отрезку АВ. Задача 9. На сторонах данного треугольника постройте вне его три равносторонних треугольника и соедините их центры. Докажите, что полученный таким образом треугольник равно- сторонний. 27
Не возникает ли у Вас сразу несколько путей доказательст- ва? Акцент на равенство всех сторон способствует выдвижению идеи использования метода координат. В самом деле, стоит вве- сти систему координат, выразить координаты центров постро- енных треугольников (обозначим их. А', В', С') через величины данных и применить формулу расстояний между двумя точками и убедиться, что А'В' = В'С' = С’А’. Идея проста, надежна, но хватит ли терпения выполнить громоздкие алгебраические пре- образования, в которых и ошибку легко допустить, а значит прийти к неправильному выводу. Поищем другой путь. Равносторонний треугольник является и равноугольным, да еще и с хорошими углами в 60°. Легко вы- полняется поворот на 60°. И если окажется, что после поворота плоскости Rg°, образ центра одного треугольника совпадает с центром другого, то задача решена (/?|°°: АНаметили путь доказательства этого совпадения. Все равносторонние треуголь- ники подобны. Значит, и треугольники, построенные на сторо- нах АВ = с и ВС = а, причем коэффициент подобия к = ale. Поворот — движение. Размеры сохраняются. Значит, образ од- ного из треугольников подобен другому с тем же коэффициен- том. Еще один поворот на угол, связанный с углом данного тре- угольника, приводит к гомотетии полученных треугольников с тем же коэффициентом а/с и совпадению центров. Для доказательства равенства А'В’ = В’С' = С'А' можно использовать и тригонометрию, считая данными стороны и углы треугольника АВС, а неизвестными отрезки А'В' = х, В'С' = у, С'А' = z- Задача 10. Иногда при не поддающейся решению задаче в качестве задачи-помощника следует рассмотреть либо частный вид фигуры, либо поместить данный элемент в особое, понра- вившееся место, либо вообще сформулировать всю задачу для частного расположения данных. Так, при решении задачи №10 Дан треугольник АВС и внутри него точка Р. Провести через эту точку прямую а так, чтобы она рассекала треугольник на две равновеликие фигуры. никак не удается выбрать еще одну точку, чтобы можно было 28
провести прямую, рассекающую треугольник на равновеликие части, а также остановить свое внимание на отборе параметров, входящих в многочисленные формулы вычисления площадей многоугольников. Попробуем изменить положение данной точки, через кото- рую надо проводить искомую прямую. Вообще говоря не так уж много вариантов возможного положения: а) внутри треугольни- ка (как в нашем случае), б) вне треугольника («хрен редьки не слаще»), в) на стороне его и г) в вершине. Самое легкое, предельно очевидное, требующее минимума знаний, будет решение в последнем случае: при одной и той же высоте площади треугольника равны при равных основаниях. Рассмотрим случай в) (См. рис. 6). Наличие точки М на стороне треугольника увеличивает число отрезков и сводит ре- шение задачи к нахождению одного отрезка х, отложив ко- к торый от вершины, найдем / вторую точку прямой, рассе- / кающей данный треугольник / х. на две равновеликие фигуры. / х. Выражение длины отрезка х I через известные отрезки мо- Мт х. жет основываться на различ- / ных формулах площади тре- д! \ угольника, включая формулы, использующие тригономет- Рис-6 рию. Рассмотрим решение, основанное на применении одной из первых формул площади треугольника: S = l/2ha. Введем обо- значения: AM = к, МС = р, АВ = с, СВ = d и СН = х, где Н — искомая точка. Отрезки перпендикуляров, опущенных из точки М на стороны АВ и СВ обозначим соответственно hi и h^- По условию задачи площадь треугольника МСН равна пло- щади четырехугольника АМНВ или сумме площадей двух тре- угольников: AM В и ВМН. hix = h\c + «2(fl _ *) откуда x = 1 z . А если знаешь /«2 формулу 5-=l/2 absinC, то решение мгновенно: 2рх = ab. Не- трудно видеть, что если точка М будет серединой стороны АС, то точка Н совпадет с вершиной В, если МС < МА, то Н будет лежать на АВ, если же МС > МА, то Н будет лежать на ВС. 29
Рис. 7 Теперь можно приступить и к решению основной задачи, где М внутри треугольника АВС (рис. 7). Пусть искомая прямая пересекает стороны треугольника АС и ВС соответ- ственно в точках К и Н. Зада- ние точки М внутри треуголь- ника позволяет использовать при решении еще больше до- полнительных отрезков, но ис- комая прямая отсекает треугольник, в котором известен только угол С и отрезок СМ, разбивающий этот треугольник на два треугольника с общей вершиной М, из которой мож- но провести высоты hi, h2- Эти отрезки будут высотами и для треугольников ВМН и АМК, ah — высотой треугольни- ка АСВ. Обозначим КС = х, СН = у. Выражая зависимость между площадями отсеченного и данного треугольника дву- мя различными способами получим систему двух уравнений с двумя переменными, решение которой приведет к нахож- дению двух различных прямых, обладающих указанным в условии свойством. + 2/)2у = he 2ху = ab Иногда во время поиска решения задачи назойливо лезут в голову спутники-предатели. И от них трудно избавиться. Тогда надо переформулировать задачу. Рассмотрим этот процесс на примере задачи №11. Задача 11. Доказать, что сумма длин диагоналей выпуклого четырехугольника меньше периметра, но больше полупериметра этого четырехугольника. Приступим к решению. Обозначим данный четырехуголь- ник ABCD и запишем заключение первой части доказываемого утверждения: АС + BD < АВ + ВС + CD + AD. Конечно же, всплывает в памяти теорема о том, что в любом треугольнике каждая сторона меньше суммы двух других, т. е. АС < АВ + ВС, АС < AD + DC, BD < ВС + CD, BD < АВ + CD. Сложив почлен- но неравенства одинакового смысла, получим доказательство первой части утверждения. 30
Окрыленные успехом, бодро принимаемся за вторую часть, AB+BC+CD+AD „ записав ее так: АС + BD > -------------. Предательски на- 2 прашивается теорема о том. что в треугольнике каждая сторона больше разности двух других. Но из этих разностей никак не удается получить полупериметр. Рассмотрение неравенств в тре- угольниках, сторонами которых являются диагонали не приво- дит к успеху. Надо искать другие треугольники, в которых двумя сторонами являлись бы диагонали или их части, а третьими сто- ронами — части периметра. Хорошо бы переформулировать за- дачу так, чтобы можно было использовать суммы отрезков, а не разности, то есть привлечь для решения теорему о сравнитель- ной длине сторон со знаком «меньше». Переформулируем условие задачи так: Доказать, что полупериметр меньше суммы диагоналей. А значит, периметр меньше удвоенной суммы диагоналей. Обозначив точку пересечения диагоналей через О, мы по- лучаем желанные треугольники, из свойств которых следует: АВ < АО + OB, ВС< ВО+ ОС, CD < ОС + OD, AD < АО + OD. Складывая почленно неравенства, получим: АВ + ВС + CD + DA < 2(АО + ОС + ВО + OD), ч. т. д. Задача 12. В треугольнике ABC AD — медиана, точка О — середина медианы. Доказать, что прямая ВО пересекает сто- рону АС в точке К, такой, что АК: КС =1:2. Итак, надо доказать, что точка К делит АС в отношении 1 : 2, то есть ЛХ: КС = 1 : 2. Перебираем в памяти известные фигуры, в которых имеют- ся пропорциональные отрезки: а) в подобных треугольниках стороны пропорциональны, но при имеющихся данных наличие подобных фигур сомнительно; б) пропорциональные отрезки в круге — совсем не похоже; в) биссектриса угла делит противоположную сторону на части, пропорциональные прилежащим отрезкам. Эта идея привлекает. Даже на чертеже похоже, что ВК — биссектриса угла В и отно- шение сторон вроде бы такое же. Сделали несколько различных по форме треугольников в соответствии с условием, гипотеза с биссектрисой назойливо лезет в голову, но увы нет никаких данных ни о биссектрисе, ни о сравнительной длине сторон. Созрела мысль о новой задаче с биссектрисой, но о ней после. Сейчас надо заставить себя отвлечься от свойств биссектрисы и 31
извлечь из памяти другие случаи пропорциональности отрезков, причем не любой пропорциональности, а такой, где один из отрезков вдвое больше другого. Сделаем рисунок (рис. 8а). Внимательно изучаем данные и вдруг вспоминаем даже сразу две теоремы, в которых один отре- зок вдвое больше другого: средняя линия треугольника вдвое меньше основания и медианы точкой пересечения делятся в от- ношении 1 : 2. Уже есть выбор. Начнем с любого. Выполним соответствующие дополнительные построения (рис. 86). Точка D — середина стороны ВС. Какую среднюю линию провести? Параллельно АВ или АС? Прикинули, решили провести парал- лельно АС, то есть DE 11 AC. F — точка пересечениях/^ и ВК. В треугольнике КВС FD — средняя линия, значит FD •) КС =1:2, надо АК: КС =1:2. Осталось показать, что FD можно заменить на АК. Так это же просто. &.OFD = \АОК, так как О — середина AD и нужные углы равны по свойству параллельных и секущей. Решение найдено. Попробуем другое решение: сделаем АС медианой какого-то треугольник^, а К — точкой пересечения медиан его (рис. 8в). СА — медиана треугольника СВ... пусть СВО, где О лежит на прямой ВА, причем АО = ВА, отложили АО = ВА, провели ОС (рис. 8в), обозначили через М точку пересечения ОС с ВК и ви- дим, что ВМ — медиана этого треугольника ОВС, так как AD — его средняя линия, которая точкой Q разделена пополам. Реализуя намеченные планы, мы вдруг увидели еще одно решение, основанное на теореме Фалеса: проведем DE 11 ВК (рис. 8а). Так как АО = ODn BD = DC, то АК = КЕ= ЕС. Зна- чит, АК: КС = 1 : 2. Рис. 8а Рис. 86 Рис. 8в
Вернемся к первой гипотезе о биссектрисе. Подтвердим или опровергнем ее. Пусть в треугольнике АВС также проведена ме- диана AD. Прямая BQ, проведенная через середину этой медиа- ны пересекает АС в точке К, которая делит сторону АС в отно- шении 1 : 2. Достаточно ли данных, чтобы доказать или опро- вергнуть, что ВК — биссектриса угла в ДАВС. Если нет, что можно добавить, чтобы получилась наша но- вая задача. Вернемся к чертежу (рис. 8а) без дополнительных построений. Если отрезок ВК будет биссектрисой угла в Д4ВС, то ВО должен быть биссектрисой того же угла в &ABD, но по условию, ВО — медиана. А Медиана является и биссектрисой и высотой только в равнобедренном треугольнике. Но данные задачи не обеспечивают этой «равнобедренности». И наоборот, имеются лишние, зависимые данные: О — середина AD и К делит АС в отношении 1:2. Итак, сформулируем варианты условия задачи, в которой нужно будет доказать, что ВК — биссектриса. А. В треугольнике АВС ВС =2 АВ. Через точку О — середи- ну медианы AD проведена прямая ВО, пересекающая сторону АС в точке К Доказать, что ВК — биссектриса угла В. Б. В треугольнике АВС через вершину В проведена прямая, перпендикулярно к медиане AD, которая пересекает сторону АС в точке К. Известно, что АК: КС =1:2. Доказать, что ВК — биссектриса угла В. Задача 13. В треугольнике АВС, где АВ > ВС проведена ме- диана ВО. Доказать, что ХАВО < ХОВ С (медиана с большей стороной образует меньший угол). Для доказательства надо актуализировать знания о сравне- нии отрезков и углов, но не в отдельности («каждая сторона с треугольнике меньше суммы двух других сторон» или «внешний угол треугольника больше внутреннего, с ним не смежного»), а в комбинации углов и отрезков. На поверхность всплывает предложение «В треугольнике против большей стороны лежит больший угол». В нашей задаче известна большая сторона и до- казать надо, что один из углов больше другого. Только углы эти не лежат против сравниваемых сторон. Значит надо выполнить такое дополнительное построение, заменив, если надо, данные отрезки равными, но выполнить так, чтобы получить треуголь- ник, в котором сравниваемые искомые углы оказались бы рас- 2—1224 33
в Рис. 9 положенными против сторон, о которых известно, какая из них больше какой. Это построение можно выполнить так (рис. 9). Продолжив медиану на ее длину (OD = О В) и соединив AD и DC отрезками, получим параллелограмм (по признаку диагона- лей), в котором AD = ВС, ZADB = Р, и рассматривая &BAD за- ключаем, что а < р, так как AD < АВ. Иногда при решении зада- чи, рассчитанной на опреде- ленный запас знаний, возника- ет мысль об излишних ограни- чениях. Не упускайте возмож- ности испробовать свои силы для создания и решения более общей задачи. Удовлетворение и польза от полученного ре- зультата компенсирует затра- ченный труд. Рассмотрим процесс работы над такой задачей. Задача 14. В равнобедренную трапецию вписан круг. Доказать, что отношение площади круга к площади трапе- ции равно отношению длины окружности к периметру тра- пеции (рис. 10); Вводя обозначения для отрезков от вершин до точек каса- ния, ты не можешь не подумать: «А почему только у равнобед- ренной трапеции?» В самом деле, пусть в трапецию ABCD вписан круг. Вспом- ните условия, при которых это возможно. Пусть точки касания сторон трапеции 7\, 7\, 7} и Т4, радиус круга — г. Обозначим отрезки сторон трапеции T4D = DT3 = а; Т$С = СТг = Ь; Т^В = ВТ\ = с, Т}А = АТ4 = d и выразим нужные нам величины. Рис. 10 a+b+c+d 2Г _ кг 2 a+b+c+d 2nr: 2(а + b + с + d) = nr a+b+c+d 34
Свойство равнобедренности трапеции не использовано. А не будет ли верным это утверждение для любого описанного четырехугольника? А n-угольника? Мы еще вернемся к обобще- ниям этой задачи. Задача 15. В треугольнике АВС угол С — тупой, D — точка пе- ресечения прямой DB, перпендикулярной к АВ, и прямой DC, пер- пендикулярной к АС. Высота &ADC, проведенная из вершины С, пересекает АВ в точке М. Известно, что AM = а, МВ = Ь. Найти АС. Не почувствовали ли Вы дискомфорта при чтении необо- зримого условия? Зачем эти многочисленные перпендикуляры, когда данные и искомый отрезки являются элементами одного произвольного, но почему-то тупоугольного треугольника? Мо- жет быть для остроугольного или прямоугольного треугольника задача не имеет решения, а может, наоборот, проще решается? Попробуем сформулировать и решить ее для треугольника АВС с прямым углом С. Выполним построения, указанные в ус- ловии и попробуем уловить общее в роли их для решения зада- чи независимо от угла С. Проведем через точку В прямую перпендикулярно к АВ и прямую, проходящую через точку С перпендикулярно к АС. За- мечаем, что точка D совпадает с точкой В. Поэтому высота ДСЛД проходящая через вершину С, совпадает с высотой &САВ, а, сле- довательно, точка пересечения ее со стороной данного ДАВС бу- дет принадлежать гипотенузе данного треугольника (рис. Па). Роль СМ в этом частном случае очевидна: СМ отсекает от ЛАВС треугольник АМС, подобный данному. АС легко выража- ется через отрезки а и b следующим образом: АС = Ja(a + Ь).
Попробуем обосновать подобие аналогичных треугольников в случае тупого угла С, то есть подобие треугольников АМС и АВС. В предыдущем случае подобие треугольников обосновыва- лось равенством углов со взаимно перпендикулярными сторо- нами и наличием общего угла. Общий угол имеется и в этом случае (ZCAM и ZCAB), а вот угол Z.CDH = ZMCA (как углы с соответственно перпендикулярными сторонами) не является уг- лом ДСВ4. Чтобы использовать уже найденный ход решения (пропорциональность сторон в подобных треугольниках) хоро- шо бы доказать равенство Z.CDA = ЛСВА (рис. 116). Указанные углы опираются на один и тот же отрезок СА. Если удастся доказать, что четыре точки А, С, В и D принадле- жат одной окружности, то равенство углов, а, следовательно, и подобие треугольников будет обосновано и задача решена. Читаем еще раз условие, и нас осеняет мысль: вот зачем столько перпендикуляров! Ведь геометрическим местом точек плоскости, из которых данный отрезок виден под прямым углом является окружность, построенная на этом отрезке как на диа- метре (за исключением концов диаметра). Выделяем прямо- угольные треугольники с общей гипотенузой AD : &ACD и AABD, строим на AD как на диаметре окружность, которая проходит через точки В и С. Из подобия треугольников АВС и АСМ нахо- дим соотношение АС = Jafa + b). Задача решена и для тупо- угольного треугольника АСВ. А не решить ли ее и для остроугольного треугольника? Рассмотрев различные случаи остроугольных треугольников (ZC > ZB, ZC = ZB, ZC < ZB), Вы подходите к неожиданному результату: наличие именно тупоугольного треугольника в усло- вии задачи обеспечивало случай решения для ZC> ZB. Но это совсем не влияет на ход рассуждений, на получение подобных треугольников, а значит на выражение стороны АС через отрез- ки а и Ь. Выполните для каждого случая соответствующие ри- сунки и обоснуйте подобие треугольников АВС и АСМ. В большинстве задач приходится выражать длину отрезка через длины других отрезков. Метрические соотношения в раз- личных фигурах чаще всего связаны с пропорциональностью отрезков в подобных треугольниках и в круге. Для обоснования подобия треугольников достаточно равенства двух пар углов. Равенство углов имеет место при параллельности или перпендикулярности соответственных сторон, а так же при использовании свойства вписанных в окружность углов, опи- 36
рающихся на одну и ту же хорду. Последнее связано с умени- ем увидеть четыре точки, принадлежащие одной окружности или видением четырехугольника, вокруг которого можно описать окружность. Рассмотрим несколько таких задач. Задача 16. (Погорелов А.В., Геометрия 7-11. §11 №№ 32, 33). В остроугольном треугольнике АВС проведены высоты AD, BE и CF. Доказать, что биссектрисы углов треугольника DEF лежат на высотах треугольника АВС. Выразить углы треугольника DEF через углы А, В, С треугольника АВС (рис. 12). Проведем в треугольнике АВС высоты AD, BE, CF, построим треугольник EFD и выделим равные углы на основании указан- ных свойств. ЛОАС = ЛСВЕ = a, ЛОАВ = ЛВСР = р, ЛАСР = ЛАВЕ = у как углы с соответственно перпендикулярными сторонами. Точки О, D, С, Е лежат на окружности диаметра ОС, так как ЛООС и ЛОЕС — прямые. На том же основа- нии описываем окружности около четырехугольников OFAE и OFBD. Устанавлива- ем равенство вписанных уг- лов: ЛОЕВ = ЛОСВ = Р, ЛООЕ= ЛОСЕ = у; ЛОРО = ЛОВО = а, ЛООР = ЛОВР = у; ЛОРЕ = ЛОАЕ = а, EOEF= ЛОАР = р. Таким образом, равенства углов OFO = OFE, OEF = OED, ODE = ODF доказывает принадлежность биссектрис &EFD высо- там ДАВС. Выразим углы треугольника EFD через углы А, Ви С. ЛЕРВ = 2а, но из &ВЕС Ла = 90° — ЛС, тогда 2а = 180° — 2ЛС. Аналогично: ЛРОЕ — 2у, но из \ACFZy = 90° — ЛА, тогда 2у = 180° — 2ЛА. ЛРЕО = 2р, но из &CFB ZP = 90° — ЛВ, тогда 2Р = 180° — 2ЛВ. Приобретя опыт разыскивания и удачного использования подобных треугольников с помощью свойств вписанных в ок- ружность углов, попытайтесь самостоятельно решить две задачи: 37
Рис. 13 Задача 17. Доказать, что 3 произведение двух сторон 1 треугольника равно произ- ведению высоты, опущен- ной на третью сторону, на диаметр описанной окруж- ности (рис. 13). Указание. Проведите диаметр через вершину С и рассмотрите подобные тре- угольники &.СНВ ~ ДСЛС1 (обоснуйте подобие). 4 = 4 => ab - hd. h b Ответ: R = Задача 18. Выразите радиус описанной окружности через стороны треугольника. __________abc____________ a+b+c)(a+b-c)(a+c-b)(b+c-a) Успешное решение этих задач может натолкнуть на мысль об установлении метрических соотношений между элементами других многоугольников, вписанных в окружность. К таким за- дачам-теоремам относятся задачи, устанавливающие зависи- мость между отрезками пересекающихся хорд и отрезками се- кущих, проходящих через внешнюю точку, а также теорема Птолемея1, связывающая диагонали и стороны вписанного вы- пуклого четырехугольника. Задача 19. (Теорема Птолемея). Произведение диагоналей вписанного в круг четырехугольника равно сумме произведе- ний его противоположных сторон. Сделав рисунок, сразу обнаруживаем две пары подобных треугольников. Сделайте попытку вывести нужное соотношение из пропорциональности сторон подобных треугольников. Если эта попытка не приводит к успеху попробуем с помощью до- полнительных построений получить другие подобные треуголь- ники (рис. 14). 1 1 Птолемей Клавдий (ок. 100 - 178) — астроном, математик, географ. Уроженец Египта, жил и работал в Александрии. 38
Одна из диагоналей .... (например, АС) разбивает четырехугольник на два тре- / п угольника: АВС и ADC. Об- / / | разуем два других треуголь- / / I |\ ника, подобных выбранным l/\ | I так, чтобы их сторонами бы- | / 1 I ли стороны четырехугольни- \? >xi/ ка и отрезки второй диаго- ]№-...... - /с нали. Этого можно добиться, \ / разбив один из треугольни- ков, образованных второй диагональю BD и двумя сто- ронами. Так, если мы вы- Рис- 14 брали за основные треуголь- ники АВС и ADC, то им подобные следует получать из &ABD (или BDC) выбором на диагонали BD такой точки F, чтобы, со- единив ее с вершиной А (или Q, получить треугольники соот- ветственно подобные треугольникам АВС и ADC. Для обеспече- ния подобия мы располагаем равенством углов, опирающихся на общую хорду. У треугольников AFD и АВС имеется по одно- му равному угла ADF ia АСВ. Следовательно луч AF надо провес- ти так, чтобы Z.DAF= Z.BAC Таким образом имеем &ABC~&AFD, так как Z.ADB = Z.ACB, Z.DAF = ABAC, AABC=AFD. AADC-&.FAB, так как ZACD = ZABF, ZADC = ZAFB, как дополняющие равные углы до 180°, ADAC = AFAB. Из подобия треугольников следует: = = или AC FD = AD-ВС; AC-BF = АВ-CD. AD FD BF АВ Складывая почленно равенства, получим A C(BF + FD) = АВ- CD + AD- ВС или А С- BD = АВ- CD + AD ВС ч.т.д. Рекомендуется провести аналогичные рассуждения для другого разбиения, а именно: для &.BAD и &BCD достроить по- добные треугольники путем разбиения &АВС (или ADC). Задача 20. Эта задача взята из школьного учебника «Геомет- рия 10-11», М., 1992, написанного коллективом автором (Л.С. Ата- насян и др.) из раздела «Задачи повышенной трудности» (№ 814). 39
Рис. 15 Все высоты тетраэдра пересекаются в точке Н.' Докажите, что точка Н, центр О описанной сферы и точка G пересечения отрез- ков, соединяющих вершины с точками пересечения ме- диан противоположных граней тетраэдра лежат на одной прямой (прямая Эй- лера), причем точки О и Н симметричны относительно точки (7 (рис. 15). Дано только одно условие: все высоты пересекаются в одной точке. Наверное оно очень сильное. Но вам не извест- ны свойства такого тетраэдра. (Кстати, он называется орто- центрическим). Эти свойства придется получать самостоя- тельно, чтобы использовать в доказательстве минимум трех предложений, сформулированных в заключении данной зада- чи-теоремы. Просмотрев условия предыдущих задач из этого раздела, мы обнаружим задачи-спутники (№ 769, № 792), решение которых можно будет использовать при решении задачи №814. Добытая информация о многочисленных перпендикуля- рах в данном тетраэдре и необходимость доказать специаль- ное расположение трех точек на одной прямой побуждает привести в готовность соответствующий теоретический мате- риал и выбрать метод для доказательства по своему вкусу: чисто геометрический или векторный. Наличие перпендику- лярности многих отрезков позволит использовать важное свойство скалярного произведения, а доказательство принад- лежности точек одной прямой с указанием отношения между ними можно свести к установлению линейной зависимости векторов. Так как отрезки, соединяющие точку Н с вершиной тетра- эдра, перпендикулярны противоположным граням, то они пер- пендикулярны любым прямым, лежащим в этих гранях. Поэто- му удобнее всего ввести радиус-векторы вершин тетраэдра, при- 40
няв за начало точку Н. Это даст возможность выразить через эти векторы все остальные, нужные для доказательства векторы, вплоть до вектора HG, а также использовать необходимое и достаточное условие ортогональности векторов для выражения вектора НО. Пусть НА = г\ , НВ = г2 ,НС = ry , HD = г4 . Тогда АВ = ry- q ,ВС - гу-гу ,АС-гу- ri DA = Г] - r4 ,DB = Гу- r4 ,DC = гу- г4 Чтобы выразить векторHG через Ц , г2, г3 , г4 , достаточно выразить через эти векторы хотя бы один из векторов AG, BG ,CG, или DG. G — точка пересечения отрезков АА\ и ВВ\, где Ai и В} — центры тяжести треугольников BCD и ACD. AAi = AD + ^(DC+DB) = Г4+Г2+^~3 г! ; ЛС = а Д , BBi = BD +\{ОА + DC ) = BG^BBi. Найдем а и p из условия: AG+ GB = AB. —> —> —> —> > о —> —> —> —> —> —> a AAt- pBJ?) = AB или у (/2+ гз~3 И) - у(*! + гз~3 Гг) ~ г2+ Л = О (а а iA”* (а РА~> ( Р 1^”* ~п ^- + p-lJr2+^--^Jr3-^a+|-ljr1 = 0. —(х В Так как г{, г2 и г3 линейно независимы, то j - j = 0, a = р и a + f - 1 = 0, a = р = 4 • 3 4 41
Тогда 7g = | ИЙИ; ic = . hg- HAt AG=r'+ Zlilzllli; /7с = j-’-'i-y1-'*. 4 4 Так как HD±AB, HD1BC, to r4(r2-r}) = 0, r4(r3- r2) = 0- Следовательно rx r4 = r2 r4 = r3 r4 . Аналогично Л Ъ = r2 r3 = r3 r4 = r, r2 . Отсюда получаем r, r4 = r2 r4 = r3 r4 = r2 r3 = i\ r3 = /] r2 (*). Предположим, что точка M обладает тем свойством, что НМ = 2 HG. Покажем, что эта точка одинаково удалена от вершин тетраэдра, то есть совпадает с точкой О. Вектор НМ = 2 HG, то есть НМ = . Тогда 2 2 Г|+Г2-Г3+Г4 . ^м- rx+r2+rj+r4 2 ’ 2 Используя равенство (*), найдем скалярные квадраты векторов МА,МВ,МС и MD,u убедимся, что точка Л/одинаково удалена от вершин. Задача 21. В прямоугольном параллелепипеде ABCDAiBiQDi I AD I = a, I АВ I = b, IЛЛ11 = с. Найти расстояние между (AD\) и (DBX) и вычислить его при а = 4, 6 = 6, с = 4-Уз. 1. Геометрическое решение. Прямые AD\ и DBX — скрещивающиеся. Чтобы найти расстояние между ними, достаточно через одну из них (например, DBX) провести плоскость П, параллельную другой прямой (AD\). 42
Плоскость П вполне определяется прямыми (DB\) и (DE), где (Д£)| I (AD\). Плоскость ГТ пересекает грани парал- лелепипеда по FB\PD (Рис. 16). Искомое рас- стояние сводится к рас- стоянию от произволь- ной точки прямой ADi до плоскости n(EDB\). Най- дем расстояние от точки D\ до плоскости EDB\. Рис. 16 Проведем D\KE В\Е. Тогда по теореме о трех перпендикулярах DK1. EF. Значит, плоскость DD\KL пл. П. Проведем D\L 1 DK. I D\L I — искомое расстояние. Проектируя ортогонально прямую AD\ на плоскость П, получим прямую A'D\', параллельную прямой AD{. Точку пересечения прямых (Л'Р/) и (DB\) обозначим через М'. Эта точка является проекцией точки М, принадлежащей прямой (ЛР1). ММ' — искомый перпендикуляр. Длина этого перпенди- куляра является расстоянием между скрещивающимися пря- мыми. Для вычисления этого расстояния можно найти рас- стояние от любой точки прямой (Л/>1) до плоскости П. В на- шем случае (рис. 16) это D\L. Из треугольника D\EF следует I EF\ 2 = а2 + &/4 = , I Д1А1 = аЬ/у/4a2 + b2 . Из тре- 2Л2 угольника D\KD имеем |Z)Xl2 = с2 + —57 и I D\L\ = 4а2 + Ь2 = I ММ'\ = , аЬс . ^4а2с2 + Ь2(а2+с2) 2. Векторное решение. Введем векторный базис. Пусть базисными векторами будут DA = а , DC = b , DD} = с , а их модулями соответственно а, b 43 4
и с. Найдем разложение данных векторов AD{ и ОВХ по базис- ным (рис. 17): AD} = - а + с , DBX - а+ Ь+ с . Пусть MN — искомый отрезок общего перпендикуляра. Вы- разим вектор MN через базисные: MN = MA+AD+DN; MN = -aADl+ AD+$DBX = -a(a + c) + (a) + P(a + b+ c) = = a a - ас-о + ра + р^ + рс. Так как MN 1ADX и MN 1DBX , то скалярные произведе- ния (MN • AD}) = 0, (MNDBX ) = 0. Из этого условия находим аир. (a а - а с - a + Pa + p/>+Pc)(-o + c) = 0. (а а-а с - a + Po + pZ>+Pc)(o+ b+ с) = 0. Учитывая ортогональность базисных векторов, получим: 44
(a - р + 1 )й2 + (р - a)c2 = О, (а + р - 1)о2 + р/>2 + (р - а)с2 = О. Из этой системы находим значения а и р, а затем и модуль вектора MN. ^4а2с2+Ь2а2+с2Ь2 3. Координатное решение. Нахождение расстояния между скрещивающимися прямыми может быть сведено как и в первом случае к нахождению рас- стояния от любой точки одной прямой до плоскости, которая проходит через другую прямую, параллельно первой. Эту плос- кость можно задать так: пусть Р — середина [АВ] nF - середи- на [PiCj], тогда, (PF) | | (ADi), а плоскость DPB\F проходит че- рез (DB\), и C4Z>i) | | плоскости DPB\. Выбрав в пространстве прямоугольную систему координат, найдем уравнение плоскости DPB\ а затем и расстояние от ка- кой-нибудь точки прямой (AD\) до этой плоскости. Пусть D(Q, 0, 0) — начало координат, (DA) — ось абсцисс, (DC) — ось ординат а (DD\) — ось z -аппликат. Тогда уравнение плоскости, определяемой точками ДО, 0, 0), Р(а, Ь/2, 0) и В\ (а, Ь, с) будет иметь вид: bcx — 2асу + abz = 0. Z>i(0, 0, с) или А(а, 0, 0) — точки прямой (ЛД). Расстояние abc . ^4а2с2 + b2a2 + с2Ь2 р (Д, пл. DPB\) = 4. Координатно-векторное решение. Сущность его в том, что базисные векторы задаются коор- динатами DA(a, 0, 0), £>С(0, Ь, 0), />Д(0, 0, с). Аналогично рас- суждению во втором случае, находим вектор MN(x,y,z) и запи- сываем условие перпендикулярности векторов (равенство нулю скалярного произведения) тоже в координатной форме: MN = y/x2+y2+z2 abc ^4а2с2 + b2a2+с2Ь2 45
Задача 22. Из всех прямоугольников, вписанных в окруж- ность найти прямоугольник наибольшей площади. Пусть АО = R, АВ = х; тогда ВС = ^4R2 - х2 (рис. 18). Площадь прямоугольника S(x) = Xyj4R2 - х2 = ^4R2x2 -х4 (при внесении множителя х под знак радикала учтено, что х > 0). Функция S(x) определена в замкнутом промежутке [-2Л, 2Л], но по условию задачи может удовлетворять лишь внутренняя точка промежутка [0, 2Л]. В концевых точках х = 0 и х = 2R функция S(x) = 0, однако для этих значений х прямоугольник не сущест- вует, он «вырождается» соответственно в диаметры PQ и MN. Геометрически очевидно, что среди прямоугольников, вписан- ных в окружность, нет прямоугольника с наименьшей площа- дью. Если х увеличивается от х = 0 до х = 2R, то функция сна- чала возрастает, а затем убывает. Значит, в открытом промежут- ке (0, 2R) существует такое значение х для которого функция S(x) принимает максимальное значение. Учитывая, что в про- межутке (0, 2R) функция S(x) достигает наибольшего значения в той же точке, что и функция Дх) = &(х) = 4Л2х2 — х4, находим f'(x) = 8Л2х — 4х3 = 4х(2Л2-х2). Уравнение; /'(х) = 0 имеет три корня: X] = 0, Х2 = —R-J2 , хз = Я>/2 . Но условию удовлетворяет только последний из них. При х = R^2 и ВС = Rj2 , то есть искомый прямоугольник есть квадрат. Рис. 18 Эту задачу можно ре- шить иначе. Пусть а — угол между диагоналями прямоугольника, тогда S = 2R1sin а. Функция 5 принимает максимальное значение при а = я/2. Сле- довательно, искомый пря- моугольник есть квадрат. 46
Ill часть ЗАДАЧИ - ВСЕМОГУЩИЙ КОНТРОЛЕР И РЕПЕТИТОР 1. Контрольный минимум для проверки геометрических знаний Задача 1. Дан угол и внутри него две точки А к В. Постро- ить трехзвенную ломаную минимального периметра и такую, чтобы ее концами были данные точки А и В, а две другие — по одной на сторонах данного угла (Рис. 19). Задача 2. Построить тре- . угольник, сторона которого у' ' проходила бы через данную точку М, а биссектрисы его лежали бы на трех данных пересекающихся в одной точ- у' ке прямых (г, h (рис. 3). у' Задача 3. В плоскости .у д заданы две прямые а и b и не / * принадлежащая им точка С. ......... .... Построить равнобедренный рис 19 прямоугольный треугольник АВС так, чтобы вершина прямого угла совпадала с данной точ- кой С, а две другие вершины А и В лежали соответственно на прямых а и Ь. Задача 4. В плоскости заданы пять точек, из которых ника- кие три не лежат на одной прямой. Построить параллелограмм так, чтобы одна из этих точек являлась точкой пересечения диагоналей, а каждая из остальных принадлежала одной пря- мой, содержащей сторону параллелограмма. Задача 5. Построить четырехугольник, если заданы три сто- роны и углы, прилежащие к четвертой стороне. Задача 6. В данный ‘ сегмент, угловая величина которого меньше 2d, вписать прямоугольник с заданным отношением сторон так, чтобы две его вершины принадлежали хорде, а две другие дуге сегмента. Задача 7. Дан треугольник АВС и внутри его точка Q. Про- вести через эту точку прямую I так, чтобы она рассекала тре- угольник на две равновеликие фигуры. 47
Задача 8. Доказать, что сумма длин диагоналей выпуклого четырехугольника меньше периметра, но больше полупериметра этого четырехугольника. Задача 9. Доказать, что параллелограммы, вписанные в прямоугольник так, что их стороны параллельны его диагона- лям, имеют один и тот же периметр. Задача 10. В треугольнике АВС через вершину В и середи- ну медианы AM проведена прямая / до пересечения со сторо- ной АС в точке F. Найти отношение I BD I: I DF\, где D — се- редина AM. Ответ: |ДР|: |ДГ| = 3 : 1. Задача 11. В треугольнике АВС, угол С — тупой, D — точка пересечения прямой DB, перпендикулярной к АВ, и прямой DC, перпендикулярной к АС. Высота треугольника ADC, проведенная из вершины С, пересекает АВ в точке М. Известно, что I AMI = а, IВМI = Ь. Найти I АС). (См. Образец решения №15). Ответ: I АС\ = Ja(a + Ь) . Задача 12. Доказать, что высоты треугольника явля- ются биссектрисами углов треугольника, образованного основаниями высот. Задача 13. В концах диаметра проведены к ок- ружности две касательные. Доказать, что отрезок любой касательной, заключенный между двумя данными каса- тельными, виден из центра под прямым углом. (Рис. 20). Задача 14. Известно, что п (п > 3) диаметров делят окруж- ность на 2п равных дуг. Возьмите любую точку Q внутри круга. Докажите, что основания перпендикуляров, опущенных из этой точки на все диаметры являются вершинами правильного л-угольника. Задача 15. Доказать, чтобы в выпуклый четырехугольник можно было вписать окружность необходимо и достаточно, что- бы суммы противоположных сторон его были равные. 48
Задача 16. Шесть точек пересечения прямых, про- веденных через основания высот произвольного тре- угольника перпендикуляр- но к двум другим сторонам, лежат на одной окружно- сти. Доказать. (Рис. 21). Задача 17. Во всяком тре- угольнике точки, симметрич- ные с точкой пересечения вы- сот относительно трех сторон треугольника, лежат на опи- санной окружности. Доказать. Задача 18. Три окруж- ности, имеющие своими диаметрами стороны тре- угольника, попарно пересе- каются в точках, принадле- жащих сторонам этого тре- угольника. Доказать. Задача 19. В произволь- ном треугольнике АВС: О — центр описанной окружности, Q — центр тяжести и Е — такая точка прямой OQ, что QE — 2OQ (Q между О и Е). Доказать, что высоты тре- угольника АВС проходят че- рез точку Е. (Рис. 22). Задача 20. Три окружности oi, 02, °з одного радиуса г про- ходят через точку О. Окружности попарно пересекаются, в точ- ках А, В и С (<jj п оз — А, 02 п оз = Д 03 г> oi = Q. Докажи- те, что радиус окружности, проходящей через точки А, В и С, равен г. Задача 21. Плоские углы трехгранного угла равны а = 45°, Р = 45°, у = 60°. Вычислить величину двугранного угла, заклю- ченного между гранями плоских углов в 45°. Ответ: 90°. 49
Задача 22. Доказать, что отрезок, соединяющий середины двух проти- воположных ребер тетраэдра прохо- дит через центр тяжести любого по- перечного сечения, параллельного этим двум ребрам. (Рис. 23). Задача 23. В тетраэдре с пря- мым трехгранным углом при верши- не О даны площади Qi, Q2, Q3 трех граней, сходящихся в этой вершине. Найти площадь Q грани, противоле- жащей вершине О. СКъ&т. Q = + Сг + Сз • Задача 24. Построить тетраэдр по шести его ребрам. Задача 25. Доказать, что центры граней куба могут являться верши- нами вписанного в этот куб пра- вильного октаэдра (рис. 24). Задача 26. Найти расстояние между скрещивающимися диагоналями двух смежных граней куба с ребром а. Ответ: —-. 3 Задача 27. В основании пирамиды SABCD лежит трапе- ция ABCD (ВС 11 AD, AD : ВС = 2). Через вершину S пира- миды проведена плоскость, параллельная прямой ВС и пе- ресекающая отрезок АВ в точке М такой, что AM: МВ = 2. Площадь получившегося сечения равна Q, а расстояние от ребра ВС до плоскости сечения равно d. Найти объем пира- миды. Ответ: V= . 4 Задача 28. Доказать, что плоскость, делящая пополам дву- гранный угол при каком-либо ребре тетраэдра, делит противо- положное ребро на части, пропорциональные площадям граней, заключающих этот угол. 50
Задача 29. Сторона ос- нования четырехугольной пирамиды равна а, боковое ребро равно Ь. Найти пло- щадь сечения, проведенного через середины двух смеж- ных сторон основания, па- раллельного боковому ребру, которое проходит через точ- ку пересечениях этих сторон. Ответ: -^-aby/l . Задача 30. Найти пло- щадь сечения куба плоско- стью, проходящей через сере- дины двух смежных ребер куба и наиболее удаленную вершину его, если длина реб- ра куба равна а. (Рис. 25). Ответ: la24vi 24 Задача 31. Вычислить угол между скрещивающи- мися диагоналями: Рис. 26 а) двух соседних граней куба; б) двух йротивоположных граней куба. Ответ: 60° и 90°. Задача 32. В основании наклонного параллелепипеда лежит ромб ABCD со стороной а, причем, Z.BAD = . Боковое ребро АА' расположено в плоскости проходящей через АС перпенди- кулярно к плоскости ромба; длина АА' также равна а, а угол А С АС = у. Три другие боковые ребра, наклонные к основа- нию П ВВ', СС и DD'. Переменная плоскость (IT), парал- лельная плоскости П, пересекает АВ', в точке М, ВС' — в точке N, CD' — в точке Р и DA' — в точке Q. Каков тип четырех- угольника? Найти геометрическое место середины МР, когда точка Л/описывает АВ’ (Рис. 26). 51
Рис. 27а А Рис. 276 Задача 33. Шар касается всех 12-ти ребер куба. Найти объем той части, которая за- ключается внутри этого куба, если известно, что ребро куба равно а. (Рис. 27 а, б). „ о3(15 - 8-V2) Ответ: — 'п. 12 Задача 34. Найти радиус сферы, описанной около правиль- ного тетраэдра, зная длину ребра этого тетраэдра а. (Рис.28 а, б). Рис. 286 Рис.28а Задача 35. В сферу радиуса R вписана треугольная пирамида так, что центр основания ее совпадает с центром сферы. Стороны основания пирамиды а, b и с. Найти отношения полной поверхности пирамиды и сферы, объема пирамиды и шара (Рис. 29). Ответ: a-JSR2 - a2 + b^8R2 - b2 + c-JsR2 - c2^R+abc 16лЛ3 abc 16лЯ3 ’ Задача 36. Построить изображение куба, вписанного в сферу. 52
Задача 37. Центр шара, вписанного в правильную четырех- угольную пирамиду, делит высоту пирамиды в отношении 1 : 2. Найти двугранный угол при основании пирамиды. Ответ: 60°. Задача 38. Дан шар радиуса R. Найти радиус основания и об- разующую вписанного в него цилиндра, имеющую наибольшую площадь боковой поверхности. Ответ: г = -Д; / — R^2 . V2 Задача 39. Параллельно оси цилинд- ра проведено сечение, отстоящее от оси на расстояние d и отсекающее от окруж- ности основания дугу в а радиан. Пло- щадь сечения равна S. Найти объем ци- линдра. (Рис. 30). Ответ: И„ = . sin а Задача 40. Около шара радиуса 12 см описан усеченный конус, радиусы основа- ния которого относятся как 4 : 9. Найти объем усеченного конуса. Ответ: 4256 п см3. Рис. 30 Задача 41. В правильную четырехугольную пирамиду вписан шар. Доказать, что отношение объемов этой пирамиды и шара рав- но отношению площадей полной поверхностей пирамиды и сферы. Задача 42. Площадь поверхности шара, вписанного в конус равна площади основания конуса. Найти угол между образую- щей конуса и плоскостью его основания. Ответ: а = 2 arctg . Задача 43. В треугольную призму, основанием которой являет- ся прямоугольный треугольник с острым углом 30° вписан круго- вой цилиндр. Найти отношение площадей боковых поверхностей и объемов описанного цилиндра, призмы и вписанного цилиндра. Ответ: : S2 : S3 = 2л : (7з + 3): л(>/3 - 1); И : К2 : И3 = 2л : V3 : (2 - . 53
S' A. 04 A, 1 О £ Рис. 31a Рис. 316 Задача 44. На плоско- сти лежат три равных ша- ра радиуса R, попарно ка- сающиеся друг друга. Ос- нование конусов принад- лежат этой плоскости, а данные шары касаются их соответственно внутрен- ним и внешним образом. Высоты конусов — 4R. Найти отношения объемов этих конусов. (Рис. 31). Задача 45. На каком расстоянии от вершины конуса с высо- той Н нужно провести сечение, чтобы оно разделяло боковую поверхность конуса на две части, имеющие равные площади? л н Ответ: -==. 41 Задача 46. Диагональ прямоугольника составляет с одной из сторон угол а. Найти отношение объемов цилиндров, образо- ванных вращением прямоугольника около каждой из смежных сторон. (Рис. 32). Ответ: V1:V2 = sina : cosa. Задача 47. В конус с радиусом основания R и высотой И впи- сан цилиндр. Найти линейные размеры цилиндра rnh, при кото- рых его объем наибольший. (Рис. 33). л R Н Ответ: г = 2 —; h = —-. 3 3 54
Задача 48. Около правильной треугольной пирамиды опи- сан шар радиуса R. Вычислить объем пирамиды, если ее ребро наклонено к плоскости основания под углом а. (Рис. 34 а, 34 б). JlR3 sin2 2а sin а 2 Ответ: V= Задача 49. Основанием пирамиды служит прямоугольник со сторонами а и 2а. Все боковые ребра равны между собой и рав- ны 10 75. Через диагональ основания параллельно боковому ребру проведено сечение. Найти значение а, при которых пло- щадь сечения является наибольшей. (Рис. 35). Ответ: При а = 20; Q = 200 ед2. Задача 50. В данную правильную усеченную треугольную пирамиду с боковым ребром I мож- но поместить сферу, ка- сающуюся всех граней, и сферу, касающуюся всех ребер. Найти сто- роны основания пира- миды. Ответ: а = 4(3 + Тб ); b = 4(3 - 4б ). Рис. 35 55
2. Задачи-спутники, помогающие выбрать нужные знания Задачи-спутники к задаче № 1. 1.А. Внутри острого угла аОв дана точка С. Построить тре- угольник АВС наименьшего периметра так, чтобы одна из вер- шин совпала с данной точкой С, а две другие А и В лежали со- ответственно на сторонах Оа и ОЬ. 1.Б. Дана прямая / и точки А и В вне ее. На прямой / найти такую точку М, чтобы сумма отрезков AM + МВ имела наи- меньшую длину: а) точки Ли В лежат по разные стороны от /; б) точки А и В лежат по одну, сторону от /. 1 .В. По одну сторону от прямой / заданы точки А и В. Из- вестна также точка В', симметричная точке В относительно прямой /, и точка пересечения прямых / и АВ' — Mq. Доказать, что для любой точки М прямой I, отличной от Mq, справедливо неравенство AMq + ВМо < AM + ВМ. Задачи-спутники к задаче Ns 2. 2 .А. Известно, что две пересекающиеся прямые /д и /в яв- ляются биссектрисами углов А и В, а две не принадлежащие им точки Ми Nлежат на прямых, на которых лежат стороны этого треугольника. Построить треугольник АВС. 2 .Б. Построить треугольник АВС, если данные пересекаю- щиеся прямые /А и /в являются биссектрисами его углов А и В, а данная точка С, не принадлежащая этим прямым, его вершиной. 2 .В. Построить треугольник АВС, если дацы две его верши- ны В и Си прямая I, на которой лежит биссектриса угла А. Задачи-спутники к задаче Ns 3. З .А. Дана прямая I и точка О вне ее. Найти образ прямой I при повороте . З .Б. Дан равнобедренный прямоугольный треугольник АВС с прямым углом С. Укажите центр и угол поворота, при котором образом катета СА будет другой катет СВ. Задачи-спутники к задаче № 4. 4 .А. Дан параллелограмм ABCD. Пусть М — любая точка прямой АВ. Доказать, что точка М', симметричная точке М от- носительно точки пересечения диагоналей параллелограмма ле- жит на прямой DC. 56
Задачи-спутники к задача № 5. 5 .А. Построить четырехугольник, если заданы две противо- положные стороны, диагонали и угол между диагоналями. 5 .Б. Построить трапецию по четырем сторонам: двум осно- ваниям и двум боковым сторонам. Задачи-спутники к задаче № 6. 6. А. В данный сектор, угловая величина которого меньше 180° вписать квадрат так, чтобы две его вершины принадлежали дуге сектора, а две другие — радиусам. 6. Б. В данный треугольник АВС вписать треугольник KLM так, чтобы стороны последнего были параллельны медианам первого. Задачи-спутники к задаче № 1. 7 .А. Дан треугольник АВС и на его стороне АВ точка М. Провести через эту точку прямую / так, чтобы она отсекала от данного треугольника такой треугольник, площадь которого в два раза меньше площади данного. Задачи-спутники к задаче № 8. 8. А. Использовать теорему о неравенстве треугольника. Задачи-спутники к задаче № 9. 9 А. Найти периметр параллелограмма, вписанного в прямо- угольник так, что одна из сторон вписанного параллелограмма делит одну из сторон данного прямоугольника в отношении т : п. 9. Б. Найти периметр параллелограмма, вписанного в пря- моугольник так, что вершинами параллелограмма являются се- редины сторон прямоугольника. Задачи-спутники к задаче № 10. ЮЛ. Средние линии треугольника АВС вместе с половинами сторон образуют три параллелограмма. Через точку пересечения диагоналей D одного из них и вершину данного треугольника, отличную от вершины этого параллелограмма проведена прямая р, пересекающая стороны параллелограмма в точках Р и О. Найти отношение DP: ВО, где В — вершина треугольника АВС. 10 .Б. В треугольнике ABC MN — средняя линия (М сере- дина АВ, N — середина ВС). Доказать, что любой отрезок BQ, где точка Q принадлежит стороне АС, делится средней линией пополам. 57
Задачи-спутники к задаче №11. 11 .А. На отрезке AD как на гипотенузе построены в одной полуплоскости прямоугольные треугольники ABD и ACD. Высо- та СР треугольника ACD, опу- щенная на гипотенузу, пересе- кает сторону АВ в точке М. До- казать: ЬАМС - &АСВ. (Рис. 36). Задачи-спутники к задаче №12. 12 .А. В треугольнике АВС Q — точка пересечения высот ЛЯА, 5Яв и СЯс- Доказать, что = ZBQHC = ЛС(2Нй. 12 .Б. Доказать, что во вся- ком четырехугольнике, вписан- ном в окружность, сумма проти- воположных углов равна 180°. Задачи-спутники к задаче №13. 13 .А. Около окружности описана равнобедренная трапе- ция. Отрезки, соединяющие центр окружности с вершинами разбивают ее на четыре тре- угольника. Доказать, что тре- угольники, содержащие боко- вые стороны, прямоугольные. 13 .Б. Из точки Q, взятой вне окружности проведены к ней две касательные. Пусть М и N — точки касания. Доказать, что ДОЛ/0 = дояе. Задачи-спутники к задаче № 14. 14 .А. найти геометрическое место середин хорд окружности, проходящих через данную точку, взятую внутри этой окружности. 14 .Б. В произвольной окружности с центром в точке О проведены радиусы ОМ\, ОМ?, ОМт,, ОМ4 так, что они образу- ют равные центральные углы по 30°. На отрезке ОМ$ взята произвольная точка Q, и на диаметре OQ построена окруж- ность, пересекающая данные радиусы соответственно в точках М, 'Vz, Q, N*- Доказать, что £N\NiQ — Z.N2QN4. Найти вели- чину этих углов. (Рис. 37). 58
Задачи-спутники к задаче № 15. 15 .А. Доказать: чтобы около выпуклого четырехугольника можно было описать окружность необходимо и достаточно, что- бы сумма противоположных углов его была 180°. Задачи-спутники к задаче № 16. 16 .А. В остроугольном треугольнике АВС проведены высоты AAi, ВВу СС|. Найти углы треугольника А] В] Су зная углы тре- угольника АВС. 16 .Б. Доказать, что биссектрисы треугольника А]В\С\ лежат на высотах треугольника АВС, где Ау By С\ — основания высот АВС. 16 .В. В треугольнике АВС через основание высоты АА\ проведены прямые, перпен- дикулярные АВ и АС до пе- ресечения в точках В\ и Су В полученном треугольнике ABiC[ снова через основание высоты AAi опущены пер- пендикуляры на стороны ABi и АСу А2В2 и А2С2. Доказать, что В2С2 II ВС. (Рис. 38). 16 .Г. В четырехугольни- ке ABCD, вписанном в ок- ружность, диагонали АС и BD пересекаются в точке Q. Доказать, что ABCQ и AADQ, ABAQ и ACDQ - попарно по- добны. Задачи-спутники к задаче №17. 17 .А. В треугольнике АВС, вписанном в окружность прове- дены прямые, содержащие высоты этого треугольника до пере- сечения с окружностью в точках Ау В}, Су Пусть Q — точка пересечения высот. Доказать, что точки Ay By Q симметричны точке Q относительно сторон треугольника ВС, АС и АВ. Задачи-спутники к задаче № 19. 19 .А. Доказать, что высоты треугольника пересекаются в одной точке. 19 .Б. Прямые, проведенные через вершины данного треуголь- ника АВС, параллельно противоположным сторонам образуют но- вый треугольник А\В\Су Доказать, что центр описанной около 6A\B\Ci окружности совпадает с ортоцентром треугольника АВС. 59
Задачи-спутники к задаче № 20. 20 .А. Условия и данные — те же, причем известно, что 00} = ОО2 = ОО3 = OtA = О2А = О2В = ВО3 = О3С = COi. (Почему?). Доказать, что существует такая точка М, что каждый из отрезков МА, МВ, МС равен г. 20 .Б1. Решить задачу средствами планиметрии, акцентируя внимание на свойствах ромба. 20 .Б2. Решить задачу средствами стереометрии, акцентируя внимание на проекцию параллелепипеда. Задачи-спутники к задаче № 21. 21 .А. Доказать теорему, обратную теореме Пифагора. Задачи-спутники к задаче № 22. 22А. Плоскость, проходящая через середины двух противо- положных ребер тетраэдра, делит его на равновеликие много- гранники. Доказать. 22 .Б. БиссектОриальная плоскость двугранного угла тетра- эдра делит противоположное ребро тетраэдра на отрезки, про- порциональные площадям граней, образующих этот двугранный угол. Доказать. Задачи-спутники к задаче № 23. 23 .А. В тетраэдре с прямым трехгранным углом при вер- шине О даны ребра ОА = a, OB = b и ОС = с. Найти объем тетраэдра. л rz abc Ответ: V= —г~. 6 23 .Б. В тетраэдре с прямым трехгранным углом при верши- не О даны ребра ОА = a, OB = b и ОС = с. Найти площадь полной поверхности тетраэдра. ~ е ab+ac+bc+Ja2b2 + а2с2 + Ь2с2 Ответ: S = 2 ' 23.В. В тетраэдре с прямым трехгранным углом при верши- не О даны ребра ОА = a, OB = b и ОС = с. Найти апофему гра- ни, противолежащей вершине О. n <Ja2b2 + а2с2 + Ь2с2 Ответ: -----г— ........ 4а\Ь> 60
Задачи-спутники к задаче № 27. 27.А. В основании пирамиды SABCD лежит трапеция (ВС 11 AD, AD = 2ВС). MN — средняя линия трапеции. Зная объ- ем пирамиды SMNB, вычислить объем пирамиды SABCD. 27.Б. В основании пи- рамиды лежит равносторон- g q ний треугольник АВС. Зная > площадь боковой поверхно- / \ сти пирамиды — Q и рас- М /-___________________—N стояние от вершины основа- / ния до противоположной / 1 боковой грани d, вычислить / \ объем пирамиды SABC. / 1 27.В. В трапеции ABCD • | AD = 2ВС, MN 11AD, и АВ = = 2МВ. Зная площадь тре- угольника ANB, найти пло- Рис- 39 щадь трапеции. (Рис. 39). Задачи-спутники к задаче № 28. 28 .А. Доказать, что отношение площадей двух смежных бо- ковых граней пирамиды равно отношению отрезков сторон ли- нейного угла того двугранного угла, которому принадлежат эти грани. 28 .Б. Доказать, что биссектриса угла треугольника делит противоположную сторону на части пропорциональные боко- вым сторонам. Задачи-спутники к задаче № 29. 29 .А. Построить сечение правильной четырехугольной пи- рамиды SABCD плоскостью, проходящей через середины сторон АВ и AD параллельно ребру SA и доказать, что полученный в сечении пятиугольник можно разбить диагональю на прямо- угольник и треугольник. Задачи-спутники к задаче № 30. 30 .А. Найти площадь сечения координатного угла (х, у, г) с вершиной О плоскостью, заданной тремя точками Л(0, 0, 12), В(4, 4, 0), С(2, 8, 0). Ответ: 5 = 36 4б . 61
ЗО.Б. Три взаимно пер- пендикулярных плоскости, проходящие через точку А, образуют восемь трехгран- ных углов. В одном из них построен куб ABCDAiB[C[Di (рис. 40) так, что вершина куба совпадает с точкой А, а ребра принадлежат ребрам трехгранного угла. Постройте плоскость определяемую точками Ау Е, F где Е и F соответствен- но середины ребер ВС и CD куба. Укажите линии пересечения этой плоскости с гранями куба и гранями трехгранного угла. Задачи-спутники к задаче № 31. 31 .А. Дан куб ABCDA\B\C\Dy Через одну из скрещиваю- щихся прямых, содержащих ребро или диагональ провести плоскость, перпендикулярную другой прямой в следующих случаях: а) скрещивающиеся диагонали двух параллельных граней; б) скрещивающиеся диагонали двух смежных граней; в) диагональ куба и диагональ одной из граней. Задачи-спутники к задаче № 32. 32 .А. На сторонах параллелограмма ABCD от вершин отло- жены равные отрезки: AAq = BBq = CQ = DDq = m. Доказать, что точка пересечения отрезков AqCq и BqDq совпадают с цен- тром симметрии параллелограмма. 32 .Б. Параллельный перенос, определяемый вектором АА' переводит параллелограмм ABCD в А ’В’ С’ D’ (АА', АВ, AD — не компланарны). На сторонах параллелограмма отложены рав- ные отрезки ААо = BBq = CCq = DDq = т. Доказать, что четы- рехугольник AqBqCqDq — образ четырехугольника Aq Bq Cq Dq при параллельном переносе АА' является параллелограммом, причем точки пересечения диагоналей соответственных четырехугольников AqBqCqDq и определяют вектор ЭД = АА ’. 62
Задачи-спутники к задаче № 33. ЗЗА. В квадрат ABCD со сторо- ной а вписана окружность (О, г) и около него описана окружность (О, R). Два взаимно перпендикулярных диаметра, проведенные через точки касания Т\ и разбивают каждый круг на четыре равные части. Найдите площадь следующих фигур (рис. 41): F\ — сектор меньшего круга; Fi — сегмент большего круга; F3 — площадь прямоугольного тре- угольника СТ\Т\ с криволинейной гипотенузой СТ{; F4 — площадь треугольника T\CTi с криволиней- ной гипотенузой l\Ti, F$ — площадь трапеции TiTiTiTj (с пря- молинейными и криволинейными основаниями). ЗЗ .Б. Шар касается двенадцати ребер куба, ребро которого а. Найти объем сегмента этого шара, отсекаемого плоскостью ка- кой-нибудь грани куба. Задачи-спутники к задаче № 34. 34 .А. Найти радиус окружности, описанной около равно- стороннего треугольника со стороной а. 34 .Б. В сферу данного радиуса R вписан правильный тетраэдр SABC. Построить изображение сечения комбинации тел плоско- стью, проходящей через высоту тетраэдра и медиану основания и найти отношение площадей сечений тетраэдра и сферы. Задачи-спутники к задаче № 35. 35 .А. Дан тупоугольный треугольник со сторонами а, Ь и с. Построить окружность, описанную около этого треугольника и выразить площадь треугольника через радиус этой окружности. Ответ: «УД = 4R Задачи-спутники к задаче № 38. 38 .А. В данный круг вписать прямоугольник наибольшей площади. 38 .Б. В треугольник АВС вписать прямЬугольник MNM\N\ так, чтобы две вершины М\ и N\ лежали на АВ, а две другие М и N соответственно на АС и ВС и чтобы площадь этого прямо- угольника была наибольшей. 63
Задачи-спутники к задаче № 40. 40 .А. Около круга радиуса г = 4 см. описана равнобедренная трапеция, основания которой относятся как 3:5. Найти пло- щадь трапеции. Задачи-спутники к задаче Ns 41. 41 .А. В равнобедренный треугольник АВС вписан круг. До- казать, что отношение площадей треугольника и круга равно отношению периметра треугольника к длине окружности. 41 .Б. В равнобедренный треугольник АВС (АВ — основание) вписана окружность с центром О. Доказать, что отношение длин основания треугольника к диаметру окружности равно от- ношению длин боковой стороны к расстоянию от вершины тре- угольника до центра окружности. Задачи-спутники к задаче № 43. 43 .А. Найти углы прямоугольного треугольника зная, что радиус описанной окружности R относится к радиусу вписанной окружности г как 5 : 2. Задачи-спутники к задаче № 44. 44 .А. На плоскости лежат три равных шара радиуса R, по- парно касающиеся друг друга. Основание конуса принадлежит этой же плоскости, а данные шары касаются его извне. Высота конуса R. Найти радиус его основания (рис. 42). В Ответ: -==. Л 44 .Б. Три равных окружности радиуса г попарно касаются друг друга. Найти центр и радиусы окружностей, касающихся данных окружностей внешним и внутренним образом. (Рис. 43). Рис.42 Рис.43 64
Задачи-спутники к задаче № 45. 45 .А. В каком отношении разделит высоту треугольника прямая, проведенная параллельно основанию, если известно, что площадь отсекаемого этой прямой треугольника в три раза меньше площади основания оставшейся трапеции. Ответ: высота разделится на равные части. Задачи-спутники к задаче № 46. 46 .А. Найти отношение объема к боковой поверхности тела, полученного от вращения прямоугольника вокруг оси, содер- жащей данную сторону а, если известно, что диагональ данного прямоугольника составляет с другой стороной угол а. Задачи-спутники к задаче № 49. 49 .А. Найти площадь сечения четырехугольной пирамиды плоскостью, проходящей через диагональ основания и середи- ну бокового ребра, если известно, что основание — прямо- угольник со сторонами а = 6, b = 8, а боковое ребро / = 20. Доказать, что секущая плоскость параллельна противополож- ному ребру пирамиды. 49 .Б. Найти площадь сечения правильной четырехугольной пирамиды плоскостью, проходящей через диагональ основания параллельно боковому ребру, если сторона основания — а, вы- сота пирамиды h. Ответ: 5= . 4 Задачи-спутники к задаче № 50. 50 .А. Найти радиус окружности, вписанной в равнобедрен- ную трапецию, основания которой а = 4(3 + 4б ), b = 4(3 — 4б ), боковые стороны 7=12. Выполнить построение. 50 .Б. Можно ли вписать окружность в трапецию, если из- вестно, что основания а = 3 см, b = 17 см, а боковые стороны /1 = 15 см, (г = 5 см? 3—1224 65
3. Решения конечных задач-спутников 1.В. Пусть М — произвольная точка прямой /. Проведем АВ', AM, ВМ, В'М, BMq, где Mq — точка пересечения I и АВ'. Она существует, так как А и В’ — в разных полуплоскостях относительно /. Тогда в треугольнике АМВ’ АВ' < AM + МВ'. Учитывая, что. АВ’ = АМ$ + М$В’, М$В' = MqB, получим: АМ0 + BMQ< МА + ВМ. 2.В. Используйте свойство биссектрисы угла: биссектриса угла является осью симметрии его. З .Б. Поворот R“ °. 4 .Б. Точка симметрии параллелограмма является его цен- тром симметрии. 5 .Б. Пусть искомая трапеция ABCD и DC —а — верхнее ос- нование, АВ= b — нижнее основание, AD = с и СВ = d — боко- вые стороны. Замечаем, что образом отрезка AD при параллель- ном переносе на вектор DC является отрезок СА'. Тогда по- строение трапеции можно свести к построению АА'ВС по трем сторонам: b — а, с, d. 6 .Б. Пусть в данном треугольнике ААХ, ВВХ, ССХ — медиа- ны. Построим сначала треугольник K'L'M' гомотетичный ис- комому KLM с центром гомотетии в какой-нибудь вершине, например, А и такой, чтобы только две вершины (К' и £') ле- жали на сторонах АВ и А С, а все три стороны были бы парал- лельны медианам: K'L'II ССХ, L'M’llAA,, К’М'\ IВВХ. Тогда точку М на стороне ВС получим в пересечении этой стороны с лучом AM’. Аналогично найдем другое решение, взяв перво- начально K'L’ 11 ВВХ. 7 .А. Применить алгебраический метод. Пусть N — точка пересечения прямой / со стороной ВС. Обозначим стороны &АВС а, Ь, с, отрезок ВМ = т и искомый отрезок BN = х. Тогда площади треугольников MNB и АВС выразим так: Sabc = yacsinp, SMNB = у тх sin£. Учитывая, что SABC = £ £ находим х = ab 2т' 66
9 .Б. Пусть АС= BD= d. KL — средняя линия Д4ВС KL = d/2. Аналогично: LM = d/2. Тогда p = d, 2p = 2d. 10 .Б. Средняя линия треугольника параллельна соответст- вующей стороне. Тогда по теореме Фалеса BS = SQ, где S — точка пересечения BQ и MN. 11 .А. Треугольник АМС и АСВ подобны по двум равным уг- лам: /.САМ — общий, /АСР = /ADC = /АВС. 12 .Б. Вписанный в окружность угол измеряется половиной соответствующего центрального угла. 13 .Б. Треугольники прямоугольные. Имеют общую гипоте- нузу и равные катеты ОМ и ON как радиусы одной окружности. 14 .Б. Воспользоваться свойством углов вписанного четырех- угольника: сумма противоположных углов вписанного четырех- угольника равна 180°. 16 .Г. Обе пары треугольников подобны по трем равным уг- лам. В ДЛВС и &.CDQ /ВАС (значит и BAQ) и /BDC и /QDC измеряются половиной дуги ВС, a /ABD = /ACD как опираю- щийся на хорду AD и, следовательно, измеряющиеся половиной одной и той же дуги AD. Аналогично доказывается подобие тре- угольников BQC и AQD. 17 .А. Докажем симметричность точек Q и А\ относительно прямой ВС. (Рис. 44). QAi ± ВС по условию. Докажем равенст- во отрезков QH{ и Н\А\, где Н\ — основание высоты треуголь- ника АВС. Равенство указанных отрезков следует из равенства прямоугольных треугольников CQH\ и СА\Н\ с общим кате- том СН\ и равными углами QCH[ и А]СН[ (/QCH\ = /ВАН\) как углы с соответственно перпендикулярными сторо- нами. Но /А\АВ = /А\СВ, как вписанные и опирающие- ся на одну хорду. Следова- тельно /QCH\ = AiCHi, а значит, точки Q и А\ симмет- ричны относительно СВ. 18 . Указание: исполь- зовать то, что линия центров двух окружностей является осью симметрии этих окруж- ностей и средней линией тре- угольника. з* 67
Рис. 45 27.В. Решение: Введем обозначения (рис. 45) ВС = а, AD = 2а, MN=c, ВМ= 1,АМ = 21. Площадь SAMN — 2S так как AM = 2ВМ. и 52 площади SBNC и SAND. Из подобия треугольников МВК и ABF имеем: |, а 3 откуда с = 4а/3. Выразим 5] и Si через данную величину S. Так как треугольники MBN и BNC име- 51 а За 3 „ ют одну и ту же высоту, то -^ = - = ^- = 7,то есть 5[ = 35/4. . 5, 2а 2а 3 3 с -ю п Аналогично: —j- =— = откуда S? = 35. Площадь тра- 25 с 4а 2 пеции SAbcd = 5i + 35 +52 = 275/4. 28.Б. Провести через одну из вершин основания (Q (Рис. 46) треугольника АВС прямую, параллельную стороне не содержащей эту вершину, до пересечения с прямой, со- держащей биссектрису угла В. Пусть биссектриса угла В - BL и AL = т, LC = п, АВ = с, ВС = а и СВ\ 11 АВ где В\ лежит на прямой BL. Рассмотреть подобные треугольники ALB и CLB\. Показать, что В\ С = ВС. 30. Указание. Площадь искомого сечения — пяти- угольника можно найти как разность площадей треугольника — сечения трехгранного угла данной плоскостью и двух тре- угольников, отсекаемых от него гранями куба (рис. 47) то 68
31 .А., ЗЕБ. Сделайте соответствующий рисунок. 32 .Б. Так как отрезки AAq и CCq, В Bq и DDq по- парно равны и параллель- ны, то четырехугольник AqBqCqDq — параллело- грамм с центром симмет- рии Oq (рис. 48). Так как, параллельный перенос — движение, то Aq Bq Cq Dq также параллелограмм, равный AqBqCqDq и ОцРо = 33.Б. Радиус шара И = 1 = а(л/|--^ • Тогда Усегм. = Я2л(ЗА - Д)/3. Подстав- ляя значения R и Н, выраженные через ребро куба, получим Усегм. = ^(15 - 8V2 )/12. 34 .Б. В плоскости сечения будем иметь (рис. 49): АА\ = h — высота равностороннего треугольника — грани правильного тет- раэдра с ребром a. hfi, = A4i = , О\А — . Из АОО^А выра- 2 3 I / /хл2 жаем 00} ~ I > а из &SO\A имеем: SO2 + <?i Л2 = SA2 или (R + OOt )2 + ОХА2 = а2. Подставив вместо ОО\ и О\А их значения после преобразо- 2R46 вания находим а = —-—. 36. Указание. Построе- ние изображения можно вы- полнить в следующем порядке: 69
а) строим изображение сферы; б) строим изображение двух одинаково удаленных от центра параллельных сечений; в) строим изображения квадратов, вписанных в сечения; г) строим изображения остальных ребер куба. 38.Б. Введем обозначе- ния (рис. 50). АВ = т, CD = h — постоянные величины, ММ\ = = NN{ = х, MN = M}N{ = у - переменные, причем искомая величина S = ху. Выразим площадь прямоугольника S как функцию одной из пере- менных величин, например х. Для этого рассмотрим подобные треугольники АВС и MNC. т h m(h-x) — = — откуда у = —4—-. у h-x h ио cv \ /И (Л - х)х Но S = ху, следовательно S(x) = ' . Для определения хь, при котором площадь прямоугольника принимает наибольшее значение найдем производную функции и ее критические точки 5-(Х) = Рис. 51 Следовательно SMax при л?о = Л/2. 41.Б. Пусть в АЛИС CD — высота, СО — расстояние от вершины С до центра вписанной окружности, Т — точка касания окружности боковой стороны СВ. Рассмотрите подобные тре- угольники DBC и ТОС. Учиты- вая, что основание АВ = 2 DB, а диаметр окружности 2ОТ полу- чите искомое отношение. 44.Б. Пусть О], б?2 и Оз — центры, а 7\, Т2, Tj — точки попарного касания данных окружностей (рис. 51). Тогда 70
центр искомых окружностей, касающихся трех данных — суть центр правильного треугольника О со сторонами 2г каждая. Расстояние от центра О до любой вершины 7} равно 2/3 со- ответствующей высоты й, где h = д/4г2 -г1 = г4з. Радиусы искомых окружностей выражаются через 2й/3 и г следую- щим образом: = 2й/3 — г, - Ih/'i + г. ~ „ г(2Л±3) 3 46. А. Пусть в прямоугольнике ABCD сторона AD = а, Z.CAB = а, ось вращения — прямая AD. При вращении получим цилиндр, радиус которого АВ = г, а высота AD = ВС = а (ВС — образующая). Объем цилиндра V = nr^h, а боковая поверх- ность Sq = 2nrl. Из SABC АВ = г = a ctga. Тогда V = na3ctg2a, а S = 2Tta2ctga и ViS = actga:2. 47. (См. образцы решения). 48. Указание. Целесообразно воспользоваться сечениями двумя взаимно перпендикулярными плоскостями: плоскостью основания — равносторонний треугольник, вписанный в окруж- ность и треугольник, полученный в сечении пирамиды плоско- стью, проходящей через высоту пирамиды и высоту треугольника, лежащего в основании пирамиды (рис. 52). Решение. Из ASOK имеем SK = Asina, тогда SB = 2 Asina; из SSO\B SO\ = 5Asina и BO\ = SO^ ctga. BO\ = SB sina ctga = 2A sin2a ctga = 2A sina cosa = Asin2a. Итак, BO\ = Asin2a, DB = 3Asin2a/2. Пусть AB = а, тогда a = А Л sin 2a, h = SO} = 2ASin2a, И= 2д) w? a Рис. 52 71
49. Решение. 1) Най- дем условия существования пирамиды. Из &DSB имеем DB < DS +SB. Так как DO = ОВ и DS = SB, то это неравен- ство можно переписать так: OD < DS. (рис. 53). Следовательно для су- ществования пирамиды не- обходимо, чтобы ее боковое ребро было не меньше по- ловины диагонали основа- ния, то есть 10 75 > АО, где АО= а45 /2; тогда а < 20. 2) Пусть ОЕ11 AS; тогда ОЕ — средняя линия SASC и SE= ЕС. 3) Пусть ЕР ± (АВС), и пусть PM ± DB, тогда ЕМ 1 BD. 4) S.BDE — искомое сечение, где BD — основание, ЕМ— высота. 5) Так как CN1 BD, то PM = CN/2; CN = 2а/ 4$ , а РМ = а/45 . 6) Так как SO = , 500 - ~,то ЕР = V 4 2 7) Из SEMP находим ЕМ = а2 500 5а2 7ГоО2-9д2 5 + 4 ~ 16 ~ 4?5 I—5---г 0 < а < 20 8) Искомая площадь Q = а-уЮО - 9а /8, где ( , [а < 100 / 3. 9) Для отыскания критических точек функции Q(a) найдем Г1,, . 1002—18а2 производную и приравняем ее нулю. Q (а) = —=== 8д/1002-9а2 Q'(a) = 0 при 1002 - 18а2 = 0, то есть при а = 5041 /?> Таким образом, в промежутке (0, 50 41 /3) Q’(a) > 0 и функция Q(a) возрастает. Так как (0, 20) е (0, 50 42 /3), то и в промежут- ке (0, 20) Q(a) возрастает и достигает наибольшего значения при а = 20, то есть QMax = Q(20) = ^VlOO2 - 9 -202 = = 200. О о Ответ: при а = 20 QMax = 200 (кв. ед.). 72
4. Дополнительные задачи для тренировки Г Для подбора задач этого раздела использованы школьные учебники и сборник конкурсных задач по математике под ре- дакцией М.И.Сканави. Большинство задач относится к стерео- метрии. Но решение их обязательно требует знаний по плани- метрии. Порядок задач соответствует расположению основного теоретического материала, используемого при решении. Так, например, для решения задачи №39 (8, 9), нужны знания о свойствах сферы и конуса, а-также свойства, выражающие от- ношения между ними и связь их с другими фигурами. В спра- вочном материале (Приложение №3) по всем трем разделам можно ознакомиться с теоретическим материалом, нужным для решения задачи. «Адресом» этого -материала является наличие цифр 8 и 9 в любых сочетаниях с другими цифрами. Задача 1. (3, 8) Какому условию должны удовлетворять ра- диусы трех шаров, попарно касающихся друг друга, чтобы к ним можно было провести общую касательную плоскость? Ответ: л > г, т.е. n > —=A^=-v. Задача 2. (5, 7) Площади боковых граней наклонной тре- угольной призмы, пропорциональны числам 20, 37, 51. Боко- вое ребро равно 0,5 дм, а площадь боковой поверхности рав- на 10,8 дм2. Найдите объем призмы. Ответ: Упр = 6,12 дм3. Задача 3. (6, 7) В равнобедренном треугольнике АВС угол при вершине С равен 20°. Из вершины А и В проведены лучи, составляющие с основанием углы 50° и 60° и пересекающие противоположные стороны в точках Z) и Е соответственно. Най- ти углы треугольника ODE, если О — точка пересечения отрез- ков AD и BE. Ответ: Z.OED = 30°, Z.ODE = 80°, Z.DOE = 70°. Задача 4. (6, 7) Стороны основания прямого параллелепи- педа равны 7 см и 3 V2 см, а острый угол основания равен 45°. Меньшая диагональ параллелепипеда составляет угол в 45° с плоскостью основания. Найдите объем параллелепипеда. Ответ: Упр = 215 = 105 (см3). 4—1224 73
Задача 5. (6, 7) Диагональ боковой грани правильной тре- угольной призмы равна d и составляет угол <р с плоскостью другой боковой грани. Найдите объем призмы. Ответ: Кпр = у J3sin2<p-^3 - 4sin2<p ед3. Задача 6. (6, 7) Диагональ прямоугольного параллелепипеда составляет угол а с плоскостью боковой грани и угол Р с плос- костью основания. Найти объем параллелепипеда, если его вы- сота Н. Задача 7. (6, 7) Двугранный угол при основании правильной треугольной пирамиды равен а, боковая поверхность пирамиды равна S. Найти расстояние от центра основания до середины апофемы пирамиды. Ответ: ОМ = лин. ед. бу cosa Задача 8. (6, 7) Основанием пирамиды служит равнобедрен- ный треугольник, у которого боковая сторона равна а, а угол при вершине равен а. Все боковые ребра наклонены к плоско- сти основания под углом р. Найти объем пирамиды. a sin-tgP Ответ: Кпир = ---— ед3. О Задача 9. (6, 7) Высота правильной треугольной пирамиды равна Н, двугранный угол при основании равен а. Найти пол- ную поверхность пирамиды. Ответ:\„„=^^едЗ. 2sin2^ 2 Задача 10. (6, 7) В правильной треугольной пирамиде пло- ский угол при вершине равен <р, а сторона основания равна а. Найдите объем пирамиды. Ответ: Упир = ^3ctg2 | - 1 ед3. 74
Задача 11. (6, 7) Высота правильной треугольной пирамиды равна h, а двугранный угол, ребром которого является боковое ребро пирамиды, равен 2<р. Найдите объем пирамиды. Ответ: Кпир = В3(3tg2<p - 1) ед3. Задача 12. (6, 7) Основанием пирамиды является ромб со стороной а. Две боковые грани пирамиды перпендикулярны к плоскости основания и образуют тупой двугранный угол <р. Две другие боковые грани составляют с плоскостью основания дву- гранные углы а. Найдите объем пирамиды. Ответ: VnMp = |e3sin2<p tga ед3. Задача 13. (6, 7) Основанием пирамиды является ромб со стороной а и острым углом <р. В пирамиду вписан конус, обра- зующая которого составляет с плоскостью основания угол а. Найдите объем конуса. Ответ: ел’. Задача 14. (6, 9) Через вершину конуса проведена плоскость, делящая окружность основания в отношении р : q . Эта плоскость отстоит от центра основания конуса на расстоянии d и составляет с высотой конуса угол, равный а. Найти объем конуса. 3cos a • sin 2a cos2 _3£_ Р+Я Задача 15. (6, 9) Найти угол при вершине осевого сечения конуса, если центральный угол в развертке его боковой поверх- ности равен Ор радианам. (X Ответ: <р = 2arcsin--. т 2л Задача 16. (7, 7) Площади трех попарно смежных граней прямоугольного параллелепипеда равны Si, S2 и S3. Выразите объем этого параллелепипеда через <Ji, S2, S3 и вычислите его при 51 = 6дм2, S2 = 12дм2, S3 = 18 дм2. Ответ. 1*пар ^S^S^Sy ДМ3, Fjjap 36 ДМ3. 4* 75
Задача 17. В прямоугольном параллелепипеде диагонали трех граней, выходящие из одной _вершины, равны 7 см, 8 см и 9 см. Найдите объем параллелепипеда. Ответ: И=48-УГГсм3. Задача 18. (7, 7) Боковое ребро прямоугольного параллеле- пипеда равно а. Сечение, проведенное через две стороны раз- ных оснований, является квадратом с площадью Q. Найдите объем параллелепипеда. Ответ: Ипар = abc = a^Q <]Q-a2 (ед3). Задача 19. (7, 7) Правильная четырехугольная пирамида, у которой сторона основания равна а, а плоский угол при верши- не равен а, вращается вокруг прямой, проходящей через верши- ну параллельно стороне основания. Найдите объем полученного тела вращения. Ответ: И= (2ctg2 + 1) ед3. Задача 20. (7, 8) Выразить диагонали вписанного в окруж- ность четырехугольника через его стороны. Ответ: /. = V bc+ad . (c2+d2)ab+(a2+b2)dc 2 у ab+dc Задача 21. (7, 8) Используя результат задачи 20 докажите тео- рему Птолемея: произведение диагоналей вписанного в окружность четырехугольника равно сумме произведений его сторон. Задача 22. (5, 7) Выведите формулу Герона1 для площади треугольника: 5 = у/р(р - а)(р - в)(р - с) , где а, Ь, с — длины сторон тре- а + Ь+с угольника, а р = —-— — полупериметр. 1 Герои Александрийский — древнегреческий ученый, живший в I в.н.э. 76
Задача 23. (7, 8) Доказать, что площадь четырехугольника, вписанного в окружность, может быть вычислена по формуле S = у](р - а)(р - b)(p - с)(р - d), где р — полупериметр, a, b, с, d — стороны четырехугольника. Используя эту формулу, вывести формулу Герона. Задача 24. (7, 8) Доказать, что площадь треугольника, вписанного в окружность, может быть вычислена по форму- ле S д = , где а, Ь, с — стороны треугольника, R — радиус описанной окружности. Следствием задачи №24 является утверждение: Задача 25. Радиус описанного около треугольника круга вы- числяется по формуле: R = —, а^с- . 4J р(р-а)(р-Ь)(р-с) Задача 26. (7, 8) Показать, что радиус описанного около че- тырехугольника круга вычисляется по формуле © — 1 1(ас + bd)(ab+cd)(ad+be) 41l(p-a)(p-b)(p-c)(p-d) ’ Задача 27. (7, 8) Показать, что радиус г вписанной в тре- 2S угольник окружности выражается формулой г = -------. а + Ь+с Задача 28. (5, 7) Доказать, что в треугольнике АВС биссек- триса АА\ вычисляется по формуле „ А 2OCCOS— АА\ = —-----—, где b = АС, с = АВ. Ь+с Задача 29. (7, 8) Если в треугольнике АВС вписана окруж- ность, то расстояние от вершин А, В и С до точек касания рав- ны соответственно р — а, р — Ьи р — с. Доказать. Задача 30. (7, 8) На плоскости лежат четыре шара радиуса R, причем три из них попарно касаются друг друга, а четвертый касается двух из них. На эти шары положены сверху два шара меньшего радиуса г, касающиеся друг друга, причем каждый из них касается трех больших шаров. Найдите радиус малень- ких шаров. Ответ: г = . . 3 77
Задача 31. (7, 8) В пирамиду, основанием которой явдяется ромб со стороной а и углом а, вписан шар. Найдите объем ша- ра, если каждая боковая грань пирамиды составляет с основани- ем угол <р. на3 sir? atg3^ Ответ: ИПиР= ----------- (ед3). Задача 32. (7, 8) Найдите множество всех точек, из которых можно провести к данной сфере три попарно перпендикуляр- ные касательные прямые. Задача 33. (10, 7) Диагонали осевого сечения цилиндра пе- ресекаются под углом, равным а, обращенным к основанию. Объем цилиндра равен V Найти высоту цилиндра. 34Kctg2^ Ответ: Н= \-------- лин. ед. I л Задача 34. (10, 7) Плоскость, проведенная через образую- щую цилиндра, составляет с плоскостью осевого сечения, со- держащую ту же образующую, острый угол, равный а. Диаго- наль прямоугольника, полученного в сечении цилиндра этой плоскостью, равна / и образует угол, равный Р, с плоскостью основания. Найти объем цилиндра. Ответ: V= 8cos2 а Задача 35. (9, 8) В конус вписан полушар: большой круг по- лушара лежит в плоскости основания конуса. Найти объем по- лушара, если образующая конуса равна / и составляет с плоско- стью основания угол, равный а. Ответ: V- . 12 Задача 36. (8, 9) Докажите, что площадь сферы равна площади полной поверхности конуса, высота которого рав- на диаметру сферы, а диаметр основания равен образующей конуса. Ответ: бонуса = 4лЛ2 (площадь полной поверхности конуса). 78
Задача 37. (8, 9) В конус вписан шар. Докажите, что от- ношение объемов конуса и шара равно отношению площа- дей полной поверхности конуса и сферы, являющейся гра- ницей шара RH=r(l+ R). Задача 38. (8, 9) В шар вписан конус, радиус основания ко- торого равен г, а высота равна Н. Найдите площадь поверхности и объем шара. Ответ: 5^- ед- ед». Н ЬН Задача 39. (8, 9) В конус, радиус основания которого ра- вен R, а образующая равна /, вписана сфера. Найдите длицу линии по которой сфера касается боковой поверхности ко- нуса. Ответ: L - дин. ед. Задача 40. (8, 10) В шар вписан цилиндр, в котором угол между диагоналями осевого сечения равен а. Образующая ци- линдра равна /. Найдите объем шара. Ответ. Рщара ~ еД3- 6cos3£ 2 Задача 41. (8, 10) Плоскость, параллельная оси цилиндра, делит окружность основания в отношении т : п. Площадь сече- ния равна 5. Найти боковую поверхность цилиндра. Ответ: 5бок = —ед2. пт sm----------------- т+п Задача 42. (9, 10) Разность между образующей конуса и его высотой равна d, а угол между ними равен а. Найти объем ко- нуса. л . з а nd ctg — ctga Ответ: К= --------2---- ед3. 79
КРАТКИЕ УКАЗАНИЯ 1. См. рис. 54. Пусть г{> Г2> г^. Про- ведем к первым 2-м шарам касательную плоскость. Кроме того, через центры трех шаров проведем плос- кость, перпендикулярную к касательной плоскости и рассмотрим окружность ра- рис 54 диуса г, касающуюся двух получившихся в сечении больших кругов и их общей касательной прямой. Третий шар может, очевидно, касаться первых двух шаров и касающейся их плоскости, если он не очень мал, а именно 75 > г. Имеем: CQ = АВ = АО + ОВ. С&2 найдем из Д(?1 СО^, АО найдем из ДС4О. ОВ найдем из &О}ВО J(n + /2) - (б - Гг)2 = V(ri + г> " <Г1 ~ r)2 + V(r2 + О2 - (г2 - г)2 74г(Г2 = + 7^г2г 5 Vrlr2 = ФФ + ФФ ’ Ф"(ф\ + - Фл '> - _ rlr2 (Ф\+Фг)2 Согласно условию зада- чи, площади боковых граней можно записать: 51 = 20х, Д', = 37х, 53 = 51х •%ок= *^1 +*$2 +*% = = 20х + 37х + 51х = 108х Далее, 108х = 10,8; х = 0,1. 51 = 2дм2, S2 = 3,7дм2, 53 = 5,1 дм2 Искомый объем приз- мы равен произведению площади перпендикуляр- ного сечения призмы на боковое ребро. Обозначим высоты боковых граней со- ответственно hi, hi, h$. 80
Тогда Л1 - £ = A = 4; A;= g = -g _ 7,4; „3. g = 10,2. Найдем площадь перпендикулярного сечения 5 = Jptp-AjXP MP-M = 12,24 Гпр =12,24 -0,5 = 6,12 дм3. 4. См. рис. 56. ^пар = *$осн 'Н. *$'осн нахо- дим, используя формулу площади треугольника по двум сторонам и углу между ними 5осн = 7-3-J2 sin45°= г- -1Л^=21. Теперь найдем Л С по тео- реме косинусов. Из ДАОС АС2 = AD2 + + СО2 - 2AD COcos45° = = 49+18 -2-7-3-j2^y = 25; АС = 5. Так как ZCACj = 45°, то ДЛСС1 — равнобедренный Рис. 66 и СС1 = АС = 5. 6. См. рис. 57. Так как параллелепипед прямоугольный, то ВС{, BD — проекции диагонали BDi на соответствующие грани. По условию DDi = Н, Z.C\BD = a, Z.DBD\ = р. Из ДРООр BD\ = Рис. 57 Н . sinp ’ BD = Н ctgp. Тогда, из ДВСОр QA = POpsina Но так как CD = CjPj, то из &.BCD : ВС = JBD2 - CD2 ; ихт zr2 2а Я2 sin a Н I ~ ВС = Jff ctgzB------—— = -г— -Jcos В - sin2 a V sinp smp’ K V= BC- CD H=^- Jcos2 p - sin2 a • Я si”a • H. sinp ’ r smp 81
Рис. 58 7. См. рис. 58. Обозначим сторону осно- вания а, апофему — Ь. •5бок=^; 3ab = 2S. Так как OD = , то из AKOD: 6 ауЗ , —— = ocosa. 3 3ab = 2S Из этой системы уравнений исключим а. Разделим почленно первое уравнение. на второе. gz^cosa = уД; J-lM.TaK 73 6ocosa V9cosa 3Vcosa как AKOD — прямоугольный и M — середина гипотенузы KD, то | ОМ = b : 2 как радиус описанной вокруг \KOD окружности. | 1 I 5^3 Ответ: ОМ = -,1 . 6 V cosa 8. См. рис. 59. Так как все боковые ребра пи- рамиды наклонены под одним уг- лом, то вершина пирамиды проек- тируется в центр О описанного около треугольника круга. Этот центр лежит в точке пересечения серединных перпендикуляров тре- угольника. 5 основания находим по формуле: 5^ = oMna. Чтобы найти Н, предварительно найдем ОС из прямоугольного треуголь- ника ODC, опустив перпендикуляр из точки О на сторону АС. ОС = —-—. Теперь из Д.КОС: 2cos— 2 82
з . a R H= OCtgp = Тогда Гпир = | I a2sina-^^-=------------Д— . Э___« 3 2 “ O 9. См. рис. 60. *Skbih = *%ок + ^осн- Пусть апофема — b, сторона основания — a. м><ложж. с - n2V3 с - За к- с - Заб о27з Имеем. ОосН - , обок у ^полн ~2~ + 4 Из &KOD: b= OD = Я-ctga. Но OD = , следова- sin a 6 яд/З тт х 6/7ctga тельно —— = Я-ctga; а = —. 6 V3 с 3 6#ctga Н 43 36#2ctg2a а + z 1 , _ 2 ' ~7Г' + Т------3 3^Н С,8“(^ + Ct8“’ “ _ зУзЯ^сгеаС! + cosa) _ cosa 2cos _ з7зяг cosa sin a sin2 a 2sin2« 2 10. См. рис. 61. ^пир = *5осн’Н = |2~ ’ Н . Задача сводится к нахождению высоты. Чтобы найти высоту Н, рассмотрим два прямоугольных треугольника: KOD и KDA и выразим из них b через Ни а. 83
Из &КОА. Ь = у • ctg |. Приравниваем правые части урав- нений: Jh2 + уу =у ctgy . Находим Я = у Jctg2 у -1 V 1 • /cte2 -1 - — /зсй>2 - 1 (ел3) Ипи₽ 12 2 V 8 2 3 ~ 24} ё 2 3 Д Задача сводится к оп- ределению а. Для опреде- ления а рассмотрим &КСМ и запишем его площадь двояко: CM Н = CK DM(X) и СМ= 2 Из ЬКОС. СК= ^Я2 + у-. Из ЛАВМ: DM= ^ctgcp. Подставим полученные выражения в (1): Разделим обе части уравнения на а 2 Я>/3=^Я2 +у ctgcp. Возведем обе части уравнения в квадрат: ЗЯ2 = (Я2 + ^-) ctgcp => а = - С<?2(Р • Находим объем. 84
12. См. рис. 63. ^пир— J *^осн' Н. Sow— fl2sin<p. Из \СМВ. h = o sin(180°— <p) = asintp. Из &KCM‘. Н = й-tga = a sincptga Ипир=| o2sin<ptgaasin<p = | a3sin2 <p tga. 13. См. рис. 64. Ипир=|лА2-Н. Из точки С на сторону АВ опустим перпен- дикуляр. Очевидно ОМ = CN: 2. CN найдем из &.CNB : CN = a-sirup и ОМ = . Я найдем из&КОМ: Я = ОЛ/tga Я= g ^Ф^ 2 v _1 fl2-sin2<р fl sin9 tga__ ПИР з" 4 2------ э . -1 _ яд sin <p tga 24 14. См. рис. 65. V= ±nR2H. Н найдем из &.КМО'. Н = —— . sina Рис. 65 85
Так как , то Z.COD = = ф. u Я p+q CBD 4 Из &OMN: ON = —. cosa Из &CNO: 0С= R= J±l_ =------------. Ф пр cos— cos a cos—— 2 p + q v = £ d2 d_ _2nd3 3 cos2 a cos2 s*na 3cosasin2a cos2— p+q p+q 15. См. рис. 66. Пусть <p — угол при вершине К. г — радиус основания конуса. Развертка — сектор круга радиуса R. (R = t) Длина дуги сек- тора равна произведению радиуса на угол, выраженный в ра- дианах, то есть 2лг = /ар. Разделим обе части уравнения на 2л/. Получим у = Ор 2л ' Из прямоугольного треугольника КОЛ: Г ф . ф Ир ф . ар ~ ар - = sin -х, sin^- = ; тг = arcsin —; ф = 2 arcsm -z—. 1 2 2л 2л 2 2л 2л 16. См. рис. 67. Обозначим стороны параллелепипеда через а, в, с. Тогда Kiap = obc; Si = ab, Sj = ас, Sj = be. Перемножим почленно эти равенства: S^S^a^c2. abc = ^S^S^S^. . Рис. 66 86
17. См. рис. 68. Обозначим стороны парал- лелепипеда а, Ь, с. ЙПар = abc Тогда а2 + б2 = 49 о2 + с2 = 64 62+ с2= 81 Сложим почленно равенства: 2д2 + 2^2 + 2с2 = 49 + 64 + 81 или в2 + & + с2 = 97 Используя предыдущие ра- венства, получим: 49 + с2 = 97 или с2 = 48 64 + А2 = 97 или е2 = 33 81 + а2 = 97 или в2 = 16. Перемножая почленно равенства, dl tP- c1 = 16-33-48 abc = 716 -33 18 = 48 TH . 18. Введем обозначения сторон основания параллелепи- педа: бис Рпар ~ abc. Так как сечение является квадратом, то АВ\=с, Q = с2, с Из ДАВВр b = 4с2 - а1 = ^Q-a2 *пар = abc = a^Q ^Q-a2 (ед3). 19. См. рис. 69. Через ось / и центр основания пирамиды проведем плоскость Р. Она образует в сечении пирамиды некоторый SSMN. Повернем SSAB около оси / так, чтобы он лег в плоскость Р. Так как АВ = MN, а высота SO меньше вы- соты боковой грани SK, то треугольники расположатся так, как на чертеже. Рис. 69 87
Любое другое сечение SEF пирамиды попадет внутрь пяти- угольника SMABN, а все сечения дважды покроют этот пяти- угольник. Остается определить объем тела, полученного от вра- щения этого пятиугольника SMABN вокруг оси I. Половину иско- мого объема можно получить в виде разности объемов цилиндра, полученного от вращения прямоугольника SKBL и конуса, полу- ченного от вращения &SNL. 20. Указание. Каждая из диагоналей легко выражается через две стороны и угол между ними по теореме косинусов: = Ь2 + с2 — 2Z>c cosZ С или = а2 + d2 — 2adcosZA По свойству углов вписанного четырехугольника ЛА = 180° —ZC. A cos(180° - ZQ= -cosZC. Тогда F2 + с2 — 2bc-cosZC = а2 + d2 — 2edcosZC, откуда 2cosC = - • 2(bc+ad) 2 Выполнив алгебраические преобразования выражения /, , приходим к ответу. Аналогично выражаем вторую диагональ че- рез стороны а, Ь, с, d. Рис. 70 21. Используя результат задачи 20 докажите теорему Птолемея: произведение диа- гоналей вписанного в окруж- ность четырехугольника равно сумме произведений его сто- рон. 22. См. рис. 70. Известно, что 1 ab sin( а, b). По теореме косинусов с2 = а2 + № — 2ab-cosZC. Отсюда cosC = а ** .~С , а sin2C = 1 — cos2C = (1 — cosQ(l + cosQ. Подставляя значения cos С = ———, находим 2ab sin2C = —5-7 (a + b + c)(b + c — a)(a + c — b)(a + b - c). 4a b Ho a + b + c = 2p, b + c — a = 2(p — a), a + c — b = 2(p — b), 88
a + b- с = 2(p - с). Тогда sinC = Jp(p - a)(p - b)(p - c) , a площадь 5= ~ab sinC= Jptp - a)(p - e)(p - c) 23. Указание. Рассмотрите площадь четырехугольника как сумму площадей двух треугольников, образованных одной диагональю и парой смежных сторон. Площади треугольников найдите двумя способами: а) по формуле 5Д= у ab sin( а, b) и 5 = Jp(p - а)(р - в)(р - с). Укажите более рациональный из них. 24. Указание. Проведем из какой-либо вершины треуголь- ника, например из А, диаметр AM. Тогда Z.AMB = ZACB, как углы, опирающиеся на одну и ту же хорду, sin С = sirUMS . 2 л Площадь треугольника АВС может быть вычислена по формуле S = 4- ab sinC. Заменяя sinC его значением, получим: S = . 2 4Л 26. Указание. В формулу, выражающую радиус описан- ного круга около одного из треугольников, отсекаемых диагона- лью (следствие задачи №24) подставить значение этой диагона- ли, выраженной через стороны (задача №20). 27. Указание. Обратите внимание на то, что радиусы, проведенные в точки касания, являются высотами треугольни- ков с вершинами в точке О. Представив площадь данного тре- угольника АВС в виде суммы площадей треугольников АОС, СОВ и ВОА, получим искомую формулу. (Рис. 70). 28. См. рис. 71. Искомый отрезок АА\ можно выра- зить через данные отрезки и углы, если использовать одну из формул площади треугольника: Площадь треугольника АВС равна сумме площадей треуголь- ников АВА\ и АА\С. 4 Ac sin А = 4с • А41 sin 4 + х АА, • Asin4 2 2 1 2 2 1 2 После элементарных преобразова- ний получим: 5—1224 89
A tasind = A4tsin — (c + b) / bcsmA Откуда ЛЛ1=? --------j (Z> + c) sin у be 2 sin— • cos4“ 2bc cos^ 2 2 =1 (6 + c)siny b+c' 29. Введем обозначения: AF=AE^l\ CE = CD = n\ BF=BD=m AB = c, AC — b, BC — a Тогда: m + n = a, 1+ m = c, 1+ n = b. Периметр ДЛБС a + b + c = 2(1 + m + n), p = I + m + n. Откуда I = p - (m + n) = p — a, m = p — (I + n) = p — I, n = p — (I + m) = p — c. 30. Пусть А, В, C, D — цен- тры больших шаров. Рассмот- рим проекцию всех шаров на плоскость &BCD. Так как боль- шие шары одинакового радиу- са, равного R, то треугольники АВС и BCD равносторонние со стороной 2R. (См. Рис. 72). Так как центры малых шаров равноудалены от центров соответствующих больших шаров (R + г), они спроектируют- ся в центы тяжести Oi и ф равносторонних треугольников АВС и BCD. Так. как радиусы малых шаров по условию задачи равны, то отрезок, соединяющий их центры, параллелен рас- сматриваемой плоскости и делится точкой касания Щаров пополам. Поэтому проекция точки касания' окажется на от- резке ВС. Отсюда следует, что малые шары спроектируются в круги, вписанные в треугольники АВС и BCD. Следовательно = АВ^ = 2R^ = Г б 6 3 ’ 90
31» См. рис. 73. Центр шара лежит на вы- соте пирамиды и шар касает- ся боковых граней в точках, лежащих на высотах боко- вых граней. Так как в осно- вании ромб, то его площадь ^CH = ,o2sina. 1с -с _ a2sina’. ~ Оосн ^ДАОВ “ , с. - SbACB _ °2 sina гр’, cos<p 4cos<p *$>.гр.= ~2 аЬ- „ a2 sina 1 . _ asma Приравняем: -----= -ав , о — -----. 4cos<p 2 2oos<p Из ЛКОМ: ОМ = ^cos<p = cos<p = Из ЬОХОМ:'г = OAftg| = . ла3 sin3 atg3^ V '= 2 ш. . . 6 . " 32. См. jpHc. 74. Пусть M — одна из множества то- чек, удовлетворяющих условию задачи. Тогда отрезки МА, МВ, МС, будучи отрезками касательных, проведенных к сфере из одной точки, равны между собой. Поэтому прямоугольные тре- угольники АМВ, AMQ, ВМС — равны. Следовательно, А4ДС — равносто- ронний. Очевидно, что отрезок ОМ пересечет его в центре тяжести О\. Пусть AM = а, тогда АВ = a-Jl и 5* 91
А0\ = -у-. Запишем площадь &ОАМ двояко и приравняем: ом аох = оа ам. ом.^- = ОМ = = ^Я. ' 3 Тб 6 2 Таким образом М лежит на сфере радиуса ОМ с центром в О. Вращая данную сферу вместе с касательными МА, МВ, МС вокруг центра О, мы получим геометрическое место точек, удовлетворяющих условию. Рис. 76 33. См. рис. 75. Из условия задачи V= nR2H (1) ИзДСЛЛ: Н = 2/?ctgy (2) Из(1):Л=^. Подставим во (2): 2 V Пп 2 КП н= i 4Kctg2y л 34. См. рис. 76. Ицил = ML ИзДЛр4Б: Я=&тр, АВ = Zcosp. Из прямоугольного ДЛВС: 2/J = _ Zcosp cosa cosa /{= ^COSP 2 cos a г/ _ nZ2cos2p j . Q Кхил --------2--’ ’ Sin P 4 cos a _ n/3cospsin2p 8 cos2 a 92
35. См. рис. Тк Рассмотрим осевое сечение конуса и полушара. >пол=|^3 * 5- Из ЬКОА найдем радиус основания конуса R: R= /cosa Из &ADO найдем радиус полушара: r= R sina = / cosa sina = _ I sin 2a = —-—. Тогда 2 ft sin3 2a я/3 sin3 2a v= 3’ — 8~ = —12----------” 36. См. рис. 78. Рассмотрим осевое сечение шара и конуса. ^ПОЛН.КОН. = ~ = пг-2г + п/2 = Злг2. Из Л.КОА выразим г через Л 4г2 - г2 = 4/?2, Зг2 = 4Я2 ^ПОЛН.КОН. ~ 4л7?2. 37. См. рис. 79. Рассмотрим осевое сечение конуса и шара. v _ лЯ2Я 'к Иш = I’"'3 И, п.к. 3 г K _ r2h Ии 4r3 5n.K = nRl+ л/?2 = nR(l + R); 5Ш = 4л/*2 5п.к. _ nR(l+R) _ /?(!+/?) *7Ш 4лг2 4r2 u R2H R(l+R) Нам надо: —5- = v . 4r3 4r2 Рис. 78 98
к Рис. 80 Из подобных треугольников КОХА и КМО => ~ L; RH- Rr = 1г, RH = r(l + R) ч. т. д. 38. См. рис. 80. *Suapa=4^7?2; Kuapa= • КзЬКОВ- /=7я2 +г2 , bKOtC-MOB : R : КС= I: Н R : = 7я2+г2: Н ; R = Н2+г2 2Н с = 4л(Я2+г2)2 _ л(Я2 + г2)2 шаРа ~~~~4Н2 ед2 , _4л(Я2 + г2)3 _п(Н2 + г2)2 з шара 38Я3 " 6Я3 еД 39. См. рис. 81. Рассмотрим осевое сечение конуса и сферы. Пусть L — длина линии, по кото- рой сфера касается боковой поверхности конуса — это окружность радиуса г. То- гда L = 2кг. Сначала из &.КСВ найдем высоту конуса Н: Н- 712 - R2 Далее рассмотрим два подобных треугольника: КСВ и K0N и определим радиус шара г: H-r _ 1 _ Hr _ W/2-A2 Н ~T~~R ,r I ~ (l + R)l ’ _ W/2-/J2 7/2-Л2 _ R(l2-R2) = R(R-R) Г (l+R)l (l+R)l I j = 2kR(1-R) 94
40. См. рис. 82. 1/ = ^ _ пЗ кшара 'Jкл • Из ДАВС: 2R = V ~ 4nl3 гшара ъ п 3 • 8 • cos3 — 2 —l—; R = а COSy л/3 з - =------ед< г з« 6 cos — 2 / 2cos^ 2 Рис. 82 41. См. рис. 83. •$бок.ц = 2nRH. По условию и АСВ т j.— —, то есть угол (р = ADB П 2 л т т + п ’ Из ДОШ: AM = Asin——- т + п АВ=2АМ = 2Rsm~^~ т + п S = HAB=H2Rstn — т + п 2RH = ..... ; ^бок.ц= KS ед2. . пт ^х,к-ц пт sin-- sin--- т+п т+п 42. См. рис. 84. Рассмотрим осевое сечение iz 1 о и nR2H конуса: Кк = - Н = -у-. Найдем /? и Н. По условию / — Н= d (1) Из прямоугольного ЛАСК имеем: Н = /cosa ' j] _ dcosa 1-cosa Из полученной системы ис- ключим /. ИзДЯС/Г: /?=/ftga; Рис. 83 95
д _ б/cosatga _ d sin a 1-cosa 1-cosa’ Подставим H и R в формулу объема: у = nd* 2 sin2 a dcosa _ nd3 * * sin2 acosa _ nd3 sin3 actga 3(1-cos a)2 (1-cosa) 3(2sin2^)3 24 sin 6 01 2 2 rcJ3(2sin^cos^)3ctga n</3ctg3^ctga 24 sin 6 3 2 При самостоятельном решении дополнительных и, особенно, контрольных задач вы обращались к задачам-спутникам, а иногда составляли их сами, главным образом с целью получения помощи. Они, как правило, были или частью основной задачи, или частным случаем более общей ситуации. Исподволь для развития геометри- ческой интуиции, геометрического видения мы включали некото- рые задачи, наводящие на мысль обобщающего характера. Так в разделе «Подумаем вместе» в задаче №14 (стр. 36) о равнобедрен- ной трапеции, описанной около круга, мы сделали первую попыт- ку доказать высказанное утверждение для более общего случая многоугольника того же вида. В контрольной задаче №41 (стр. 55) с помощью подбора спутников мы намекнули на аналогию зако- номерностей в планиметрии и стереометрии пока еще на конкрет- ных объектах шара, вписанного в правильную четырехугольную пирамиду. В дополнительной задаче №37 (стр. 81) предлагается доказать то же утверждение для комбинации шара и конуса. Мы предлагаем продолжить тренировку не только в реше- нии задач, но и в составлении своих задач-спутников, цель ко- торых — пока выдвигать гипотезы, делать небольшие открытия, подтверждать их справедливость на как можно большем числе примеров, а иногда и использовать как новый способ решения, сравнив результат с другим решением или хотя бы с ответом. С этой целью мы вернемся к задаче №14 (стр. 36). Ее условие полностью заимствовано из «Сборника конкурсных задач» под ред. М.И. Сканави. Там эта задача дана в группе А под № 10. 183. Но ведь мы в задаче №14 нашего пособия уже убедились в справедливости равенства отношений длин и площадей для лю- бой описанной около круга трапеции. Не продолжить ли нам процесс обобщения полученного утверждения для четырех- угольников, не являющихся трапециями? Начнем с самого «совершенного» четырехугольника — квадрата. 96
Задача 14а. Поставим задачу: сравнить отношения площадей круга и описанного около него квадрата и длин окружности и пе- риметра квадрата (Рис. 85). Пусть сторона квадрата а, а радиус круга ОМ = г, площадь круга 5, площадь квадрата — Q, длина окружности С, длина периметра квадрата Р4. Выразим сторону квадрата через радиус вписанного круга: а = 2г. Тогда Р4= 2г- 4 = 8г. Составим ука- занные отношения: S _ яг2 _ лг2 _ я С _ 2яг _ я Q~ a1 ~(2r)2~~*' Сравнивая отношения, замечаем, что имеет место их равен- ство, как й у комбинации круга с произвольной трапецией, а не только с равнобедренной. Но квадрат — это ромб с прямым углом. Поставим такую же задачу для произвольного описанного ромба. Задача 146. В ромб с произвольным углом а вписан круг Сравнить отношение площади круга S к, площади ромба Q с от- ношением длины окружности С к периметру ромба Р (Рис. 86). Площадь круга 5 = ту2. Длина окружности С = 2лг. А площадь ромба можно находить, различными способами: как параллелограмма с основанием а и высотой MN = 2г, как сумму площадей треугольников различных разбиений (&ABD и ABDC, A4CD и Д4ВС, Д4В0, АЛОД &.ВОС и &DOC или bAOD, &DOC, \СОВ и &АОВ). Во всех случаях это равные треугольники, поэтому достаточно выразить площадь одного из них и умножить полученное значение на число треугольников. Выберем Последнее разбиение. Оно характерно тем, что в каждом треугольнике основанием является сторона ромба, а высотой радиус окружности, вписанной в ромб. Тогда Q = пл. &.АОВ + пл. &ВОС + пл. &.COD + пл. &AOD, т.е. Q = ±AB r + ^BC r + ^CD r + ^AD г = у(АВ+BC+CD+DA) = r^. И, следовательно, S _ пг22 _ 2пг ~Q~~r Р~’ а это и есть отношение длины 97
окружности к периметру описанного ромба. | Выбранное нами разбиение на треугольники подсказывает 1 следующее обобщение. Сформулируем задачу для произвольного j выпуклого четырехугольника. 1 Задача 14в. Около окружности радиуса г описан произвольный выпуклый четырехугольник ABCD. Доказать (или опровергнуть), что отношение площади круга к площади четырехугольника равно от- ношению длины окружности к периметру этого четырехугольника. ?з -д q Обозначим точки касания сторон ок- ружности через Т1; Т^, Т$, Т4 (Рис. 87). 1 /1 Тогда О7\ = ОТ2 = OTj = ОТ4 = г и —А ^2 площадь Q описанного четырехугольни- ка легко представить как сумму площа- в дей следующих треугольников: АОВ, 4 ВОС, COD и DOA, имеющих одинаковые 7J высоты, равные радиусу вписанной ок- А ружности т. е. Рис. 87 Q = ^r(AB + BC + CD + DA) = ^rP. Сравнив отношения площадей и длин, получим: S _ пг22 _ 2пг _ С Q~~P~r , Таким образом, высказанная гипотеза о равенстве указан- | ных отношений оказалась справедливой для любого описанного « около круга четырехугольника. Возникает новая задача обобщения: испытать на равенство эти же отношения у любого описанного многоугольника и круга. Но задача для любого л-угольника — это уже теорема, тре- бующая строго доказательства, основанного на определенной системе аксиом. Наша задача скромнее: проверить справедли- вость полученного утверждения для других частных видов - многоугольников, не являющихся четырехугольниками. Для начала рассмотрим некоторые правильные описанные много- угольники (для простоты выкладок вследствие непосредствен- ной связи их с окружностью), а затем — произвольные (для большей уверенности в общности). Задача 14г. В равносторонний треугольник АВС вписан круг. Доказать, что отношение площади круга S к площади треугольника Q равно отношению длины окружности С к периметру треугольника Р. 98
Пусть сторона Д4ВС АВ = ВС = = СА = а, радиус круга — г (Рис. 88). Тогда S - nr2, Q = i ah, где CH=h = Ja2-^-=^, V 4 2 С = 2nr, Р = За. Составим заданные в условии отношения и сравним их S _ лг^2 _ 2пг2 . С _ 2лг Q а^За a2-j3 ’ Р За ' Для сравнения отношений выразим радиус вписанной окруж- ности через сторону треугольника а. В правильном треугольнике ОН = г составляет 4 высоты СН, т. о. г = 4 • Исключая г. 3 3 2 6 . о За2 >па2 л 1 . aa-j3 a2j3 S яа24 получим: S = л-т- = ——. Q = -ah = „ = ——, = —г-? 36 12 2 2-2 4 Q 12а2 _ п _ лл/З . si _ 2 _ 2па4з _ ла-УЗ , р _ - . С _ ita-j3 _ n-j3 ~ зТз ” 9 ’ ПГ " ~~6 3 ’ ’ 7~ 3 3а ~ 9 А можно, используя разбиение ЛАВС на треугольники АОВ, ВОС и СОА, представить площадь С = 4 г(АВ + ВС + СА) = 4 гР. — S nr22 2nr С — е Тогда — = - - =-^- = _. Таким образом, для равносторон- него треугольника равенство S: С = С: Р справедливо. Но мы знаем, что для любого треугольника существует и единственная окружность, вписанная в него. Сравним рассмот- ренные выше отношения для произвольного треугольника. Задача 14д. В произволь- ный треугольник АВС вписан круг. Сравнить отношения площадей круга и треуголь- ника с отношением длин ок- ружности и периметра тре- угольника. (Рис. 89). Пусть ВС = а, АС — Ь, АВ = с, а радиус вписанного круга — г. 99
Тогда 5 = яг2, Q = -Jр(р - a)(p - b)(p - c), C=2nr, p = a + b + c. Выразим г через стороны a, b и с. Представим площадь ДАВС, как сумму площадей треугольников АВ О, ВОС и СОА. Тогда Q = Qi + Qi + вз, где Qi =~ar, Q2 =^br, Q3 =1Сг. С одной стороны Q = Jp(p - а)(р - b)(p - с), а с другой Q = l/fh + b + с) - ^г2р = гр. Приравнивая правые части равенств 7р(р - а)(р - Ь)(р - с) = гр, выражаем радиус вписанной окруж- Jp(p-a)(p-b)(p-c) ности через стороны треугольника: г = --------------- и со- С и — _ K(-jp(p-a)(p-b)(p-c))2 _ 5 , отделяем отношения — S _ пг2 _ ............................... _ G J р(р-а)(р-Ь)(р-с) р2^р(р-а)(р-Ь)(р-с) _ ^Р(р-а)(р-Ь)(р-с) . Р2 ’ . С _ 2n^p(p-a)(p-b)(p-c) _ it^p(p-a)(p-b)(p-c) Р р2р р* Таким образом, мы убедились, что и для произвольного тре- угольника справедливо утверждение: отношение площади вписан- ного в треугольник круга к площади этого треугольника равно отно- шению длины окружности этого круга к периметру треугольника. Рассмотрим еще один из пра- вильных многоугольников с боль- шим числом строк: (п = 5, 6, 8, 10, 12, 360). Проверим эти отношения для правильного шестиугольника (Рис. 90). Задача 14е. Показать, что для круга, вписанного в правильный шестиугольник, отношение площа- ди круга к площади шестиугольника равно длины окружности к шестиугольника. 90 Рис. описанного отношению периметру 100
Пусть сторона шестиугольника равна а. Выразим радиус ок- ружности г через сторону а. Из zLdjCMj высота О7\ — г = Тогда, оставляя прежние обозначения 5 — площадь круга, Q — площадь многоугольника (в данном случае — шестиугольника), С — длина окружности, Р — периметр, имеем: S = яг2 = ; Л 1 Дд/З х 3a2V3 г, ~ ~ йл/З /7 п г Q = — а —™ 6 = —; С = 2 пг = 2л-—— = лоуЗ , Р = 6а. Сосга- 2 2 2 2 S пЗа22 л лТз С 2na-j3 я вим отношения: — =--------^-j= = —~ = —-г-; — = ——— = —?=• Q 43a243 2-Л 6 Р 26а 2-Л Или по более общему приему. Q = г • 6а; 5 _ лг2-2 _ 2лг _ С Q~~6ar Р~~~Р' Уже можно без изображения соответствующих правиль- ных многоугольников представить разбиение их на п равных между собой треугольников с высотами — радиусами впи- санной окружности и выразить площади следующим обра- зом: Q5 =-r-5as, Qg = 2Г'8°8’612 = уг Иа\2> —>бзбо = ^г'36Оа36о, где Qn — площадь правильного л-угольника, описанного около круга радиуса г, — сторона л-угольника, a пау — периметр л-угольника Р„. Рассмотрев достаточно большее число частных видов описан- ных многоугольников, для которых имеет место равенство отноше- ний площадей и длин, приобретя опыт единообразного подхода к выражению площади многоугольника через периметр и радиус можно сформулировать задачу в такой форме: «Доказать, что пло- щадь любого описанного многоугольника равна произведению по- лупериметра на радиус окружности». Пусть многоугольник, описанный около окружности (О, г) имеет л сторон а^, аз, ..., а„. Обозначим точки касания соответ- ственно Т\, Т1, Тз, ..., Тп. Тогда ОТ\ = ОТ} = ОТ3 = ... = ОТ„ = г. Соединив центр окружности О с вершинами многоугольника получим л треугольников, основаниями которых являются стороны многоугольника щ, а^, аз, ..., а„, а высотами — отрезки ОТ\ = ОТг = ОТ3 = ... = ОТ„ = г. Площадь многоугольника Q = Qi + Qi + Q3 + — + Сл, где Qi — площадь треугольника с основанием at и высотой г. Таким 101
образом, Q = j г (ai + 02 + аз + ...+ a„) = гр, где р — полупери- метр многоугольника. Если считать известным, что площадь многоугольника не зависит от способа разбиения его на треугольники, то уже можно претендовать на строгое доказательство выдвинутой гипотезы о равенстве указанных отношений для любого многоугольника и применять это свойство при решении за- дач в частных случаях. По крайней мере 25 задач (а это 13% всех задач раздела А) из «Сборника конкурсных задач» под ред. М.И. Сканави можно решать с применением результата, добытого путем обобщения задачи до комбинации окружности с описанным многоугольником. Покажем примеры решения некоторых из задач. Задача 1. Найти площадь трапеции, зная длины ее боковых сторон AD = 13, СВ = 15 и радиус вписанного круга г = 6 (Рис. 91). Решение. Так как трапеция является четырехугольником, опи- санным около окружности, то суммы противоположных сторон равны: AD + ВС = DC + АВ = 28. Следовательно, полупериметр р = 28 и площадь Q = гр = 6 -28 т. е. Q = 168. Задача 2. (10. 027). Каждая из трех конгруэнтных окружностей радиуса г касается двух других. Найти площадь треугольника, об- разованного общими внешними касательными. (Рис. 92). Решение. Рассмотрим ок- ружность, вписанную в равно- сторонний АЛИС, площадь кото- рого Q надо найти.t Выразим сторону треугольника а и ради- ус R вписанной окружности че- рез радиус г данных окружностей R = ON = ОМ+ MN= ^O2M + r. 102
О2М — 74г2 - г2 = гл/з , значит, R = г(1 + -у-). Из подо- 2/* rJ3 бия треугольников O1O2Q3 и ЯВС имеем: — = откУДа a 3R a = 2Rj3, Q = pR = ЯЗЯТз = 3>/ЗЛ2 = Зд/З • г2(1 + ^)2. После преобразований получим Q = 2г2(2-73 +3). Задача 3 (10. 072). В равнобедренном треугольнике основа- ние равно 30 см, а боковая сторона равна 39 см. Найти радиус вписанного круга. Решение. Из найденной нами зависимости Q-рг Q_ _ ^/54 15-15 24 _ Р Р 54 Задача 4 (10. 076). Найти отношение радиуса окружно- сти, вписанной в равнобедренный прямоугольный треуголь- ник к высоте, проведенной к гипотенузе. Решение. Т. к. высота h = СН равнобедренного пря- моугольного треугольника равна половине гипотенузы, то стороны треугольника АВ = 2й, АС = ВС = h-j2 . Площадь Q = ^АСВС — h2, полупери- метр р = й(Т2 + 1). Радиус г находим из формулы Q = рг : Г = Q : р. Но Q = |йТ2й72 = й2. р = 2й+2йЛ = h(^ + т й2 Тогдаг = лдаЛ)’ _ г _ 1 ° ~ й “ Л+1 ~ - -Л 1 (Т2+1)(Т2-1) 103
Задача 5 (10. 082). Площадь равнобочной трайецйи, опи- санной около круга, равна 5, а .высота трапеции в два раза меньше ее боковой стороны. Определить радиус вписанного круга (Рис. 94). Решение. Так как дан- ная трапеция является опи- санным многоугольником, то к ней применима выведенная нами формула: S = рг. Так как трапеция является равнобед- ренной и описанным четы- рехугольником, то имеют ме- сто следующие зависимости: AD = СВ, MN=2r, DC + AB = AD + CB=2AD. По условию AD = 2MN. Следовательно, AD = 4r, а р = 8г. Тогда, используя формулу S = рг, получим: S = 8г2. Откуда г - -JS 2^2 42S Ответ: г = -г-. 4 Задача б (10. 085). В равнобедренную трапецию вписана ок- ружность радиуса г. Верхнее основание в два раза меньше ее высоты. Найти площадь трапеции. Решение. Для нахождения площади трапеции по форму- ле S = рг достаточно выразить полупериметр р через радиус впи- санной окружности г. Рис. 95 По условию задачи DC = г, а по свойству описанной тра- пеции г + АВ = р и 2AD — р. Учитывая, что АК= AN— KN = = AN — DM, находим AD из &AKD-. AD2 = AX2 + KD2. Или p2 = (p- 2r)2 + 16/2 = = p2 — 4/r + 4г2 + 16г2; 4pr = 20г2, p = 5г. Тогда площадь трапеции S = 5г2. 104
.. Задача 7(10. НО). Данный квадрат со стороной’а срезан по углам так, что образовался правильный восьмиугольник. Опре- делить площадь этого восьмиугольника (Рис. 96). Решение. Впишем в квад- рат окружность. Нетрудно пока- зать, что она будет вписанной и в образовавшийся правильный восьмиугольник. Радиус этой ок- а ружности г - —, а сторона вось- миугольника Ь = Лр42 = Л2Яз = = ... = Л7Л8 = 2г(Л - 1) = = а(^2 — 1). Полупериметр р = = 4Ь = 4а(Т2 -1). Тогда ис- комая площадь 5= 4o(-j2 — 1) = 2 <£(->/2 — 1). Задача 8 (10. 128). Вокруг квадрата, сторона которого равна а, описана окружность, а около окружности описан правильный шестиугольник. Определить площадь шести- угольника (Рис. 97). Решение. Пусть радиус окружности г, АВ = ВС = CD = = DA = а, А\А2 = А2А3 = ... = А$Аь = Ь. Тогда г = . В равностороннем ДОЛ2Л3 высота ОН = г. Из прямоуголь- 2 ного ЬОАгН имеем Ь2 - — = г2 ЗЬ2 _ 2а2 ™ 4 ' 4 ' Откуда b2=^-,b = ^y-, ЗЬ А. 4оТб р = — = 4Ъ = —, а искомая 2 3 площадь S = рг, т. е. •У = W6 W2 = 2^ 3 2 105
Задача 9 (10. 129). В равнобочную трапецию вписан круг. Одна из боковых сторон ее делится точной касания на отрезки длиной тип. Определить площадь трапеции. Рис. 98 имеем: DK2 = AD2 — АК2 ил + 2тп + п2 — п2 + 2тп — т2 = Решение. Пусть Т, М, N — точки касания, DK 1 АВ, DT = т, ТА = п (Рис. 98). Так как трапеция равнобед- ренная и описанная, то 2AD — полупериметр: р = 2(т + п). Так как Т и N — точки касания, то АТ= AN= п, DT= DM= m. Кроме того KN = DM = m. Выразим ради- ус г через тип. DK = MN = 2г, АК= AN-KN=n- т. Из прямоугольного &ADK (2г)2 = (т + п)2 - (п- т)2 = т2 + . 4г2 = 4тп, г2 = тп, г = 4тп . Площадь трапеции Q = рг= 2(т + п) 4тп . D М К N В Рис. 99 Задача 10 (10. 131). В ромб с острым углов в 30° вписан круг. Площадь круга равна Q. Найти площадь ромба. Решение. Так как высо- та ромба DK = MN = 2г и Z.DAK = 30°, то сторона ромба AD = а = 2DK = 4г (Рис. 99). Тогда периметр Р = 16г. По до- казанному нами соотношению для любых описанных много- С Q ~ угольников имеем — = у. От- с QP Q16r 80 куда S = ~ . С 2itr it Задача 11 (10. 166). Один из катетов прямоугольного треугольника равен 15 см, а радиус окружности, вписанной в этот треугольник, равен 3 см. Найти площадь этого тре- угольника. 106
• Решение. Пусть 7\, Ti и Тз — точки касания сторон тре- угольника и окружности. Тогда периметр Д4ВС Р = 2(ВТ$ + + СГ3 + 1\А) = 2(12 + 3 + Т\А), а площадь S - рг или 5 = -jACBC. Подставляя зна- чения входящих величин, име- ем (15 + 7р4)3 = |15(3 + 7] А). После преобразований по- лучим: 7}А = 5. Тогда искомая площадь S = 20-3 = 60 (кв. см). Рис. 100 Задача 12 (10. 180). Вычислить площадь трапеции по разности оснований, равной 14 см, если известно, что в трапецию можно вписать окружность и боковые стороны равны 13 см и 15 см. Решение. Так как в тра- пецию можно вписать окруж- ность, то суммы противопо- ложных сторон равны и, следо- вательно, AD + СВ — р — полу- периметр и для вычисления площади достаточно знать ра- диус вписанной окружности, который равен половине высо- ты СК. Высоту же можно найти из А.МСВ, стороны которого известны: МВ = 14, МС = 13, СВ = 15. СК = 2r = 2S^B = МВ = ДГ6^ = 12. Г_6 Тогда площадь трапеции Q = рг = 28-6 = 168 (кв. см). Рис. 102 Задача 13 (10. 244). В треугольник вписана окружность радиуса 3 см. Вычислить длины сторон треугольника, если одна из них разделена точкой касания на отрезки с длинами 4 см и 3 см. (Рис. 102). 107
Решение. Пусть в ААВС точки Т\, Т2 и Ту — точки каса- ния сторон треугольника вписанной окружности, и пусть точка Т\ делит сторону АС на отрезки АТ} = 4 см и Т\С = 3 см. Тогда, по свойству касательных, проведенных из одной точки к окружно- сти (или по равенству прямоугольных треугольников) имеем: ATi = АТ3 = 4, С71 = СТ2 = 3, ВТ2 = ВТ3 = х. Периметр &АВС Р = 2(4 + 3 + х) = 2(7 + х), а полупериметр р = 7 + х. Выражая площадь треугольника двумя способами, найдем х, а затем и стороны &АВС. Пусть 5 = рг и S = 7р(р - о)(р - Ь)(р - с) . Тогда (7 + х)3 = 7(7 + х)(7 + х-3-х)(7 + х-7)(7 + х-4-х), 3(7 + х) = 7(7 + х)4х-3. После элементарных преобразований получим: х2 - 14х - 147 = 0. Корень уравнения, удовлетворяющий условию задачи xj = 21. Тогда стороны треугольника АС = 7см, ВС = 24 см, АВ — 25 см. Можно было бы продолжить приведение примеров на широкое использование фактов, полученных нами в связи с обобщением. Началом всему послужила равнобедренная тра- пеция. Не откроем ли мы еще закономерности у этой богатой идеями фигуры с тем, чтобы применять их в решении задач сначала связанных с ней самой, а затем, возможно, и их удастся распространить на другие фигуры. При решении задачи 9 (10. 129) в качестве промежуточного этапа мы получили г = 4пм , где тип - отрезки боковой сто- роны, на которые делится эта сторона точкой касания в равно- бочной трапеции. Но мы с самого начала ознакомления с трапецией при- выкли ассоциировать среднее арифметическое двух чисел с длиной средней линии не только у равнобедренной трапеции, но и у произвольной. Возникает вопрос: не будет ли справед- ливым случайно «открытая» закономерность равнобочной тра- пеции (г = 4тп) для произвольной, но описанной около круга трапеции. 108
Возникла задача-теорема: «Если около окружности ра- диуса г описать произвольную трапецию, то радиус этой ок- ружности есть среднее гео- метрическое между отрезка- ми, на которые разбивается любая боковая сторона точ- кой касания». (Рис. 103). Решение. Если DT4 = т, Т4А = п, CTi = ки ТгВ = /, то по свойству касательных, про- веденных из одной точки к окружности, имеем: РТ3 = т, СТу = к, АТ\ = п, ВТ\ = I. При- менив теорему Пифагора к тре- угольникам ADAt и ВСВ\, полу- чим (т + л)2 + (л — т)2 = 4г2 и (к + I)2 + (I — к)2 = 4г2, откуда г = 4тп = Jkl. Теперь эта закономерность может быть использована для решения других задач, связан- ных с произвольной описанной Рис. 103 Рис. 104 трапецией. С трапецией мы уже связали нахождение двух средних ве- личин: арифметического и геометрического (пока для двух ве- личин). Не найдется ли отрезка в трапеции, характеризующего среднее гармоническое двух других отрезков (например, длин оснований)? Начнем исследование с простейшего случай равно- бедренной описанной около круга трапеции (Рис. 104). Пусть DT4 = DTy = ТуС— СТ2 — tn, Т4А = АТ} ~ Т}В = ВТ2 — п, а Т4Т2 = /. Тогда из подобия треугольников QTO и А^РА имеем: АР _ ОТ4 DAi ~ QT4 или AD QT4 = DAi-QTt. Перепишем в обозначениях т, л и /: (л» + л) = 2г • г. Учи- тывая, что г2 = тп, разделим обе части полученного равенства 109
, ml nl 2 112 на Imn. —— + —— = - и окончательно — + — = — 2lmn 2lmn I 2п 2т I т. е. 1 1 = 2 DC + АВ “ Т4Т2 ' Отрезок, соединяющий точки касания боковых сторон равнобедренной описанной трапеции есть среднее гар- моническое отрезков, являющихся основаниями этой трапеции. Но это доказательство основывалось на подобии прямо- угольных треугольников, которое имеет место только в случае параллельности отрезка I основаниям. Ясно, что этот отрезок не будет параллелен основаниям даже в произвольной описанной трапеции. D С Рис. 105 Подумаем, как же провес- ти параллельный основаниям отрезок, чтобы он оказался средним гармоническим для них. Вспоминаем задачу о средней линии трапеции, ко- торая делится диагоналями на равные части. Возникает правдоподобное предположе- ние об отрезке, проведенном через точку пересечения диа- гоналей (Рис. 105). В результате параллельности достаточно большого числа от- резков возникает масса подобных треугольников. Чтобы выбрать нужные, сформулируем заключение теоремы в данных на 2 1 1 рис. 105 обозначениях: —— = + —. Отрезок MN состоит из MN DC АВ отрезков МО и ON (в предыдущем доказательстве этот отрезок состоял из двух равных отрезков). Так как ДАСО- ДАОМ, то АО АС (1) а аог ON ОС &ONC ~ ДЛ5С, то —— = —7 ’ АВ АС (2) и А МП П А A ОП МО OD &M0D - &ABD, то —— = АВ BD (3). ОС OD МО ON Так как -Гт? = , то и —— = ——. AC BD АВ АВ Откуда МО = ON. 110
Заменяя во (2) равенстве ON = ОМ, получим (4). AD A Is Полученное равенство сложим с (1) почленно: ОМ ОМ АО ОС лг. . АП т ОМ ОМ . с» +Л«=ЛС + ЛС’ "° ЛО + ОС = ИС. Тогда ^ + ^ = 1 Разделив обе части полученного равенства на ОМ, получим: 1112 —+ —= ^^- = -^г, т. е. отрезок, проходящий через точку пересечения диагоналей трапеции параллельно ее оенованию, есть среднее гармоническое между основаниями трапеции. На комбинации трапеции с окружностью мы начали процесс обобщения предложения о равенстве отношений длин и площа- дей, дойдя до доказательства равенства этих отношений в произ- вольном многоугольнике, описанном около круга в планиметрии. Рассмотрим комбинацию аналогичных тел в пространстве и сформулируем гипотезу об аналогичных отношениях. Начнем исследование с такого вида многогранника, в сече- нии которого будут такие равнобедренные трапеции, в которые вписывается круг радиуса г. Это правильная четырехугольная усеченная пирамида, высота которой равна 2г. У нее не только сечения, но и боковые грани трапеции, элементы которых легко выражаются через г. Если круг, вписанный в осевое сечение та- кой усеченной пирамиды вращать вокруг высоты ее, то полу- ченный шар окажется вписанным в эту пирамиду, т. е. он будет касаться всех шести граней многогранника. Аналогиями длины и пло- щади в пространстве будут площадь полной поверхности границы тел и объемы этих тел. Пусть сторона верхнего ос- нования а, нижнего — в и апо- фема ММ' = I. Площадь полной поверхно- сти пирамиды Q, площадь сфе- ры 5, объем усеченной пирами- ды Ипир , объем шара — Ишара. Выразим необходимые для срав- нения величины через а и Ь, Jab зная, что г = . D' С М' Рис. 106 а, б 111
Площадь полной поверхности усеченной пирамиды склады- вается из площадей двух квадратов и четырех равных трапеций с основаниями а и b и высотой /. Но апофема I является боковой стороной трапеции MNN’M' (осевого сечения), т. е. Q = а2 + Ь2 + 4^)/= а2 + Ь2 + 4^±*)/ = а2 + Ь2 + 4^^ ; Q = 2(а2 + ab +А2). Площадь поверхности шара 5 = 4П/2 = 4п—. Тогда отношение S = 2(Д лд/Л ) ’ °бъеМ пиРамиды Кпир = |h(Qi + Qi + ^QiQi), где Qi и Qi — площади оснований усеченной пирамиды, а А — высота ее. В нашем случае h = 2r, Q\ = a2, Qi = А2. Тогда, Кпир = 12r(a2 + ab + Ь2). Но г = , следовательно, IZ Jab(a2+ab+b2) 4 3 4nab4ab naby/ab r ------j------, j— Ото- ~ Ншр 4ab(a2 +ab+b2)6 2(a2+ab+b2) шение объемов " --— -------------—= Таким 'шара ЗпоЬу ab nab образом на частном примере гипотеза о равенстве отношений площадей и объемов подтвердилась. Убедимся в справедливости этого равенства для описанного около шара куба с ребром а. Q = 6а2, г = |, ^куба = a3- S^nfi^na2-, Кшара = |яг3 =^-. е_б. ^__б S я’ Гш п Для любого описанного л-гранника по определению каждая грань касается шара. Радиус, проведенный в точку касания пер- пендикулярен грани. Следовательно если все вершины много- гранника соединить с центром описанной сферы, то данный л-гранник разобьется на л пирамид с вершинами в центре и вы- сотами равными радиусу г. Объем n-гранника можно вычислить как сумму объемов пи- рамид. Если обозначить площадь каждой грани Qh то полная по- верхность л-гранника Q = Qi + Qi + + ... + Q„. Объем одной пирамиды V{. Vf=^Qtr, тогда Кпир = |r(Q, + Q2+...+Q„) = . 112
.» 4л з у =______Г ш з Объем вписанного шара . Составим отношение обье- К„ Q мов = —=~ = —, но это отношение поверхности мно- Иш 3-4w3 4лг2 гогранника к поверхности сферы. Это справедливо для любой описанйой призмы и пирамиды, в том числе усеченной. А так как конусы и цилиндры рассматриваются как предельные поло- жения пирамид и призм, то и для них имеют место указанные отношения. Например. Пусть в конус вписан шар (Рис. 107). Тогда: _ 4яг2 _ 4г2. Иш _ J*'3 _ 4г3 _ 4г2 SK nR(R+[)~ Rp’ Гк 1я/}2д Rpr Rp' Теперь применим полученное равенство отношений объе- мов и площадей поверхностей для комбинации поверхности любого известного нам тела (многогранной, цилиндрической) описанного около шара при решении задач. В следующих задачах из «Сборника задач» под ред. Скана- ви М.И. добавим: «и выразить объем описанного тела через его пол- ную поверхность и радиус шара». Задача 14 (11. 086). В конус, осевое сечение которого есть рав- носторонний треугольник, вписан шар. Найти объем конуса, если а. 32л з объем шара равен -у см. Решение. Рассмотрим осе- вое сечение комбинации тел (Рис. 108). Пусть ОТ = г — ради- ус шара, НВ = R — радиус осно- вания конуса, СВ = / — обра- зующая конуса, 0П ~ полная поверхность конуса. Объем ко- нуса найдем из отношения S’ Для нахождения Q выразим из равностороннего треугольника Ли /через г. R = г4з , / = 2гТз . 113
Q = л/?2 + i 12л/? - Зл/'2 + 2г43кг^3 = 9лг2. Но Ик Q ЗКк 9лг2 Ик _ IZ .. „ ^г- = лл4- = —т 1 л=3; Ис = 24л. Покажем, что 32л 5 32л 4лг2 8я к 3 1 4 ч Ик = - Qr, найдя г из формулы объема шара: Кш = - лг , откуда 3 ЗКШ 3-32л о — IZ 9лг2г г = —И- = —- = 8. Тогда Кк = = 24л. 4л 3-4л 3 Задача 15 (11. 173). Около шара радиуса г описана пра- вильная шестиугольная призма. Определить ее полную по- верхность. Рис. 109 Решение. Так как приз- ма правильная, то сторона осно- 2rj3 вания а 3 , а так как она описанная, то высота ее h = 2г. В решении этой задачи можно использовать дважды получен- ное нами обобщение: для на- хождения площади основания Sf, = рг и для нахождения пол- ной поверхности Q К = -^0г, т. к. проекцией шара на плос- кость основания является круг того же радиуса г (Рис. 109). Площадь основания = Заг =2г^3г = 2-ч/Зг2, тогда объем призмы Ипр = S^h = 2-j3ri2r = 4^313. Искомая поверхность 0 = Задача 16. Около шара описана правильная четырех- угольная усеченная пирамида, у которой стороны основания равны тип. Определить отношения объемов пирамиды и шара и расстояние между точками касания противополож- ных граней. 114
Решение. В соответствии с условием задачи осевое сечение данной комбинации тел пред- ставляет собой равнобедренную трапецию, описанную около круга (Рис. ПО). Именно эта комбинация в планиметрии привела нас к но- вым знаниям о средних величи- нах. Используем эти знания для решения задачи. Т\Т2 — это рас- стояние между точками касания шара с противоположными гранями. TjTj является средним 1 1 2 гармоническим основании трапеции: — + — = откуда m п 7|72 TjT’j = lHHL. Высота трапеции Л = 2г (г — радиус круга и шара) тп+п , /-- 4тП является средним геометрическим h = Jmn , и значит г = —у. Выразим объем пирамиды и шара через данные величины тип. + s2 + Твд)- ЛАЛ-) У _ 4 з _ 4njmnmri) _ njmnmn Кшарв-зяг “ $ • Найдем отношение Кпир - Vw(m2+«2+mn)6 _ 2 т2+п2+тп "ш Зя-Jmnmn я тп Мы показали на примере одной задачи касающейся част- ного случая. Возникает мысль об обобщении, как подбираются задачи, на которых подтверждается справедливость обобщаю- щего утверждения, по возможности доведя его почти до теоре- мы в принятой аксиоматике, показали, как используется полу- ченный теоретический факт для уже решенных задач, опираю- щихся на другие геометрические факты и, наконец, нашли но- вые пути решения предстоящих задач. Но в совершенствовании искусства решать задачи важно и другое умение: последовательно узнавая новые теоретические сведения возвращаться к одной и той же задаче с тем, чтобы решить ее на основе других знаний. В качестве примера рас- смотрим несколько решений задачи №3 из раздела дополни- тельных задач. Повторим ее условие. 115
Задача 3. В равнобедренном треугольнике АВС угол при вершине С равен 20°. Из вершины А и В проведены лучи, со- ставляющие с основанием углы 50° и 60° и пересекающие противоположные стороны в точках D и Е соответственно. Найти углы треугольника ODE, если О — точка пересечения отрезков AD и BE. Решение первое. Это решение доступно школьнику с минимальным запасом знаний первых разделов геометрии. В самом деле, проведем ЛГтак, чтобы Z.CAF = 20° и проведем от- резки EF и QD, где Q — точка пересечения AF и BE. Из &AQB имеем AQ = QB = АВ. В &ADB DB = АВ (против равных углов лежат равные стороны). Тогда, заменяя отрезок АВ равным ему отрезком DB, получим, QB = DB. Следова- тельно, &.QBD равнобедренный и ZBQD = Z.BDQ = 80° и Z.FQD = 180° — (60° + 80°) = 40°. Треугольники EFQ и FDQ ока- зываются равнобедренными с одним и тем же основанием QF, поэтому ED ± QF. Используя свойства указанных равнобед- ренных треугольников легко находим искомые углы: Z.DEQ = 30°, /.EDO = 80°, /.DOE = 70°. Второе решение. Проводим ГС||ВС, ВС 1ИС, СО|ИВ=> BCFG — ромб (обосновать). Значит BF± CG и L — середина BF. L — середина CG => CD = DG и \CDG — равнобедренный. Z.BCG = 40°. ZDCG = /.DCB - Z.BCG = 60°. &DCG — равносторонний и Z.CDF = 30°. СМКВ — ромб. СК — диагональ => /ВКС = Z.CKF => /CKG=/CKE. /ВСК= 50°. \CGK= \СКЕ. \CEF = ECFD. Z.CEF = 30°. Рис. 112 116
Третье решение. ABAC — -^часть окружности — 18 | часть полуокружности. Проведем BF, Z.ABF = 30°. AFBC — вписанный, равный 80° — 30° = 50°. Из точки D проводим DF: &BCF = &CDF (св. прав, много- угольника) => ACDF = Z.FBC = 50°. Z.CDF — вписанный => про- должение DF пройдет через Я и пересечет АВ в некоторой точке Q. Совпадет ли Q и Е! Z.AHD — вписанный, опирающийся на дугу 60° = ZAHD = 30°. Z.HAK — центральный - 60° => Z.HGA = 90°. В \HGA GA = i НА => и GK. HD — серединный перпен- дикуляр к-АК => KQ = QA и A.KBQ = S.QBC. \СЕА — равнобед- ренный, ЛЕСА = 20°. Т. е. Q = Е. ZFQC = X. Из &.GQA Z.GEA = Z.DEB = 70°. 117
IV часть СПРАВОЧНЫЙ МАТЕРИАЛ Приложение 1. СТРУКТУРНАЯ ТАБЛИЦА-МАТРИЦА 1 2 3 4 5 6 7 8 9 10 11 Точка Прямая Плоскость Луч (полуплоскость, полупространство) Отрезок Угол (плоский, двугранный, многогранный) Многоугольник (многогранник) Круг (шар) Конус Числа Точка 1,1 1,2 1,3 1,4 1,5 1,6 1,7 1,8 1,9 1,10 1,11 Прямая 2,1 2,2 2,3 2,4 2,5 2,6 2,7 2,8 2,9 2,10 2,11 Плоскость 3,1 3,2 з,з 3,4 3,5 3,6 3,7 3,8 3,9 3,10 3,11 Луч (п/плоск.) (п/простр.) 4,1 4,2 4,3 4,4 4,5 4,6 4,7 4,8 4,9 4,10 4,11 Отрезок 5,1 5,2 5,3 5,4 5,5 5,6 5,7 5,8 5,9 5,10 5,П Угол (плоский) (п-гранн.) 6,1 6,2 6,3 6,4 6,5 6,6 6,7 6,8 6,9 6,10 6,11 л-угольник (многогранник) 7,1 7,2 7,3 7,4 7,5 7,6 7,7 7,8 7,9 7,10 7,11 Круг (шар) 8,1 8,2 8,3 8,4 8,5 8,6 8,7 8,8 8,9 8,10 8,11 Конус 9,1 9,2 9,3 9,4 9,5 9,6 9,7 9,8 9,9 9,10 9,11 Цилиндр 10,1 10,2 10,3 10,4 10,5 10,6 10,7 10,8 10,9 10,10 10,11 118
ПРИМЕЧАНИЕ. Содержащийся фактический материал помещен в пособии как подсобный — справочный. К нему можно обратиться как при решении имеющихся задач так и при возникновении собственных задач-гипотез, при состав- лении и решении задач-обобщений тем школьникам, у ко- торых еще недостаточно накоплено геометрических фактов. Даваемая описательная характеристика понятий не всегда соответствует только одной из принятых систем аксиом. Для тех, у кого в результате творческой работы возникнут задачи строго логического обоснования геометрических фактов мы предлагаем списки различных аксиоматик, по- ложенных в основу наиболее распространенных школьных учебников геометрии, а также своеобразную аксиоматику планиметрии, на которой основано учебное пособие по геометрии под ред. А.Н. Колмогорова. Необходимость обращения к строгому определению гео- метрических понятий может возникнуть даже при выполне- нии простейших упражнений из первой части пособия. Так, например, ответы на один и тот же вопрос «Обладает ли от- ношение параллельности прямых свойством транзитивно- сти?» будут различны в зависимости от определения парал- лельных по А.Н. Колмогорову и по А.В. Погорелову. 119
Приложение 2. АКСИОМАТИКИ ГЕОМЕТРИИ А. По Л. С. АТАНАСЯНУ, В. Ф. БУТУЗОВУ и др. Основные объекты (фигуры): точка, прямая, плоскость. Основные отношения: «лежит на», «лежать между», «наложение». Аксиомы: I. На каждой прямой и в каждой плоскости имеются точки. II. Имеются по крайней мере три точки, не лежащие на од- ной прямой, и по крайней мере четыре точки, не лежащие в одной плоскости. III. Через любые две точки проходит прямая, и притом только одна. IV. Через любые три точки, не лежащие на одной прямой, проходит плоскость, и притом только одна. V. Если две точки прямой лежат в плоскости, то все точки прямой лежат в этой плоскости. VI. Если две плоскости имеют общую точку, то они имеют об- щую прямую, на которой лежат все общие точки этих плоскостей. VII. Из трех точек прямой одна, и только одна, лежит меж- ду двумя другими. VIII. Каждая точка О прямой разделяет ее на две части — два луча — так, что любые две точки одного и того же луча ле- жат под одну сторону от точки О, а любые две точки разных лучей лежат по разные стороны от точки О. IX. Каждая прямая а, лежащая в плоскости, разделяет эту плоскость на две части (две полуплоскости) так, что любые две точки-одной и той же полуплоскости лежат по одну сторону от прямой а, а любые две точки разных полуплоскостей лежат по разные стороны от прямой а. X. Каждая плоскость а разделяет пространство на две части (два полупространства) так, что любые две точки одного и того же полу- пространства лежат по одну сторону от плоскости а, а любые две, точ- ки разных полупространств лежат по разные стороны от плоскости а. XI. Если при наложении совмещаются концы двух отрезков, то совмещаются и сами отрезки. XII. На любом луче от его начала можно отложить отрезок, равный данному, и притом только один. - XIII. От любого луча в данную полуплоскость можно отложить угол, равный данному неразвернутому углу, и притом только один. XIV. Два равных угла hk и hfa, лежащие в плоскостях, являю- щихся границами полупространств Р и Pj, можно совместить на- ложением так, что при этом совместятся полупространства Р и Р\, 120
причем это можно сделать двумя способами: в одном случае со- вместятся лучи h и h\, к и к\, а в другом — лучи h и к\, к и h\. XV. Любая фигура равна самой себе. XVI. Если фигура Ф равна фигуре Ф], то фигура Ф\ равна фигуре Ф. XVII. Если фигура Ф1 равна фигуре Фу, а фигура Фу равна фигуре Фз, то фигура Ф\ равна фигуре Фу. XVIII. При выбранной единице измерения отрезков длина каждого отрезка выражается положительным числом. XIX. При выбранной единице измерения отрезков для лю- бого положительного числа существует отрезок, длина которого выражается этим числом. XX. В любой плоскости через точку, не лежащую на данной прямой этой плоскости, проходит только одна прямая, параллельная данной. Б. По А.В. ПОГОРЕЛОВУ Основные объекты (фигуры): точка (1), прямая (2), плоскость (3). Основные отношения: принадлежность, порядок, равенство (одинаковость), параллельность. Аксиомы: I (1, 2). Какова бы ни была прямая, существуют точки, при- надлежащие этой прямой, и точки, не принадлежащие ей. Через любые две точки можно провести прямую, и только одну. II (1, 2). Из трех точек на прямой одна и только одна лежит между двумя другими. III (5, И). Каждый отрезок имеет определенную длину, большую нуля. Длина отрезка равна сумме длин частей, на ко- торые он разбивается любой его точкой. IV (2, 3). Прямая, принадлежащая плоскости, разбивает эту плоскость на две полуплоскости. V (6, 11). Каждый угол имеет определенную градусную меру, большую нуля. Развернутый угол равен 180°. Градусная мера угла равна сумме градусных мер углов, на которые он разбивается любым внутренним лучом. VI (4, 5). На любой полупрямой от ее начальной точки можно отложить отрезок, заданной длины, и только один. VII (4, 6). От любой полупрямой на содержащей ее полу- плоскости в заданную полуплоскость можно отложить угол с заданной градусной мерой, меньшей 180°, и только один. VIII (7, 7). Каков бы ни был треугольник, существует рав- ный ему треугольник в данной плоскости в заданном располо- жении относительно данной полупрямой. 6—1224 121
IX (2, 2). На плоскости через точку, не лежащую на данной пря- мой, можно провести не более одной прямой, параллельной данной. X (1, 3). Какова бы ни была плоскость, существуют точки, принадлежащие этой плоскости, и точки, не принадлежащие ей. XI (3, 1, 2). Если две различные плоскости имеют общую точ- ку, то они пересекаются по прямой, проходящей через эту точку. XII (2, 1, 3). Если две различные прямые имеют общую точку, то через них можно провести плоскость, и притом только одну. В. ПО А.Н. КОЛМОГОРОВУ. Основные понятия: точка, прямая, плоскость, расстояние от одной точки до другой. Аксиомы: 11. Каждая прямая есть множество точек. 1г- Для любых двух точек существует одна и только одна со- держащая их прямая. 1з- Существует хотя бы одна прямая; каждой прямой при- надлежит хотя бы одна точка. Пр Любым точкам А и В поставлено в соответствие неотри- цательное действительное число |ЛБ|, называемое расстоянием от точки А до точки В. Расстояние |ЛВ| равно нулю тогда и только тогда, когда точки А и В совпадают. II2- Расстояние от точки А до точки В равно расстоянию от В до точки А: |Аб| = 1Я4|. Из- Для любых точек А, В и С расстояние от Л до С не боль- ше суммы расстояний от Л до В и от В до С. | АС\ < | АВ| +1 ВС\. III1- Три точки принадлежат одной прямой тогда и только тогда, когда одна из них лежит между двумя другими. III2. Любая точка О прямой р разбивает множество отлич- ных от О точек прямой р на два непустых подмножества так, что точка О лежит между любыми двумя точками, принадлежа- щими разным подмножествам. III3. Для любого неотрицательного действительного числа а на заданном луче с началом О существует одна и только одна точка, расстояние от которой до начала О равно числу а. III4. Любая прямая разбивает множество не принадлежащих ей точек плоскости на две непустые выпуклые области. IV. Для любой пары лучей O[At и О3А2 и примыкающих к ним по- луплоскостей «1 и ос2 существует единственное перемещение, отобра- жающее луч Оу4\ на луч QA}, а полуплоскость eq на полуплоскость а2. V. Через любую точку А плоскости р проходит не более од- ной прямой, параллельной данной прямой р. 122
Приложение 3. ФАКТИЧЕСКИЙ МАТЕРИАЛ А. ХАРАКТЕРИСТИКА ОБЪЕКТОВ 1. Точка. 1.2. Прямая. 1.3. Плоскость. 1.4. Полупрямая (луч) [полуплоскость, полупространство] одна из двух частей прямой [плоскости, пространства], на кото- рые они разбиваются своей фиксированной точкой (началом лу- ча) [прямой (границей полуплоскости), плоскостью (границей полупространства)]. 1.5. Отрезок — часть прямой, состоящая из двух точек (концов отрезка) и всех точек прямой (внутренних точек), лежа- щих между концами. 1.6. Угол. 1.6.1. Одномерный угол — фигура, состоящая из точки (вершины угла) и двух полупрямых, исходящих из этой точки (сторон угла). А. Развернутый, прямой, острый, тупой. 1.6. Г. Двугранный угол — фигура, состоящая из прямой (ребра двугранного угла) и двух полуплоскостей (граней двугран- ного угла), исходящих из этой прямой. 1.6.2. Плоский угол — фигура, состоящая из вершины, двух сторон и всех лучей, проходящих между сторонами (то есть пе- ресекающих, любой отрезок с концами на сторонах угла). 1.6.2'. Двугранный угол — часть пространства, ограниченная двумя полуплоскостями, исходящих из одной прямой. 1.6.3. Линейный угол двугранного угла — угол, образованный пересечением двугранного угла плоскостью, перпендикулярной к ребру этого двугранного угла. 1.6.4 [4'] Многогранный (л-гранный) угол — пространствен- ная фигура, состоящая из фиксированной точки (вершины угла) и всех лучей, исходящих из вершины и проходящих через точки одномерного простого [плоского простого] л-угольника, при- надлежащего плоскости, не проходящей через вершину. Пло- ские углы с общей вершиной, образованные лучами, пересе- кающими стороны л-угольника — грани многогранного угла. Двугранные углы многогранного угла — углы, образованные его гранями. Точки лучей, проходящих через внутренние точки плоского многоугольника — внутренние точки многогранного угла. 6* 123
1.7. Многоугольник (л-угольник) и многогранник (т-гранник). 1.7.1 [Г] Многоугольник (л-угольный полигон, л > 3) [Многогранная, лг-гранная, т > 4, поверхность] — фигура, обра- зованная из п отрезков (сторон) [т многоугольников (граней)] так, что каждый из концов (вершин) стороны является концом (вершиной) другой стороны, смежной с первой [каждая сторона (ребро поверхности) любого из этих многоугольников (граней поверхности) является стороной еще одного многоугольника, смежного с первым]. 1.7.2 [2'] Плоский многоугольник [многогранник (полиэдр)] — конечная часть плоскости [пространства], ограниченная полиго- ном (простой замкнутой ломаной) [многогранной поверхно- стью]. 1.7.3 [3'] Выпуклый многоугольник [многогранник] — много- угольник [многогранник], расположенный в одной [в одном] полуплоскости [полупространстве] относительно любой прямой [плоскости], содержащей его сторону [грань]. 1.7.3А. Треугольник — многоугольник с тремя вершинами. 1.7.3А1. Прямоугольный треугольник — треугольник, один из углов которого прямой. 1.7.3Аг. Остроугольный треугольник — треугольник, все углы которого острые. 1.7.ЗА3. Тупоугольный треугольник — треугольник, один из углов которого тупой. 1.7.ЗА4. Равнобедренный треугольник — треугольник, две стороны которого равны. 1.7.ЗА5. Равносторонний треугольник — правильный трехсто- ронний. 1.7.3В. Четырехугольник — многоугольник с четырьмя вер- шинами. 1.7.3Bj. Трапеция — выпуклый четырехугольник, у которого две противоположные стороны параллельны (основания), а две другие непараллельны (боковые стороны). 1.7.3В1». Равнобочная (равнобедренная) трапеция — трапе- ция, у которой боковые стороны равны. 1.7.ЗВ2. Параллелограмм — четырехугольник, противопо- ложные стороны которого попарно параллельны. 1.7.3В2а- Прямоугольник — параллелограмм с прямым углом — четырехугольник с равными углами — многоугольник с четырьмя вер- шинами и равными углами. 124
1.7.3B2b[cl Ромб [квадрат] — правильный параллелограмм [прямоугольник]. 1.7.3'А. Пирамида (л-угольная пирамида) — многогранник, одна из граней которого n-угольник (основание пирамиды), а все остальные (боковые грани) — треугольники с общей вершиной (вершиной пирамиды), не принадлежащие плоскости основания. Пирамида — многогранник, полученный от пересечения многогранного угла и полупространства с границей П, которому принадлежит вершина многогранного угла, а граница П пересе- кает все ребра угла в различных точках. ' 1.7.3'А]. Правильная пирамида — пирамйда, основание ко- торой — правильный многоугольник, а ортогональная проекция вершины на плоскость основания совпадает с центром этого многоугольника. 1.7.3А2. Усеченная пирамида — часть пирамиды, ограничен- ная основанием (нижним основанием усеченной пирамиды), частями боковых граней и сечением пирамиды плоскостью (верхним основанием), параллельной основанию и не проходя- щей через вершину пирамиды. 1.7.3'В. Призма (л-угольная призма) — многогранник, две грани которого — равные л-угольники, лежащие в параллельных плоскостях (основания призмы), а остальные л граней (боковые грани) — параллелограммы. Призма — многогранник, состоящий из двух плоских мно- гоугольников, лежащих в разных плоскостях и совмещаемых параллельным переносом, и всех отрезков, соединяющих со- ставляющие точки этих многоугольников. 1.7.3'В][В2] Прямая [наклонная] призма — призма, все бо- ковые грани которой перпендикулярны [не перпендикулярны] плоскостям оснований. 1.7.3'Вз Правильная призма — прямая призма, основанием которой является правильный многоугольник. 1.7.4 [4'] Правильный многоугольник (л-угольник, л-сторонник, п > 3) [многогранник (т-гранник, 4 < /л < 20] — выпуклый мно- гоугольник, все стороны и все углы которого равны [выпуклый многогранник, все грани которого правильные равные многоуголь- ники и все многогранные углы при вершинах равны]. 1.7.4 А. Квадрат — правильный четырехугольник. 1.7.4'А] Правильный тетраэдр — четырехгранник, грани ко- торого правильные треугольники (6 ребер, 4 вершины). 125
1.7.4'А2. Правильный гексаэдр (куб) — шестигранник, грани' которого квадраты (12 ребер, 8 вершин). , 1.7.4'Аз. Правильный октаэдр — восьмигранник, грани ко- торого правильные треугольники (12 ребер, 6 вершин). 1.7.4'А4. Правильный додекаэдрдвенадцатигранник, гра- ни которого правильные пятиугольники (30 ребер,.20 вершин). 1.7.4'Ад. Правильный икосаэдр —двенадцатигранник, грани которого правильные треугольники (30 ребер, 12 вершин). 1.8. Круг и шар. 1.8.1 [1'] Окружность (сфера) — фигура, состоящая из всех точек плоскости [пространства], удаленных от данной точки плоскости [пространства] (цента окружности [сферы]) на данное расстояние (радиус окружности[сферы]). 1.8.2 [2'] Круг [шар] — фигура, состоящая из всех точек плоскости [пространства], расстояние каждой из которых до фиксированной точки плоскости [пространства] не превышает заданного расстояния г (радиуса круга [шара]). Окружность — граница круга. , Сфера — граница шара. Круг — часть плоскости, ограниченная окружностью. Шар — часть пространства, ограниченного сферой. 1.8.2 [2'] А. Круговой [шаровой] сегмент — общая часть по- луплоскости и круга [полупространства и шара]. 1.8. [2'] Bi Шаровой слой — часть шара, расположенная между двумя параллельными плоскостями. 1.8. [2'] В2. Шаровой пояс (зона) — часть поверхности ша- рового слоя. 1.8.2 [2'] С. Крутовой [шаровой] сектор — общая часть круга [шара] и центрального угла [кругового конуса с вершиной в центре шара]. 1.9. Конус и пирамида. 1.9.1. Коническая поверхность — поверхность, образованная прямой линией (образующей), имеющей неподвижную точку (вершину) и движущуюся вдоль данной линии (направляющей). 1.9.1 [Г] А. Круговая коническая [пирамидальная] поверх- ность — коническая поверхность, направляющей которой явля- ется окружность [одномерный многоугольник]. 1.9.2 [2']. Круговой конус [пирамида] — тело, ограниченное одной из плоскостей круговой конической поверхности [пирамидальной поверхности] (боковой поверхностью конуса [пирамиды]) и плоскостью не проходящей через вершину по- 126
верхностй (вершину конуса [пирамиды]). 1.9.2 [2'] А. Усеченный конус [усеченная пирамида] — тело, заключённое Между основанием конуса [пирамиды] и плоско- стью пересекающей конус [пирамиду] параллельно основанию. 1.10. Цилщдр и призма. 1.10.1. Цвйнцдрическая поверхность — поверхность, образуемая движением прямой (образующей), перемещающейся параллельно самой себе и пересекающей данную линию (направляющую). 1.10.1 [1'] А. Круговая цилиндрическая [призматическая] по- верхность — цилиндрическая поверхность, направляющая кото- рой окружность [одномерный многоугольник]. 1.10.2 [2'] Круговой цилиндр [призма] — тело, ограниченное круговой цилиндрической [призматической] поверхностью (боковой поверхностью) и двумя параллельными плоскостями (плоскостями оснований). Круговой цилиндр [призма] — тело, состоящее из двух кругов [многоугольников], лежащих в разных плоскостях и совмещае- мых параллельным переносом, и всех отрезков, соединяющих соответствующие точки этих кругов [многоугольников]. 1.10.2 [2'] А. Прямой [-ая] круговой цилиндр [призма] — круговой [-ая] цилиндр [призма], образующие которого [-й] перпендикулярны плоскости, в которой лежит направляющая. 1.11. Тела вращения. 1.11.1. Поверхность вращения — поверхность, образуемая вращением плоской линии вокруг прямой (оси вращения), рас- положенной в плоскости этой линии. 1.11.2- 5. Сфера [поверхность 3) прямого кругового цилиндра; 4) прямого кругового коцуса; 5) прямого кругового усеченного конуса] — фигура, образованная вращением полуокружности [прямоугольника, прямоугольного треугольника, прямоугольной трапеции] вокруг своего диаметра [стороны, катета, боковой стороны, перпендикулярной основанию]. 1.11.6- 9. Шар [прямой круговой цилиндр, прямой круговой конус, усеченный коцус] — тело, образованное вращением полу- круга [плоских прямоугольника, прямоугольного треугольника, прямоугольной трапеции] вокруг своего диаметра [стороны, ка- тета, боковой стороны, перпендикулярной основанию]. 1.11.10- 11. Шаровой сектор [сегмент] — тело, образованное вращением кругового сектора [сегмента] вокруг оси симметрии его. 1.12.1-4. N, Z, Q, R. 127
Б. ХАРАКТЕРИСТИКА ОТНОШЕНИЙ 2-Х ОБЪЕКТОВ 1 2 (1’ 1 Пусть задана некоторая фигура F и каждой точке ее М ста- I вится в соответствие единственная точка плоскости ’ [пространства] М'. множество точек, соответственных точкам j фигуры F образует в плоскости [пространстве] некоторую ’ фигуру F'. Говорят, что фигура F’ получена преобразованием фигуры F. Можно сказать также, что фигура F’ является обра- зом фигуры F для данного преобразования. Фигуру F называют прообразом фигуры F’. Если М’ — точка фигуры F’, соответст- вующая точке Мфигуры F, то говорят, что М' — образ точки М, а точка М — прообраз точки М'. 2. (1, 1) А. А. Движение фигуры — преобразование, сохраняющее рас- стояния между точками. А1. Параллельный перенос — движение, при котором все точки фигуры перемещаются в одном и том же направлении на г одно и то же расстояние. А2. Осевая симметрия (отражение в прямой) — преобразова- ние, при котором каждой точке данной фигур М ставится в со- ответствие такая точка М' (симметричная), что данная прямая (ось симметрии) является серединным перпендикуляром к от- резку ММ. АЗ. Поворот фигуры F вокруг центра О на данный угол О < ф < 180° в данном направлении — преобразование, при кото- ром каждой точке М ставится в соответствие такая точка М’, что ОМ = ОМ, /М0М = ф по величине и направлению. А4. Центральная симметрия относительно точки О — пово- рот вокруг точки О на 180°. 2. (1, 1) Б. Б1. Преобразование подобия фигуры с коэффициентом подо- бия к > 0 — преобразование, при котором любым двум точкам М и N фигуры ставятся в соответствие такие точки М' и N', что M’N’ = kMN. JIpMseaae — подобие с коэффициентом к = 1. Б2. Гомотетия с центром О и коэффициентом к не равным 0 — преобразование, при котором для любой точки М и ее образа М' 128
.4 ¥•$ ’’ ' , -* справедливо равенство ОМ = к ОМ. Центральная симметрия — гомотетия с k —1. 2. (1,1) В., Параллельное проецирование фигур на плоскость. Пусть П -- фиксированная плоскость, / — пересекающая эту плоскость прямая, Л/q ~ произвольная точка трехмерного про- странства. Проведем через точку Мо прямую параллельную I. Точка пересечения этой прямой с плоскостью П— М. Эта точка М — суть проекция точки Mq на плоскость П при проекцирова- нии' параллельно прямой /. Пусть Fq — плоская или пространст- венная фигура. Параллельные проекции всех этих точек фигуры Fq образуют некоторую фигуру F на плоскости П. Фигура F — параллельная проекция фигуры Fq. 2. (2, 2) [(3, 3)] Пересекающиеся прямые [плоскости] — прямые [плоскости], имеющие единственную общую точку [прямую]. 2. (2, 2) [(3, 3)] Перпендикулярные прямые [плоскости] — прямые [плоскости], угол между которыми прямой. 2. (2, 2) [(3, 3)] Параллельные прямые [плоскости] — пря- мые, лежащие в одной плоскости и не имеющие общих точек [плоскости, не имеющие общих точек]. 2. (2, 2) Скрещивающиеся прямые — прямые, которые не пересекаются и не параллельны -(прямые, через которые нельзя провести плоскость). 2. (4, 4) Дополнительные лучи — различные лучи одной и той же прямой, имеющие общую начальную точку. 2. (4, 4) Параллельные лучи — лучи, лежащие на параллель- ных прямых. 2. (4, 4) Два луча [АВ) и [С£) называются противонаправ- ленными, если они параллельны и лежат в разных полуплоско- стях относительно границы АС. 2. (5, 5) Отрезок АС меньше отрезка АВ (АС < АВ) если при наложении их окажется, что АС составляет часть АВ. 2. (5, 5) Равные отрезки — отрезки, имеющие одинаковую длину (совмещающиеся при наложении). 2. (5, 5) Параллельные отрезки — отрезки, лежащие на па- раллельных прямых. 2. (5, 5) Пересекающиеся отрезки — отрезки, имеющие единственную общую точку. 2. (5, 5) Пропорциональные отрезки — отрезки, длины кото- рых пропорциональны: отрезки с длинами а и b пропорцио- нальны отрезкам с длинами с и d, если а : с = b : d. 129
2. (5, 5) Перпендикулярные отрезки — отрезки, лежащие на перпендикулярных прямых. 2. (6, 6) Равные углы — углы, имеющие одинаковую угло- вую меру (или совмещающееся при наложении). 2. (6, 6) Угол АВС меньше угла KLM (ZABC < Z.KLM), если при наложении одного из них на другой так, чтобы сторона од- ного совместилась со стороной другого, а две другие были по одну сторону от совместившихся, окажется, что Z.ABC составит часть Z.KLM (LM совместили с ВС). 2. (6, б) Два угла называются смежными, если у них одна сторона общая, а другие стороны являются дополнительными лучами. 2. (6,6) Два угла называются вертшальными, если стороны одного из них являются дополнительными лучами сторон дру- гого. 2. (6, 6) Углы, образованные при пересечении двух прямых а и Ь, лежащих в одной плоскости, третьей прямой с (секущей) имеют следующие названия (см. Рис. 85): соответственные 1 и 5, 4 и 8, 2 и 6, 3 и 7; — внутренние накрест лежащие 4 и 6, 3 и 5; — внешние накрест лежащие 1 и 7, 2 и 8; — внутренние односторонние 4 и 5, 3 и 6; — внешние односторонние 1 и 8, 2 и 7. — дополнительные углы — углы, имеющие одну общую сто- рону, сумма которых равна 90°. 2. (7, 7) — (10, 10) Равенство [подобие]. Фигура Ф\ равна [подобна с коэффициентом Л] фигуре Ф^, если одна из них является образом другой при движении [преобразовании подобия с коэффициентом к}. Две плоские [пространственные] фигуры называются равно- великими, если они имеют равные площади [объемы]. Две фигуры называются равносоставленными, если их можно разбить на соответственно равные фигуры. 130
2.(1, 2) Точка 2.(2, 8) Прямая 2.(2, 3) Прямая 2.(6, 3) Одномерный угол 2.(3, 8) Плоскость 2.(3, 4) Плоскость « S м м Ом прямую на две полупрямых (луча) окружность на две дуги плоскость на две полуплоскости плоскость на два плоских угла сферу на два сферических сег- мента пространство на два полупро- странства 2.(1, 2, 3) 2.(1, 2, 2) 2.(1, 2, 2) 2.(1, 3, 3) 2.(1, 2, 8) СУЩ ЕСТВУЕТ ЕДИНСТВЕННАЯ плоскость прямая прямая плоскость окружность ПРОХОДЯЩ АЯ ЧЕРЕЗ точку и прямую точку, параллельно данной прямой точку, перпендикулярно данной прямой точку, не принадлежа- щую данной плоскости и параллельная данной плоскости три не коллинеарные точки 2.(1, 1) 1.(1, 2) 2.(2, 2) 2.(2, 2) 2.(1, 3) 2.(2, 3) 2.(3, 3) щ S д о? О н и и «3 Ой между точками А и В — длина отрезка АВ. от точки до прямой — длина перпендикуляра, опу- щенного из данной точки на данную прямую. между параллельными прямыми — расстояние от какой-нибудь точки одной прямой до другой прямой. между скрещивающимися прямыми — длина их общего перпендикуляра. от точки до плоскости — длина перпендикуляра, опущенного из этой точки на плоскость. от прямой до параллельной ей плоскости — расстоя- ние от любой точки этой прямой до плоскости. между параллельными плоскостями — расстояние от какой-либо точки одной плоскости до другой плоскости. 131
2.(1, 3) — проекция точки пространства на плоскость при параллельном проецировании. 2.(1, 4) — начало луча (граница луча). 2.(1, 5) — концы отрезка. 2.(1, 5) — внутренние точки отрезка. tr о н в? Си, С — начало и конец направленного отрезка. 2.(1, 6) — вершина линейного [плоского, многогранно- го] угла. 2.(1, 7) — вершина «-угольника [л-гранника]. 2.(1, 7) — ортоцентр треугольника (точка пересечения высот треугольника или их продолжений). 2.(1, 7) — центр симметрии л-угольника [л-гранника]. 2.(1, 8) — центр окружности [сферы]. 2.(1, 8) — центр круга [шара]. 2.(1, 8) — точка окружности [сферы] -граничная точка. 2.(1, 8) — внутренняя точка круга [шара]. 2.(1, 8) — диаметрально противоположные точки шара. 2.(1, 9) — вершина конуса. 2.(1, 9) — центр основания конуса [усеченного конуса]. 2.(1, 9) — внутренняя (граничная) точка конуса (принад- лежащая боковой поверхности или основанию конуса [усеченного конуса]). 2.(1, 10) — центр нижнего [верхнего] основания цилиндра. 2.(1, 10) — внутренняя [граничная] точка цилиндра. 2.(1,11) — имеет координаты на координатной прямой [плоскости, в пространстве]. 2.(2, 3) — разбивает плоскость на две полуплоскости. 2.(2, 3) — лежит [параллельна, пересекает, перпендику- лярна] в плоскости. 2.(2, 4) — граница полуплоскости. 2.(2, 4) — содержит [параллельна, пересекает, перпенди- кулярна] луч [лучу], у 2.(2, 5) — содержит'[пересекает, параллельна, перпенди- кулярна] отрезок [отрезку]. 132
2X2, 5) — медиатриса (серединный перпендикуляр) — прямая, перпендикулярная к отрезку и проходящая через его середину. 2.(2, 6) — ребро двугранного угла. 2.(2, 6) — содержит стороны развернутого угла. 2.(2, 6) — составляет угол с прямой [плоскостью]. 2.(2, 6) — ось симметрии угла. 2.(2, 7) — пересекает стороны одномерного «-угольника [грани т-гранника]. 2.(2, 7) — ось симметрии «-угольника [m-гранника] (ось * симметрии фигуры F — прямая, преобразование симмет- рии относительно которой переводит фигуру в себя). 2.(2, 8) — касательная [секущая] к окружности. 2.(2, 8) — ось симметрии окружности [круга]. 2.(2, 8) — ось симметрии сферы [шара]. 2.(2, 8) — ось вращения сферы [шара]. 2.(2, 9) [(2, 10)] — ось конуса [цилиндра]. д 2.(2, 9) [(2, 10)] — образующая конической [цилиндри- ческой] поверхности. 2.(2, 9) [(2, 10)]— ось симметрии прямого кругового ко- нуса [цилиндра]. 2.(2, 11) — на координатной плоскости задается уравне- нием Ах + By + С = 0. Л Н и о м и о ч и 2.(3, 4) — граница полупространства. 2.(3, 5) — содержит [параллельна, пересекается, перпен- дикулярна] отрезок [отрезку]. 2.(3, 6) — содержит грань двугранного [многогранного] угла. 2.(3, 6) — содержит грань многогранника. 2.(3, 6 - 10) — является плоскостью проекции [проектиру- ющей плоскостью] плоских и пространственных фигур. 2.(3, 7) — плоскость многоугольника [грани многогран- ника [основания пирамиды, призмы]. 2.(3, 7) — секущая плоскость. 2.(3, 7) — плоскость симметрии многогранников. 133
л и о и о ч й 2.(3, 7) — диагональная плоскость многогранника — плоскость проходящая через диагонали или диагональ. 2.(3, 8) — касательная плоскость сферы в точке. (Плоскость, проходящая через точку А шаровой поверхно- сти и перпендикулярная радиусу, проведенному в точку А, называется касательной плоскостью. Точка А — точка каса- ния). 2.(3, 8) — диаметральная (плоскость, проходящая через центр шара. Сечение шара диаметральной плоскостью называется большим кругом, а сечение сферы — большой окружно- стью). 2.(3, 9) [(3, 10)] — касательная плоскость к Конусу [цилиндру] — плоскость проходящая через образующую конуса [цилиндра] и перпендикулярная плоскости осе- вого сечения, содержащей эту образующую. 2.(3, 9) [(3, 10)] — секущая. Сечение конуса [цилиндра] плоскостью, проходящей через вершину конуса [параллельно оси цилиндра]. Представляет собой равнобедренный тре- угольник'[прямоугольник]. Две его стороны — образующие конуса [цилиндра], а остальные — хорды оснований. 2.(2, 1 - 10) — плоскость проекции при проектировании пространственных фигур. 2.(3, 11) — в пространственной системе координат имеет уравнение Ах+Ву + Cz+ D = 0. 2. (4, 5>— содержит отрезок. 2. (4, 6) — сторона линейного (плоского) угла. 2. (4, 6) — биссектриса линейного (плоского) угла. 2. (4, 6) — ребро многогранного угла. 2. (4, 7) — сторона угла одномерного [плоского] л-угольника. 2. (4, 8) — сторона вписанного угла. 2. (4, 8) [(4, 7)] — сторона центрального угла (плоского угла с вершиной в центре окружности) [грань дву- гранного угла]. 134
О со М Н О 2.(5,4) — пересекает границу полуплоскости [полупространства] 2.(5. 6) — поперечный отрезок плоского [двугранного] угла (От- резок, концы которого принадлежат сторонам [граням] угла). 2.(5, 7) — сторона п-угольника. 2.(5, 7) — медиана [биссектриса] треугольника. 2.(5, 7) — средняя линия треугольника [трапеции]. 2.(5, 7) — высота треугольника [трапеции, параллелограмма, параллелепипеда, призмы, пирамиды, усеченной пирамиды]. 2.(5, 7) — диагональ многоугольника [многогранника]. 2.(5, 7) — ребро многогранника. 2.(5, 7) — звено-сторона ломаной линии. 2.(5, 7) — гипотенуза [катет] — сторона прямоугольного треугольника, лежащая против прямого [острого] угла. 2.(5, 7) — апофема правильного многоугольника — отре- зок перпендикуляра, опущенный из центра многоуголь- ника на одну из его сторон. 2.(5, 7) — апофема правильной пирамиды [правильной усеченной пирамиды] — высота боковой грани. 2.(5, 8) — хорда —отрезок, соединяющий две точки ок- ружности [сферы]. 2.(5, 8) — диаметр окружности [сферы] — хорда проходя- щая через центр. 2. (5, 8) — радиус — отрезок соединяющий центр с какой- либо точкой окружности [сферы]. 2. (5, 9) [(5, 10)] — высота конуса, усеченного конуса [цилиндра] — перпендикуляр, опущенный из вершины конуса [из центра верхнего основания] на плоскость ос- нования [нижнего основания]. 2. (5, 9) — образующая кругового конуса — отрезок, соеди- няющий вершину конуса с точкой окружности основания. 2. (5, 10) — образующая кругового цилиндра — параллель- ные отрезки соединяющие соответственные при параллель- ном переносе точки окружностей оснований цилиндра. 2. (5, 9) [(5, 10)] — радиус, хорда, диаметр конуса [цилиндра] радиус, хорда, диаметр оснований. 2. (5, 11) — длина — мера отрезков, ставящая в соответст- вие равным отрезкам равные числа и некоторому фикси- рованному (единичному) отрезку — число 1. 2. (5, 11) — длина отрезка, заданного координатами кон- цов А(х\, yi) и В(х2, yi) вычисляется по формуле: I АВ I = ^(х2-х1)2 + (у2 - у,)2. 185
2.(6, 2) — между пересекающимися прямыми — угловая мера меньшего из смежных углов, образованных при пе-, ресечении прямых. 2.(6, 2) — между скрещивающими прямыми — угол меж- ду , пересекающимися прямыми, которые - параллельны данным скрещивающимся прямым. 2.(6, 5) — между ненулевыми векторами — величина об- разуемого ими угла, когда они отложены от одной точки. 2.(6, 3) — между двумя пересекающимися плоскостями — угол между пересекающимися прямыми, полученными в сечении данных плоскостей плоскостью, перпендикуляр- ной линии их пересечения. (Линейный угол меньшего из двугранных углов получен- ных при пересечении плоскостей). 2.(6, 2, 3) — между прямой и плоскостью — угол между прямой и ее прямоугольной проекцией. 2.(6, 7) — внутренний угол выпуклого м дьвша — угол, образованный лучами, содержащими две соседние стороны. . 2.(6, 7) — внешний угол выпуклого многоугольника при данной вершине — угол, смежный внутреннему углу при этой вершине. 2.(6, 8) — центральный угол в окружности — угол с вер- шиной в ее центре. Часть окружности, расположенная внутри плоского угла — дуга окружности, соответствую- щая данному центральному углу. 2.(6, 8) — вписанный в окружность — угол, вершина которого лежит на окружности, а стороны пересекают эту окружность. 2.(6, 6) — центральный угол, соответствующий вписан-' ному углу — тот угол, который соответствует дуге, не со- держащей вершины вписанного угла. 2.(6, 8) — описанный — угол образованный двумя каса- тельными к окружности, пересекающимися в одной точке. 2.(6, 11) — градусная и радианная мера угла. — Радианная мера — отношение длины соответствующей дуги к радиусу окружности: I.R = пп/180 (то есть градус- ная мера умножается на л/180°. Единица радианной меры угла — радиан (угол в один ра- диан — угол, у которого длина дуги равна радиусу). 1 радиан = 180°/л = 57°. 2.(6, 11) — тригонометрические Функции: sina, cosa, tga. 136
ОГОУГОЛЬНИК И МНОГОГРАННИК 2,(7, 7) — плоский многоугольник — грань многогран- ной Поверхности [многогранника]. , Примечание: Так как у многогранников (в отличие от многоугольников) число граней не совпадает с числом вершвд, то для удобства будем называть их двойным име- нем: ст-гранник или л-вершинник. Так, например, куб яв- ляется шестигранником и в то же время восьмивершинни- ком, а октаэдр, наоборот, является восьмигранником и шестивершинником. Для общности и многоугольники можно называть «-угольниками (ст-вершинниками). 2.(7, 7) [(7, 7)] Выпуклый л-угольник F; [л-вершинник Ff] называется вписанным в выпуклый л-сторонник F Гст-гоанник F1. если все вершины л-вершинника F{ [Ff] принадлежат сторонам F [граням Р]. 2.(7', 7') [(7', 7')[ Выпуклый л-сторонник F [ст-гранник F] называется описанным около л-вершинника Fx [ст-вер- шинника Fj'l. если все л сторон F [ст граней F] прохо- дят через все л вершин Fi [ст вершин Ff]. 2.(7, 8) [(7, 8)] Многоугольник [многогранник] называ- ется вписанным в окружность [сферу], если все его вершины лежат на окружности [сфере]. 2.(8, 7) ((8, 7)] Окружность [сфера] (если они существу- ют!) в этом случае называются описанными около многоугольника [многогранника]. 2.(7, 8) [(7, 8)] Многоугольник [многогранник] называ- ется описанным около окружности [сферы], если все его стороны [грани] касаются окружности [сферы]. 2.(8, 7) [(8, 7)] Окружность [сфера] (если они существу- ют!) в этом случае называются вписанными в много- угольник [многогранник]. 2.(7, 9) Пирамида, вписанная в конус — пирамида, основа- ние которой есть многоугольник, вписанный в окружность основания конуса, а вершиной является вершина конуса. 2.(7', 9') Пирамида, описанная около конуса — пирамида, основание которой — многоугольник, описанный около ос- нования конуса, а вершина совпадает с вершиной конуса. 2.(7, 9) Призма, вписанная в конус — призма, вершины одного из оснований которой лежат на боковой поверх- ности конуса, а плоскость другого основания совпадает с плоскостью основания конуса. 137
2.(7, 10) Призма, вписанная в цилиндр — призма, осно- ванием которой служит многоугольник, вписанный s i s 2 основание цилиндра, а высотой — высоту цилиндра. 2.(7, 10) Призма, описанная около цилиндра — призма, боковые грани которой касаются цилиндрической по- верхности, а плоскости оснований совпадают. 2.(7, 11) — (10, 11) Длина [площадь, объем! — мера от- резков, окружности и ее дуг [плоских фигур, простран- ственных тел], ставящая конгруэнтным отрезкам, ок- ружностям и их дугам [плоским фигурам, пространст- венным телам] равные числа и некоторому единичному отрезку [квадрату, кубу] — число 1. 2.(7, 11) Длины сторон. Диагоналей многоугольников, площади плоских многоугольников, длины линейных элементов многогранников, площади многогранных по- верхностей, объемы многогранников. >» Рц М S ив Н U О и и 0Й м о 0LI В S ◄ 0L| ы е и 2.(8, 8, 3) — Окружность [круг] — сечение сферы [шара] плоскостью. 2.(8, 9, 3) — Окружность [круг] сечение поверхности кругового конуса [конуса] плоскостью, параллельной его основанию. 2.(8, 9) Сфера [шар] называется вписанной [вписанным! в конус, если сфера касается основания конуса и каж- дой его образующей. 2.(8, 9) Сфера называется описанной около конуса. если основанием конуса является сечение шара, а вершина конуса принадлежит сфере. (Конус — впи- санным в сферу). 2.(8, 10, 3) Окружность [круг] — сечение поверхности кругового цилиндра [цилиндра] плоскостью, парал- лельной основаниям цилиндра. 2.(8,10) Сфера [шар] называется вписанной в ци- линдр. если она касается оснований цилиндра и каждой образующей (цилиндр — описанным око- ло шара). 138
£ 04 И s *4 U О К И >» 0М и о 2.(8, 10) Сфера называется описанной около ци- линдра, если основания цилиндра являются сече- ниями шара, ограниченного сферой, плоскостями оснований. 2.ф, 11) Длина окружности и ее дуг — I = 2nR, /(п°) = ~£\п’ плош^ круга и его частей: = л/?, — ——а; площадь сферы и ее частей: = 4л/?, 360 4 1 ‘Ххг = 2nRH; объем шара и его частей: V = — л/? , 2 Усы = уя/?* 2Я, где Я — высота шарового сегмента, составляющего часть сектора. 2.(9, 10) Пересечением конуса и цилиндра с парал- лельными основаниями является круг. 2.(10, 9) Цилиндр называется вписанным в конус. если одно из оснований цилиндра — концентриче- ский круг с основанием конуса, а второе — сечение конуса плоскостью этого основания цилиндра. очи 2.(9, И) Площадь боковой и полной поверхности конуса, усеченного конуса. 2.(9, 11) Объем конуса и усеченного конуса. 2. (10, 11) Площадь боковой и полной поверхности цилиндра. Объем цилиндра. 139
В. ТЕОРЕМЫ О СВОЙСТВАХ, ПРИЗНАКАХ ОБЪЕКТОВ И ОТНОШЕНИЙ 3. (1, 1) Свойства движения и подобия фигур в плоскости и пространстве. 3. (1, 1). 1. [Г] Два движения [подобия], выполненные по- следовательно дают снова движение [подобие] в плоскости и пространстве. 3. (1, 1). 2. [2'] При движении [подобии] точки, лежащие на прямой переходят в точки, лежащие на прямой, и сохраняется порядок из взаимного расположения. 3. (1, 1). 3. [3'] При движении [подобии] прямые переходят в прямые, полупрямые — в полупрямые, отрезки — в отрезки, плоскости — в плоскости. 3. (1, 1). 4. [4'] Движение [подобие] сохраняет углы между полупрямыми, соответственные отрезки равных фигур [подобных фигур] равны [пропорциональны]. 3. (1, 1). 5. [5'] Параллельный перенос в плоскости [в про- странстве] переводит прямую [любую прямую и любую плос- кость] в параллельную ей прямую (или в себя) [в параллельную ей прямую (или в себя), в параллельную ей плоскость (или в себя)]. 3. (1, 1). 6. [6'] Гомотетия в плоскости [в пространстве] пе- реводит любую прямую [прямую и плоскость], не проходящую через центр гомотетии в параллельную прямую [в параллельную прямую и параллельную плоскость]. Для успешного решения геометрических задач, планиметри- ческих и в особенности стереометрических, большую роль игра- ет правильно и наглядно выполненное изображение фигуры. Хотя изображение и не может заменить логического обоснова- ния геометрического факта, оно помогает разобраться в условии задачи, может натолкнуть на мысль о выборе для использования конкретного фактического материала или рационального до- полнительного построения. Изображением Фигуры называют любую фигуру, подобную параллельной проекции данной фигуры на некоторую плос- кость. Правильные изображения выполняются на основе свойств параллельного проектирования. Чаще всего использу- ются следующее свойства параллельного проектирования (при условии, что прямые и отрезки не параллельны направлению проектирования). 140
3. (2, 2'). Проекция прямой — прямая. 3. (5, 5') Проекция отрезка — отрезок. 3. ((5, 5), (5, 5')) Проекция параллельных отрезков — парал- лельные отрезки или отрезки, принадлежащие одной прямой. 3. ((5, 5), (5, 5')) Проекция параллельных отрезков, а. также проекции отрезков, лежащих на одной прямой, пропорциональ- ны самим отрезкам. (В частном случае: проекция середины от- резка — середина проекции отрезка). Изображение частных видов плоских Фигур 3. (7, 7') Изображение данного треугольника — произволь- ный треугольник. 3. (7, 7') Изображение любого параллелограмма (в том числе прямоугольника, ромба, квадрата) — произвольный параллело- грамм. 3. (7, 7') Изображении трапеции — трапеция с тем же от- ношением оснований. 3. (8, 8') Изображение окружности — эллипс, причем, изо- бражением центра окружности является центр эллипса. * Из свойств параллельного проектирования следует: 3. ((5, 5), 8) Изображением двух взаимно перпендикулярных’ диаметров окружности являются диаметры эллипса, каждый из которых проходит через середины хорд, параллельных другому диаметру (такие диаметры эллипса называют взаимно сопря- женными). 3- (7дь 8) Изображение катетов описанного около окружно- сти прямоугольного треугольника параллельны сопряженным диаметрам эллипса. 3- (7дь 8) Изображение гипотенузы вписанного прямо- угольного треугольника — произвольный диаметр эллипса. 3. (7Б 1д, 8) Изображение вписанной в окружность рав- нобедренной трапеции — всякий вписанный в эллипс четы- рехугольник с параллельными неравными противополож- ными сторонами. 3- (7Б 2д, 8) Изображение вершин вписанного прямоуголь- ника — концы двух произвольных диаметров эллипса. 3- (7Б 2б» 8) Вершинами изображения вписанного квадрата являются концы двух взаимно сопряженных диаметров эллипса; для построения изображения описанного квадрата достаточно 141
через концы каждого диаметра провести прямые, параллельные другому диаметру. 3. 7A'i Изображение тетраэда — фигура, состоящая Из сторон ' и диагоналей любого четырехугольника. V (При выполнении изображения произвольной п-угольной пирамиды можно принимать любую точку плоскости за изобра- жение вершины и соединять ее с изображением- вершин п- угольника, выполненным по правилам изображения плоских фигур). 3.7g Для построения изображения параллелепипеда (простейшей призмы) любые три отрезка АВ, AD и АА' с общим концом А, никакие два из которых не лежат на одной прямой, можно считать изображением ребер параллелепипеда. Изобра- жение граней достраиваются по правилам изображения парал- лелограммов. 3. (1, 8 [8']) Центр круга [шара] — центр симметрии. 3. (2, 2) Две прямые параллельны тогда и только тогда, ко- гда при пересечении из третьей прямой выполняется одно из условий: а) внутренние накрест лежащие углы равны; б) сумма внутренних односторонних углов равна 180°; в) соответственные углы равны. 3. (2, 2) Если две пересекающиеся прямые параллельны двум перпендикулярным прямым, то они тоже перпендикуляр- ны. 3. ((2, 2), 2) Две прямые, параллельные третьей прямой, па- раллельны. 3. (2, 3) Если две точки прямой принадлежат плоскости, то прямая принадлежит этой плоскости. 3. (2, 3) Если прямая, не принадлежащая плоскости, парал- лельна какой-нибудь прямой в этой плоскости, то она парал- лельна и самой плоскости. 3. ((3, 3), (2. 2)) Если две пересекающиеся прямые одной плоскости соответственно параллельны двум прямым другой плоскости, то эти плоскости параллельны. 3. ((3, 3), (2, 2)) Если две параллельные плоскости пересе- каются третьей, то прямые пересечения параллельны. 3. ((3, 3), (5, 5)) Отрезки параллельных прямых, заключен- ные между двумя параллельными плоскостями, равны. 142
3. (2,3) Если прямая перпендикулярна двум пересекающим- ся прямым, лежащим в плоскости, то она перпендикулярна данной плоскости. , 3. (3, (2, 2)) Если плоскость перпендикулярна к одной из двух параллельных прямых, то она перпендикулярна и к другой. 3. ((2, 2), 3) Две прямые, перпендикулярные одной и той же плоскости, параллельны. 3. (2, 3, 5) Если прямая, проведенная на плоскости через основание наклонной, перпендикулярна ее проекции, то она перпендикулярна наклонной. И обратно: если прямая на плос- кости перпендикулярна наклонной, то она перпендикулярна и проекции наклонной (теорема о трех перпендикулярах). Чтобы прямая, проведенная на плоскости через основание наклонной к этой плоскости была ей перпендикулярна, необхо- димо и достаточно чтобы она была перпендикулярна к проек- ции этой наклонной. 3. (3, 3) Если плоскость проходит через прямую, пер- пендикулярную другой плоскости, то эти плоскости пер- пендикулярны. 3. (2, 5) Параллельные прямые, пересекающие две данные прямые, отсекают на них пропорциональные отрезки. 3. ((2, 8), (3, 8')) Если R — радиус окружности [сферы], a d — расстояние от ее центра до прямой [плоскости], то: 1) Окружность [сфера] и прямая [плоскость] пересекаются в двух точках [по окружности], при R > d. 2) Касаются при R = d. 3) Не имеют общих точек при R< d. 3. (3, 7') Плоскость, пересекающая пирамиду и параллель- ная ее основанию, отсекает подобную пирамиду. 3. (3, 8') Всякое сечение шара плоскостью есть круг. Центр этого круга есть основание перпендикуляра, опущенного из центра шара на секующую плоскость. 3. (3, 8') Любая диаметральная плоскость шара является его плоскостью симметрии. 3. (2[3], 8[8']) Любой [-ая] диаметр круга [диаметральная плоскость шара] является его осью [плоскостью] симметрии. 3. (2[3], 8[8']) Касательная прямая [плоскость] имеет с кру- гом [шаром] только одну общую точку — точку касания. 3. (3, 9) Плоскость, параллельная плоскости основания ко- нуса, пересекает конус по кругу, а боковую поверхность — по окружности с центром на оси конуса. 143
3. (3, 10) Плоскость, параллельная плоскости основания ци^| линдра, пересекает его боковую поверхность по окружности,! равной окружности основания. J 3. (4, 4) Если луч а сонаправлен с лучом Ь, а луч Ъ сона-’ правлен с лучом с, то лучи вис сонаправлены. 4 ! 3. (5, 7Бз) Чтобы четырехугольник был параллелограммом необходимо и достаточно, чтобы его диагонали точкой пересе- чения делились пополам. 3. (5, 7[7']) Точка пересечения диагоналей параллелограмма [параллелепипеда] является его центром симметрии. 3. (5, 7Бг) У параллелограмма противолежащие стороны равны [у параллелепипеда противолежащие грани параллельны и равны]. 3. (5, 7Бза) Диагонали прямоугольника равны. 3. (5, 7Бгв) Диагонали ромба пересекаются под прямым уг- лом. Диагонали ромба являются биссектрисами его углов. 3. (5, 7А[7Б1]) Средняя линия треугольника [трапеции] па- раллельна основанию [основаниям] и равны, его половине [их полусумме]. 3. (5, 7А) В любом треугольнике каждая сторона меньше суммы двух других сторон. 3. (5, 7Ai) В прямоугольном треугольнике квадрат гипотену- зы равен сумме квадратов катетов. 3. (5, 7') В прямоугольном параллелепипеде квадрат любой диагонали равен сумме квадратов трех его измерений. Следствие. Если к прямой из одной точки проведены пер- пендикуляр и наклонные, то любая наклонная больше перпен- дикуляра, равные наклонные имеют равные проекции, из двух наклонных больше та, у которой проекция больше. 3. (5, 7А) В треугольнике против большего угла лежит боль- шая сторона, против большей стороны лежит больший угол. 3. (5, 8) Если хорды АВ и CD окружности пересекаются в точке S, то AS BS ~ CS DS. Если из точки Р к окружности проведены две секущие, пе- ресекающие окружность в точках А, В и С, D соответственно, то АРBP = CPDP. (В предельном случае, когда секущая займет положе- ние касательной, то есть С совпадет с D, то будем иметь АРВР= СРСР= СР2). 3. (6, 6) Сумма смежных углов равна 180°. Вертикальные уг- лы равны. 144
3. (6,7) Сумма углов выпуклого n-угольника равны 180° (л - 2). Сумма внешних углов выпуклого л-угольника, взятых по одному при каждой вершине равна 360°. 3. (6, 7А) Треугольник является равнобедренным тогда и только тогда, когда он имеет два равных угла. 3. (6, 7А)^Ьнешний угол треугольника равен сумме двух внутренних углов, не смежных с ним. 3. (6, 7Б2) У параллелограмма противолежащие углы равны. 3. (6, 8) Угол, вписанный в окружность равен половине со- ответствующего центрального угла. 3. (7, 7А) Признаки равенства [подобия] треугольников. 3. (7, 7А). 1[1'] Еслй две стороны одного треугольника соот- ветственно равны [пропорциональны] двум сторонам другого треугольника и углы, образованные этими сторонами, равны, то треугольники равны [подобны]. 3. (7, 7А). 2[2'] Если три стороны одного треугольника соот- ветственно равны [пропорциональны] трем сторонам другого треугольника, то такие треугольники равны [подобны]. 3. (7, 7А). 3. Если сторона и прилежащие к ней углы одного треугольника равны соответственно стороне и прилежащим к ней углам другого треугольника, то такие треугольники равны. 3. (7, 7А) [3'] [Если два угла одного треугольника равны двум углам другого треугольника, то такие треугольники подобны]. 3. (7, 7) Правильные выпуклые n-угольники подобны. 3. (7-10, 7-10) Площади подобных фигур относятся как квадраты их соответствующих размеров. 3. (7, 8) Правильный выпуклый многоугольник является вписанным в окружность и описанным около окружности. 3. (7, 8) Центр окружности, описанной около треугольника — точка пересечения серединных перпендикуляров к сторонам. 3. (8, 8) Две окружности не могут иметь более двух об- щих точек. 3. (8х, 8') Линия пересечения двух сфер есть окружность. 3. (Г, 11) Если А (хь yh zi), В (х2, у2, z2), О (xq, Уо, zb) — се- редина [4В], то _ X, +х2 Ух +у2 Zx +Z2 *0 -Хц 2 9 У о ”* 2 9 ~ 2 * б) I АВ I = 7(х2 “ х1)2 + (У2 - У1)2 + (z2 -Zx)2. 145
Если А' (х[, у{, z{) и В’(х{, yi, zi) — образы точек А и В прш движении, то I А'В' I = I АВ 1. 1 в) х' = х + а, у' = у + Ь, г' = z + с — параллельный перенос^ на вектор п (а, Ь, с). ! 3. (2, It) ах + by + с = 0 — уравнение прямой на коорди- натной плоскости. 3. (3, 11) ах + by + cz + d = 0 — уравнение плоскости в ко- ординатном пространстве. 3. (8 [8'1, 11) (х - ль)2 + (у - Уо)2 = Н(х “ *о)2 + (У “ Jo)2 + + (z ~ Zo)2 = г2] — уравнение окружности [сферы] с центром О (л&, Уо, Zo) и радиусом г в прямоугольной системе координат на плоскости [в пространстве]. 3.(7', 11) Для прямоугольного параллелепипеда d2 = а2 + Ь2 + с2. 3. (7А, 11) а2 = Ь2 + с2 — 2 b с cosH — теорема косинусов, а b с ------ =---- = ——— — теорема синусов, sm A sin В sin С 3. (7', И) Площадь ортогональной проекции многоугольни- ка на плоскость П: $ = Sj л . cos у- Л{Л2-..Л^ Л1Л2"ЛЛ ’ 3. (5, 11) Скалярное произведение векторов а и Ь: А юи<о,7>- а- b а b Для векторов, заданных координатами: a(qifa2,a3) + b(йц,^,^) = с (ot + 1\, + Aj, а3 + й^) а а = (00^002,003). 3. ((7, 8), 11) Формулы для радиусов вписанных и описан- ных окружностей правильных многоугольников: а) л = 3 R= , г б) я = 4 Л=-^, г в) п = 6 R = а, г = . 146
3. ((7, 8), 11) Радиусы описанной (R) и вписанной г окруж- ностей любого треугольника вычисляются по формулам: „ abc 2S R = Т7Г » Г ----1----• 45 а + b + с 3. (7А, И) Площади многоугольников: s д = | ah, , 5Д = | ab sin С, 5Д = 7/КР - «)(/»-*)(/» - <0 • 3- (7'[9], 11) Объем любой пирамиды [конуса] равен одной трети произведения площади основания на высоту: V = | SH. Объем усеченной пирамиды [усеченного конуса] вычисляется по формуле V= | h(0i + 7<?i<?2 + Сз) lv= + Л1Л2 + %)• 3. (7'[10], 11) Объем любой призмы [цилиндра] равен про- изведению площади основания на высоту V = Sh. 3. (8. 11) Длина окружности / = 2nR, Площадь круга 5 = л/?2, Площадь сектора S = —— • а, 360 Площадь сегмента S — . а ± 5Д. 3. (8, 11) Площадь сферы S = 4лЛ2. Площадь сферического сегмента ScerM = 2nRH. Объем шара Уш_ = 4/3 п№. Объем шарового сектора Усект. = 2/3 лЛ2Я. Объем шарового сегмента Усещ. = xlPiR — Н/У), где Н — высота сегмента. Общая формула для объемов тел вращения (части тела, за- ключенной между параллельными плоскостями х = а и х = в) b b У(а) — У(Ь) = J V'(x)dx =J n2f(x)dx,a < b. a a 3. (9, 11) Площадь боковой поверхности конуса [усеченного конуса] вычисляется по формулам:. S = 1/2 xRl [ = л(/?1 + Я2)1]. ПРИМЕЧАНИЕ. Нами сделана попытка компрессии (сжатия) фактического материала отнюдь не из-за экономии бумаги. Работа над так изложенным материалом позволит пользова- телю не только воспроизвести нужные факты, но и глубже про- 147
никнуть в их структуру, увидеть общее и особеннее как в раз- личных фигурах, так и в отношениях между ними, приучит гра- мотно строить определения понятий и теоремы о свойствах и признаках различных объектов. , Компрессия осуществлялась в следующих направлениях. Слово, которое является определяющим для большого числа понятий, связанных с самыми различными фигурами, напечата- но ВЕРТИКАЛЬНО слева (см., например, ТОЧКА, ОТРЕЗОК, УГОЛ...). Это дает возможность увидеть значимость ключевых понятий-объектов для перечисления выполняемых ими функ- ций. Аналогично выделены и некоторые важные отношения между объектами: РАЗБИВАЕТ, СУЩЕСТВУЕТ И ЕДИНСТ- ВЕННАЯ (НЫЙ) и др. Там, где справедливы взаимно-обратные утверждения, мы формулировали их в терминах необходимости и достаточности. Определения понятий аналогичных объектов или аналогич- ных отношений между одинаковыми объектами записаны одним ОБЪЕДИНЕННЫМ предложением с использованием квадрат- ных скобок. Это дает возможность при необходимости выделить каждое из них, но вместе с тем позволяет уловить внутренние существенные связи между ними и одинаковую структуру фор- мулировки определения или утверждения. Приложение 4. МЕТОДЫ РЕШЕНИЯ ГЕОМЕТРИЧЕСКИХ ЗАДАЧ Использование различного математического аппарата для I общего метода решения задач в широком смысле слова. Синтетический метод основан на непосредственном рас- j смотрении данных в задаче фигур и сопоставлении их с иско- мыми фигурами или отношениями между ними. При этом ме- ; тоде каждая задача требует своего подхода. Аналитический метод (метод координат), при котором ис- : следование геометрических фигур сводится к исследованию свя- < зей между координатами точек этих фигур, в применении ал- ' гебры к геометрии. Сила метода координат в приложении к Я геометрии — в его алгоритмичности. Успешность в применении — Я в умении рационально выбрать систему координат. Я 148
Векторный метод состоит в применении аппарата вектор- ной алгебры для решения задач геометрии, физики, техники и др. Техника его применения аналогична применению алгебраи- ческого метода при решении текстовых задач, в том числе и геометричесюйх. Первый этап: введя векторы, записать условие задачи в век- торной форме. Второй этап: решить задачу в векторном виде, используя аппарат векторной алгебры. Третий этап: полученным векторным соотношениям дать толкование в исходных терминах. Метод преобразований состоит в том, что, кроме данных в условии задачи фигур, рассматриваются вспомогательные фигу- ры, полученные из данных фигур или их элементов при помощи какого-либо частного вида преобразования (осевой симметрии, параллельного переноса, гомотетии и др.). Алгебраический метод состоит в следующем: задачу форму- лируют так, чтобы в качестве данных и искомых фигур были отрезки. Используя теоремы о свойствах фигур, выражают ис- комый отрезок через данные в виде формулы и по этой формуле находят этот отрезок. Метод геометрических мест. Обычно в планиметрии задачу сводят к нахождению одной точки, удовлетворяющей каким- либо двум условиям, вытекающим из условия задачи. Как пра- вило, одному условию удовлетворяет одна линия, другому — другая. Тогда искомая точка является точкой пересечения этих линий. Обобщение этого метода состоит в сведении задачи к нахождению не только точки, но и другой фигуры. Например, окружность в пространстве можно рассматривать как пересече- ние плоскости и сферы, а прямую — как пересечение двух плоскостей. Применение тригонометрии к решению геометрических задач. Сущность метода, как и алгебраического, в составлении форму- лы выражающей зависимость искомых отрезков (или углов) от данных отрезков (или углов), только формула эта содержит кроме отрезков тригонометрические функции углов. Применение начал анализа к решению геометрических задач. В формулах, связывающих геометрические величины (длину, меру уг- ла, площадь, объем) представляют одну из искомых переменных ве- личин как функцию другой переменной величины и, исследуя эту функцию средствами анализа, находят нужные значения. 149
Содержание Аннотация..............i......................3 От автора.....................................5 I часть. СОВЕРШЕНСТВОВАНИЕ КУЛЬТУРЫ УЙА (умственная гимнастика, разминка для решения геометрических задач).........................11 II часть. ПОДУМАЕМ ВМЕСТЕ (искусство поиска решения задач)..............23 А. Общие рекомендации....................... 23 Б. Примеры поиска.......................... 23 III часть. ЗАДАЧИ - ВСЕМОГУЩИЙ КОНТРОЛЕР И РЕПЕТИТОР....................................47 1. Контрольный минимум для проверки геометриче- ских знаний..................................47 2. Задачи-спутники, помогающие выбрать нужные знания....56 3. Решения конечных задач-спутников, призванных вос- становить цепочку знаний и привести к решению контрольных задач...............................66 4. Дополнительные задачи для тренировки...........73 IV часть. СПРАВОЧНЫЙ МАТЕРИАЛ.....................118 Приложение 1. Структурная таблица-матрица........118 Приложение 2. Аксиоматики геометрии..............120 Приложение 3. Фактический материал................123 А. Характеристика объектов.....................123 Б. Характеристика отношений 2-х объектов........128 В. Теоремы о свойствах, признаках объектов и отношений.... 140 Приложение 4. Методы решения геометрических задач..148 150
Учебное издание Фнскович Татьяна Терентьевна Геометрия без репетитора (Испытание геометрической эрудиции) Серия: В помощь поступающим в вузы (естественные дисциплины) Зав. издательским отделом Воронцова Е.В. Редактор Фискович Т.Т. Корректор Огородников А.Я. Компьютерный набор Мась А.В. Издательский отдел Учебно-научного центра довузовского образования МГУ им. М.В. Ломоносова 119899, Москва, Ленинские горы, МГУ, сектор «Ж», к. 105а Тел./факс (095) 939-39-34, 939-10-93 ЛР № 063833 от 28.12.94. Подписано в печать 17.10.97. Формат 60X90/16. Бум. Типографская. Усл. печ. л. 9,5. Тираж 7000 экз. Заказ № 1224. Подготовка к печати ТОО «Экономика и информатика» 107564, Москва, 1-я Мясниковская ул., 16. Отпечатано с готовых диапозитивов в ОАО «Ярославский полиграфкомбинат» 150049, Ярославль, ул. Свободы, 97.
Московский государственный университет имени М.В.Ломоносова \ А/ Учебно-научный центр V—довузовского образования Подготовительные вечерние и заочные курсы для абитуриентов МГУ и других вузов Москва, 119899, Ленинские горы, МГУ, Главное здание, сектор “Ж”, комн. 1056, тел. 939-21-37,939-33-10 Лекторий выходного дня — воскресные и субботние лекции по предметам вступительных экзаменов Москва, ул. Моховая, д.9 (корпус ф-та журналистики МГУ), комн. 302, тел. 203-98-12 Начальные школы МГУ — Мини-лицей “Чашниково” (полный пансион) и Начальная гимназия (полупансион) обучение детей 6-9 лет тел. 939-10-93 Вечерняя гимназия МГУ — специализированное обуче- ние школьников Москва, 119899, Ленинские горы, МГУ, Главное здание, сектор “Ж”, комн.106-а, тел. 939-39-34 Предметные олимпиады и Школы юных при факульте- тах университета Москва, 119899, Ленинские горы, МГУ, Главное здание, сектор “В”, комн.107, тел. 939-17-00 Научно-исследовательская часть (НИЧ) — разработка и внедрение новых методов обучения с использованием компьютерной техники Москва, 119899, Ленинские горы, МГУ, Главное здание, сектор “Ж”, комн.33а, (цоколь) тел. 939-10-93 Издательский отдел — издание и реализация учебной и учебно-методической литературы для абитуриентов и старших школьников Москва, 119899, Ленинские горы, МГУ, Главное здание, сектор “Ж", комн.105а,тел./факс 939-39-34,939-10-93 e-mail: izdat@esc.msu.ru http://www. esc.msu.ru